FISDAP PARAMEDIC FINAL LATEST EXAM 2023-2024/PARAMEDIC FISDAP FINAL EXAM TEST BANK REAL EXAM QUESTIONS AND CORRECT ANSWERS|AGRADE

fisdap paramedic final exam answers
Fisdap paramedic final exam questions
fisdap paramedic readiness exam 4 answers
fisdap paramedic readiness exam 5 quizlet
fisdap paramedic practice exam
fisdap paramedic final exam quizlet
fisdap paramedic entrance exam quizlet
fisdap paramedic readiness exam 5 answers

You have received an order for a dopamine drip. You carry 500cc bags of D5W. How much dopamine should you add to the bag to create a 1600mcg/1mL dilution?
A. 800mg
B. 1600mcg
C. 1600mg
D. 400mcg
A. 800 mg

  1. During the absolute refractory period, what happens to the cardiac cell?
    A. It is unable to be stimulated.
    B. The cell is vulnerable to partial stimulation.
    C. Potassium is being pumped out of the cell.
    D. Calcium has just rushed into the cell.
    A. It is unable to be stimulated.
  2. A 29 year old female fell from an 18 foot roof. During your rapid trauma assessment, where are you most likely to find life-threatening injuries?
    A. Head, neck, extremities
    B. Head, chest, abdomen
    C. Abdomen, pelvis, arms
    D. Chest, pelvis, extremities
    B. Head, chest, abdomen
  3. Unequal pupils are a result of pressure on which structure?
    A. Central medulla
    B. Trochlear membrane
    C. Vagus nerve
    D. Occulomotor nerve
    D. Occulomotor nerve
  4. What is an abnormally high level of carbon dioxide in the blood called?
    A. Pneumonia
    B. Hypoxia
    C. Hypercapnia
    D. Emphysema
    C. Hypercapnia
  5. What is oxygen deficiency in the body called?
    A. Hypercarbia
    B. Hyperoxia
    C. Hypoxia
    D. Hypoxemia
    C. Hypoxia
  6. What is your initial priority at a trench collapse?

A. Establishing medical command
B. Maintaining control of the crowd
C. Preventing a secondary collapse
D. Protecting the patient’s airway
A. Establishing medical command

  1. A 23 year old male complains of neck stiffness and flu symptoms for the past few days. He also describes a high fever with a severe headache, dizziness and blurred vision. What should you suspect?
    A. Hemorrhagic stroke
    B. Migraine
    C. Meningitis
    D. Influenza
    C. Meningitis
  2. What causes an emphysema patient to have a pink appearance?
    A. Elevated PCO2 level
    B. Polycythemia
    C. Anemia
    D. Elevated PO2 level
    B. Polycythemia
  3. Which of the following is a musical, squeaking or whistling sound heard in inspiration and expiration while auscultating lung fields?
    A. Wheezing
    B. Snoring
    C. Stridor
    D. Gurgling
    A. Wheezing
  4. You are treating a 82 year old male in hypovolemic shock. In order to increase their blood pressure you should administer:
    A. 0.45% NaCl via IV
    B. isotonic fluids orally
    C. dextrose 5% in water via IO
    D. isotonic crystalloid via IV
    D. isotonic crystalloid via IV
  5. A 35 year old man is pulseless and apneic. He was found in a lake after falling through the ice. His wife says he left the house 3 hours ago. His is cold and rigid. What should you do?
    A. Perform chest compressions
    B. Begin active rewarming techniques
    C. Obtain IO access and administer a warm fluid bolus
    D. Contact the medical examiner
    A. Perform chest compressions
  6. What does a pulsating mass in the abdomen suggest?
    A. Abdominal aortic aneurysm
    B. Perforated ulcer
    C. Hernia
    D. Ruptured ectopic pregnancy
    A. Abdominal aortic aneurysm
  7. What is the safest and most effective landing pattern for a helicopter?
    A. At a slight angle like that of a jet
    B. Nose of the helicopter pointing up with a tail wind
    C. Straight up and down in a vertical motion
    D. Nose of the helicopter pointing down with a slow descent
    A. At a slight angle like that of a jet
  8. During inspiration, what happens to blood flow in the thoracic cavity?
    A. afterload increases
    B. blood flow increases to the right atrium
    C. blood flow through the vena cava is reduced
    D. pressure in the aorta increases
    B. blood flow increases to the right atrium
  9. What is the period of the cardiac cycle when stimulation will not produce any depolarization?
    A. Impulse refractory period
    B. Absolute refractory period
    C. Relative refractory period
    D. Transient refractory period
    B. Absolute refractory period
  10. While treating a 56 year old male for chest pain, you note a few PVCs on the monitor. You print a strip and measure the distance between the normal beats on each side of the PVC. The normal sinus rhythm continues on both sides of the PVC as if it were the normal beat, even though the PVC happens sooner than the sinus beat would have. The pause after the PVC would be:
    A. A sign of continued ischemia
    B. A compensatory pause
    C. A non-compensatory pause
    D. Reynaud’s Pause
    B. A compensatory pause
  11. A 5 month old male is in cardiac arrest. Resuscitation efforts have been ongoing for 30 minutes. His heart rhythm is and has been asystolic throughout treatment. What should you do?
    A. Continue CPR and ACLS drugs for another thirty minutes
    B. Begin termination of resuscitative efforts
    C. Report suspected child abuse to the police
    D. Wait for the coroner before stopping CPR
    B. Begin termination of resuscitative efforts
  12. Which of the following requires ATP?
    A. Active transport
    B. Osmosis
    C. Diffusion
    D. Facilitated diffusion
    A. Active transport
  13. A 52 year old male was the unrestrained driver in a frontal impact car crash. Which of the following injuries would lead you to believe the patient traveled “up and over” the dashboard instead of “down and under”?
    A. Patellar dislocation
    B. Cervical spine fracture
    C. Ruptured diaphragm
    D. Flail chest
    C. Ruptured diaphragm
  14. How does the blood flow through the heart?
    A. Right ventricle to right atrium to lungs to left ventricle to left atrium
    B. Right atrium to right ventricle to lungs to left atrium to left ventricle
    C. Left ventricle to left atrium to lungs to right ventricle to right atrium
    D. Left atrium to left ventricle to lungs to right atrium to right ventricle
    B. Right atrium to right ventricle to lungs to left atrium to left ventricle
  15. As soon as an infant’s head is delivered, what should you do?
    A. Determine the location of the umbilical cord.
    B. Rotate the shoulders to the anterolateral position.
    C. Examine the vulva and perineum for lacerations.
    D. Exert strong steady pressure on the fundus.
    A. Determine the location of the umbilical cord.
  16. A patient has aphasia following a stroke, which lobe of the brain was affected?
    A. Temporal
    B. Occipital
    C. Parietal
    D. Frontal
    A. Temporal
  17. What does the P wave represent with an ECG?
    A. Ventricular depolarization
    B. Atrial depolarization
    C. Ventricular repolarization
    D. Atrial repolarization
    B. Atrial depolarization
  18. A lethargic 22 month old male presents with hoarseness and coughing. He has sternal retractions and his belly and chest move in opposite directions with each breath. He is breathing at a rate of 64. What should you suspect?
    A. Cardiopulmonary failure.
    B. Respiratory failure.
    C. Cardiopulmonary arrest.
    D. Respiratory distress.
    B. Respiratory failure.
  19. A patient was ejected from an automobile and is apneic and pulseless. ECG reveals a regular rhythm with an R-R interval of 1.24 seconds and a QRS width of 110 milliseconds. You should consider
    A. transcutaneous pacing.
    B. atropine 0.5 mg IV.
    C. epinephrine 1 mg IV.
    D. adenosine 6 mg rapid IV.
    C. epinephrine 1 mg IV.
  20. What organ is located in the upper right quadrant?
    A. Kidney
    B. Liver
    C. Pancreas
    D. Spleen
    B. Liver
  21. A 24 year old male was stabbed in the thigh. Bright red blood spurts from the wound. What should you suspect that the patient severed?
    A. Vein
    B. Artery
    C. Ligament
    D. Tendon
    B. Artery
  22. A 24 year old male was working underneath his car when it fell off of the jack. He was under the car approximately 20 minutes before being extricated. He has bluish discoloration of his skin above the nipple line. What should you suspect?
    A. Traumatic asphyxiation
    B. Pericardial tamponade
    C. Cushing’s triad
    D. Pulmonary contusion
    A. Traumatic asphyxiation
  23. A unrestrained 24 year old female driver hit an electrical pole and struck the steering wheel. She appears pale and complains of nausea and abdominal pain. What organ is LEAST likely to suffer an injury as a result of this impact?
    A. Spleen
    B. Diagphram
    C. Liver
    D. Stomach
    D. Stomach
  24. What causes Down’s syndrome?
    A. Brain trauma
    B. Extra chromosome
    C. Bacterial infection
    D. Psychological trauma
    B. Extra chromosome
  25. Where would you insert the needle to decompress the chest?
    A. Below the 3rd rib midclavicular line
    B. Below the 4th rib midclavicular line
    C. Above the 4th rib midclavicular line
    D. Above the 3rd rib midclavicular line
    D. Above the 3rd rib midclavicular line
  26. A 67 year old male complains of bloody stool with accompanying nausea and emesis. He states symptoms began 3 days ago. Vital signs BP 100/86, P 88, R 18. What should you do?
    A. IV fluid bolus
    B. Administer a diuretic
    C. Administer an antiemetic
    D. Apply oxygen
    A. IV fluid bolus
  27. How much time do you have to successfully intubate a patient?
    A. 30 seconds.
    B. 15 seconds.
    C. 60 seconds.
    D. 45 seconds.
    A. 30 seconds.
  28. What condition results when the alveoli collapse?
    A. Pneumothorax
    B. Pneumonia
    C. Effusion
    D. Atelectasis
    D. Atelectasis
  29. Why should you administer IV fluid to a patient in hypovolemic shock?
    A. Increase the blood pressure
    B. Maintain adequate tissue perfusion
    C. Increase the body’s oxygen carrying capacity
    D. Maintain the body’s homeostatic state
    B. Maintain adequate tissue perfusion
  30. What best defines shock?
    A. Deficiency of vascular tone due to hemorrhage.
    B. Inadequate cerebral blood flow.
    C. Hemorrhage exceeding 50% of total blood volume.
    D. Lack of adequate perfusion to the tissues.
    D. Lack of adequate perfusion to the tissues.
  31. What causes priapism in a trauma patient?
    A. Rupture of the urinary bladder
    B. Total autonomic nervous system dysfunction
    C. Parasympathetic nervous system dominance
    D. Sympathetic nervous system dominance
    C. Parasympathetic nervous system dominance
  32. What is the primary function of insulin?
    A. Increase glucose transport out of the cells and increase pancreas glycogen levels
    B. Increase glucose transport out of the cells and increase liver glycogen levels
    C. Increase glucose transport into the cells and increase liver glycogen levels
    D. Decrease glucose transport into the cells and decrease pancreas glycogen levels
    C. Increase glucose transport into the cells and increase liver glycogen levels
  33. A 60 year old male fell onto a coffee table yesterday and he woke up this morning complaining of diffuse, dull abdominal pain. You note periumbilical bruising. What is this known as?
    A. Crohn’s sign
    B. Cullen’s sign
    C. Kehr’s sign
    D. Grey-Turner’s sign
    B. Cullen’s sign
  34. After placement of an ET tube, you find breath sounds are diminished on the left side but present on the right side. What should you do?
    A. Contact Medical Direction and ask for further instruction.
    B. Deflate and withdraw the tube while listening over the left side.
    C. Deflate the cuff, remove the tube and ventilate with a BVM.
    D. Continue ventilating the patient and secure the tube.
    B. Deflate and withdraw the tube while listening over the left side.
  35. A 25 year old male has a closed head injury and a knife impaled in his back. He is combative and screams incomprehensible words while thrashing. What should you do?
    A. Titrate diazepam administration to gain control of the patient.
    B. Use straps and spine board to gain control of the patient.
    C. Use rapid sequence intubation to protect the patient.
    D. Request the police to restrain the patient for transport.
    C. Use rapid sequence intubation to protect the patient.
  36. What does the body use for fuel in diabetic ketoacidosis?
    A. Acids
    B. Fats
    C. Glucose
    D. Ketones
    B. Fats
  37. What neurotransmitter found in the brain that regulates mood, anxiety, and sleep?
    A. Acetylcholine
    B. Gamma-aminobutyric acid
    C. Norepinephrine
    D. Serotonin
    D. Serotonin
  38. A 38 year old female complains of sudden onset of shortness of breath. She has no previous history and no allergies. Her only medications are birth control pills. Her last meal was 8 hours earlier at an airport prior to her flight home. What should you suspect?
    A. Allergic reaction
    B. Pulmonary embolus
    C. Influenza
    D. Undiagnosed asthma
    B. Pulmonary embolus
  39. A 35 year old epileptic is actively seizing (full clonic movement). What medication should you consider?
    A. Verapamil
    B. Procainamide
    C. Lorazepam
    D. Calcium
    C. Lorazepam
  40. What causes Tuberculosis?
    A. Bacteria
    B. Fungus
    C. Antibody
    D. Virus
    A. Bacteria
  41. A 68 year old female with CHF took 400 mg of furosemide, and is now unresponsive. Vitals signs are BP 110/70, P 100, R 6. What should you do?
    A. Perform an IV fluid bolus
    B. Begin manual ventilations
    C. Administer sodium bicarbonate
    D. Administer nitroglycerin
    B. Begin manual ventilations
  42. Which stage of delivery begins with complete cervical dilation and ends with delivery of the baby?
    A. Second
    B. Third
    C. Fourth
    D. First
    A. Second
  43. A 7 year old boy is found supine at the base of 15 foot high jungle gym. His teacher is unsure if he fell from the equipment. Vital signs are BP 100/90, R 0, P 42, SaO2 93%. What should you do?
    A. Insert an oropharyngeal airway.
    B. Open his airway with a head tilt-chin lift maneuver.
    C. Open his airway with a jaw thrust maneuver.
    D. Start CPR.
    C. Open his airway with a jaw thrust maneuver.
  44. What process involves a gas that travels from an area of higher concentration to lower concentration?
    A. Diffusion
    B. Osmosis
    C. Activated Transport
    D. Filtration
    A. Diffusion
  45. A 12 year old male has suffered a blast injury. What phase would projectiles have caused injury to the patient?
    A. Primary
    B. Quaternary
    C. Tertiary
    D. Secondary
    D. Secondary
  46. A 25 year old female began to experience shortness of breath, chest tightness, and coughing. She has difficulty speaking in full sentences. Vital signs are BP 116/64, P 96 and regular, RR 24 and labored, SpO2 93% on room air. You auscultate her lung sounds and hear high-pitched lung sounds on exhalation. What should you do?
    A. Assist with her prescribed nitroglycerin
    B. Administer albuterol via a nebulizer
    C. Administer abdominal thrusts
    D. Assist ventilations with a bag-valve mask
    B. Administer albuterol via a nebulizer
  47. You have been assigned to triage at an MCI. Your patient is bleeding from a cut on his head, R 22, P 100 radially, and he keeps asking “What happened?”. What priority should be assigned to this patient?
    A. Yellow
    B. Red
    C. Black
    D. Green
    A. Yellow
  48. A 32 year old female seated at a restaurant has severe wheezing. She is flushed, appears anxious, and has a runny nose. She states that she is allergic to shellfish and peanuts. What should you suspect?
    A. Anaphylaxis
    B. Epiglottitis
    C. Asthma
    D. Aspiration
    A. Anaphylaxis
  49. You arrive on scene to find a 14 year old male leaning forward in a chair. He appears anxious, is drooling and you hear high-pitched sounds when he inhales. His mother states he has a fever and has been complaining of a sore throat. What should you suspect?
    A. Anaphylaxis
    B. Asthma
    C. Croup
    D. Epiglottitis
    D. Epiglottitis
  50. How should you ensure that your glucometer is functioning properly?
    A. Insert a test strip and ensure that the glucometer turns on.
    B. Measure your partner’s blood glucose level and compare to normal.
    C. Measure your own blood glucose level and compare to your normal.
    D. Insert a test strip and acquire a drop of the glucometer control solution
    D. Insert a test strip and acquire a drop of the glucometer control solution
  51. What is the biggest difference between cardiogenic shock and hemorrhagic shock?
    A. Pulmonary edema
    B. Hypotension
    C. Cardiac Arrest
    D. Cyanosis
    A. Pulmonary edema
  52. When you apply an end-tidal CO2 monitor, what are the 3 main items you are measuring?
    A. Metabolism, perfusion, and ventilation
    B. Metabolism, perfusion, and respiration
    C. Metabolism, cardiac output, and ventilation
    D. Perfusion, ventilation, and oxygenation
    A. Metabolism, perfusion, and ventilation
  53. What do you call progressively deeper, faster breathing alternating gradually with shallow, slower breathing?
    A. Kussmaul’s respirations
    B. Cheyne-Stokes respirations
    C. Biot’s respirations
    D. Agonal respirations
    B. Cheyne-Stokes respirations
  54. What happens during phase 0 of the cardiac action potential?
    A. The sodium-potassium pump is initiated.
    B. Potassium moves rapidly into the cell.
    C. The slow calcium channels close.
    D. The fast sodium channels open.
    D. The fast sodium channels open.
  55. A 26 year old female jumped from the second floor of her house. She states she was getting away from her abusive husband. What should you do?
    A. Obtain a set of vital signs.
    B. Check for bilateral ankle fractures.
    C. Determine if the scene is safe.
    D. Asses for compression injury to the spine.
    C. Determine if the scene is safe.
  56. What structure has baroreceptors?
    A. Vena cava
    B. Ventricular wall
    C. Atrioventricular node
    D. Aortic arch
    D. Aortic arch
  57. An alert, 78 year old female complains of dyspnea. She speaks in full sentences and answers all your questions. Vital signs are BP 150/70, P 110 and irregular, R 26 and labored. You note peripheral cyanosis. What should you do?
    A. Begin positive-pressure ventilation with a BVM.
    B. Sedate and intubate orotracheally.
    C. Apply a nasal cannula with high flow oxygen at 2 lpm.
    D. Apply high flow oxygen at 12-15 lpm.
    D. Apply high flow oxygen at 12-15 lpm.
  58. When you are caring for more than one trauma patient at a time, when should you change your gloves?
    A. For each new patient
    B. For each new procedure
    C. Whenever time permits
    D. Whenever they become soiled
    A. For each new patient
  59. A 2 year old male has ingested an unknown amount of gasoline. What is your primary concern in your initial assessment?
    A. Establishing IV access TKO.
    B. Determining if the ingestion was intentional.
    C. Evaluating his cardiac rhythm.
    D. Evaluating his airway.
    D. Evaluating his airway.
  60. You arrive on scene to find a 4 year old male with a pulse but inadequate respiratory effort. At what rate do you administer assisted ventilation?
    A. 1 breath every 5 to 6 seconds
    B. 1 breath every 2 to 3 seconds
    C. 1 breath every 3 to 5 seconds
    D. 1 breath every 1 to 2 seconds
    B. 1 breath every 2 to 3 seconds
  61. A 55 year old male remains in ventricular fibrillation after three shocks and an IV epinephrine. What should you do?
    A. Epinephrine (1:1000) in the sequence:1mg, 3mg, 5mg.
    B. Epinephrine (1:10,00) by endotracheal tube.
    C. Epinephrine (1:10,000) every three to five minutes.
    D. Epinephrine (1:1,000) every three to five minutes.
    C. Epinephrine (1:10,000) every three to five minutes.
  62. A 45 year old male complains of difficulty breathing after being removed from a home contaminated with carbon monoxide. His SpO2 is paradoxically 100%, what does this mean?
    A. CO affects the nail beds, making SaO2 unreliable.
    B. CO is displacing O2 on erythrocyte hemoglobin.
    C. CO is binding to the alveoli preventing O2 from being exchanged.
    D. Carbonic anhydrase is inhibiting gas exchange.
    B. CO is displacing O2 on erythrocyte hemoglobin.
  63. A 22 year old female complains of joint pain and flu-like symptoms over the past two days. Her abdomen is soft and non-tender, but she appears to have a circular rash on her lower left quadrant. What should you suspect?
    A. Cholecystis
    B. Gastroenteritis
    C. Lyme’s disease
    D. Influenza
    C. Lyme’s disease
  64. What the mechanism of action for glucagon?

A. Convert glucose to glycogen
B. Reduce blood glucose levels
C. Move glucose into the cell
D. Stimulate gluconeogenesis
D. Stimulate gluconeogenesis

  1. When delivering a radio report, it is important to
    A. speak continuously in order to expedite the report.
    B. provide a field assessment of the patient’s condition.
    C. clearly state and spell the patient’s name.
    D. provide a comprehensive health history.
    B. provide a field assessment of the patient’s condition.
  2. What is the primary action of insulin?
    A. Add a functional group during gluconeogenesis.
    B. Reduce the glucose consumption of the cells.
    C. Facilitate glucose transport into cells.
    D. Increase blood glucose levels.
    C. Facilitate glucose transport into cells.
  3. What cells form the initial clot following vascular injury?
    A. Erythrocytes
    B. Lymphocytes
    C. Thrombocytes
    D. Monocytes
    C. Thrombocytes
  4. What is the first confirmation of tube placement after orotracheal intubation?
    A. Condensation in the ET tube.
    B. Auscultation of the epigastrium
    C. Visualizing the tube passing the vocal cords.
    D. End tidal CO2 confirmation.
    B. Auscultation of the epigastrium ?
  5. A 88 year old male is combative and angry. His wife states that he is normally pleasant and calm. What intervention would narrow your differential diagnosis?
    A. A response to oxygen therapy
    B. A blood glucose reading
    C. If he responds to forceful control
    D. Cincinnati stroke scale
    B. A blood glucose reading
  6. What does preload refer to?
    A. The pressure in the filled ventricle at the end of systole.
    B. The pressure in the filled ventricle at the end of diastole.
    C. The pressure in the filled atria at the end of the systole.
    D. The pressure in the filled atria at the end of the diastole.
    B. The pressure in the filled ventricle at the end of diastole.
  7. Which immunoglobulin is mainly responsible during inflammation?
    A. IgM
    B. IgE
    C. IgG
    D. IgA
    B. IgE
  8. What percent of oxygen is delivered with a pocket mask on room air?
    A. 23
    B. 92
    C. 17
    D. 10
    C. 17 ?
  9. A 57 year old male complains of chest pain, shortness of breath and nausea. A 12 lead ECG shows ST elevation in Ieads V2, V3 and V4. What should you administer?
    A. Dopamine and oxygen.
    B. Aspirin and oxygen.
    C. Epinephrine and nitroglycerin.
    D. Nitroglycerine and oxygen.
    B. Aspirin and oxygen.
  10. Which of the following is an autosomal disorder of the exocrine glands which results in physiological changes in the respiratory, GI, integumentary, and muscoskeletal systems?
    A. Tuberculosis
    B. Sickle cell anemia
    C. Pneumonia
    D. Cystic fibrosis
    D. Cystic fibrosis
  11. What structure prevents food or liquid from entering the trachea?
    A. Thyroid cartilage
    B. Uvula
    C. Larynx
    D. Epiglottis
    D. Epiglottis
  12. What is diphenhydramine HCL?
    A. Beta agonist
    B. Antihistamine
    C. Sympathomimetic
    D. Diuretic
    B. Antihistamine
  13. An 18 year old male was struck in the head by a large rock. When caring for this patient, what should you continually reassess?
    A. Level of consciousness.
    B. Locomotor function.
    C. Pulse motor sensory.
    D. Quality and degree of pain.
    A. Level of consciousness.
  14. What is the onset of action for succinylcholine?
    A. 15-30 seconds
    B. 60-90 seconds
    C. 3-4 minutes
    D. 2-3 minutes
    B. 60-90 seconds
  15. You arrive at a house and find the occupants unconscious with the stove on. You should suspect
    A. carbon monoxide poisoning.
    B. carbon dioxide poisoning.
    C. medication overdose.
    D. cyanide poisoning.
    A. carbon monoxide poisoning.
  16. A 76 year old female is pulseless and apneic. The ECG monitor shows asystole. What should you do?
    A. Administer epinephrine
    B. Start chest compressions
    C. Attempt a precordial thump
    D. Perform immediate defibrillation
    B. Start chest compressions
  17. Which of the following structures serves as a passageway for both the respiratory and digestive system?
    A. Trachea
    B. Pharynx
    C. Esophagus
    D. Cricoid cartilage
    B. Pharynx
  18. What is a highly infectious disease that causes bronchiolitis?
    A. RSV
    B. Pneumonia
    C. Common cold
    D. Asthma
    A. RSV
  19. A 39 year old male has an angulated deformity to the humerus. You cannot feel a distal pulse and the limb is blue. What should you do?
    A. Contact medical control for pain medication orders.
    B. Attempt to pull gentle traction and realign the bones.
    C. Splint with a sling and swathe, apply ice, and transport.
    D. Transport emergently to the closest hospital
    B. Attempt to pull gentle traction and realign the bones.
  20. A 23 year old male complains of diffuse lower leg pain and tension after a car accident. Pedal pulses are absent bilaterally and you note slight cyanosis in his feet. Vital signs are BP 130/90, P 88, R 16. What should you suspect?
    A. Iliac arterial rupture
    B. Bilateral tibial fractures
    C. Compartment syndrome
    D. Lower leg embolism
    C. Compartment syndrome
  21. What happens to hemoglobin as the blood pH decreases?
    A. Releases oxygen.
    B. Releases carbon dioxide.
    C. Binds to carbon dioxide.
    D. Binds to oxygen.
    A. Releases oxygen.
  22. A 24 year old male screams at you to leave after your arrive on scene. He states that there are people after him and that they can hear his thoughts. He is agitated nervous, and constantly peering out the windows. His mother states that this is abnormal for him. What is he suffering from?
    A. Organic psychosis
    B. Dementia
    C. Autism
    D. A neurotic disorder
    A. Organic psychosis
  23. A 62 year old female complains of chest pain and shortness of breath. She is extremely anxious. Vital signs are BP 80/56, P 118, R 24 with crackles, SpO2 87%. What is the most likely cause?
    A. Pulmonary embolus
    B. Hypovolemic shock
    C. Right-sided heart failure
    D. Cardiogenic shock
    D. Cardiogenic shock
  24. A 62 year old female complains of chest pain and shortness of breath. She is extremely anxious. Lung sounds are present with crackles in the bases. Vital signs are BP 80/56, P 128, R 24, SpO2 87%. What should you do?
    A. Administer nitroglycerin
    B. Start a dopamine drip
    C. Give a fluid bolus
    D. Administer Aspirin
    D. Administer Aspirin
  25. You suspect your partner has intubated the esophagus. What should you do?
    A. Check the end tidal CO2 monitor.
    B. Leave the tube in place and ventilate by BVM.
    C. Extubate and ventilate by BVM.
    D. Extubate and reattempt to intubate the patient.
    C. Extubate and ventilate by BVM.
  26. A 79 year old male complains of shortness of breath. He smokes cigarettes for 60 years. On physical exam you notice that he has a barrel shaped chest. What pathophysiology is most likely causing his symptoms?
    A. Collapsed bronchial tubes
    B. Spasm of bronchial muscle
    C. Inflammation of the trachea
    D. Excessive mucus production
    A. Collapsed bronchial tubes
  27. Which of the following is NOT part of the Cincinnati Stroke Test?
    A. Arm drift
    B. Slurred speech
    C. Ability to walk
    D. Facial droop
    C. Ability to walk
  28. During intubation, what is the maximum amount of time to allow without ventilating your patient?
    A. 10 seconds
    B. 15 seconds
    C. 60 seconds
    D. 30 seconds
    D. 30 seconds
  29. A 36 year old male with a history of asthma is in respiratory arrest. His carotid pulse corresponds to the ECG below. After establishing an airway and beginning ventilations, what should you do?
    A. Administer adenosine.
    B. Administer an in-line albuterol nebulizer.
    C. Perform synchronized cardioversion.
    D. Administer epinephrine.
    B. Administer an in-line albuterol nebulizer.
  30. What is the area where the base of the tongue and the epiglottis meet called?
    A. Alveoli
    B. Vallecula
    C. Carina
    D. Bronchi
    B. Vallecula
  31. A 54 year old female is trapped in a car and complains of chest pain. You are unable to open the door, and the fire department is unable to get their extrication tools to work. What should you do?
    A. Break the window farthest from her to gain access.
    B. Use the extrication tools without power.
    C. Use a hand saw to cut the narrowest part of the frame to gain access.
    D. Wait 30 minutes for the replacement tools to arrive.
    A. Break the window farthest from her to gain access.
  32. What occurs during compression of the inferior vena cava?
    A. Supine hypotensive syndrome
    B. Eclampsia
    C. Preeclampsia
    D. Pregnancy induced hypertension
    A. Supine hypotensive syndrome
  33. What will stimulation of a B1 receptor in the heart do?
    A. Slow the heart with an increase contraction strength
    B. Speed up the heart rate and increase contraction strength
    C. Speed up the heart rate and decrease contraction strength
    D. Slow the heart with an decrease contraction strength
    B. Speed up the heart rate and increase contraction strength
  34. What is abnormality of breathing rate, pattern, or effort called?
    A. Dysphonia
    B. Hypoxia
    C. Dyspnea
    D. Anoxia
    C. Dyspnea
  35. What are you required to report directly to law enforcement?
    A. Presence of illegal drugs
    B. Trauma caused by alcohol use
    C. Suspected child abuse
    D. Suspected sexual assault
    D. Suspected sexual assault
  36. What is the function of the ductus arteriosus?
    A. Provide a way for fetal wastes to leave material circulation.
    B. Allow blood to bypass the fetus’s lungs before birth.
    C. Allow the neonate to take a breath following delivery.
    D. Decrease ambient pressure in the lungs before birth.
    B. Allow blood to bypass the fetus’s lungs before birth.
  37. You are dispatched to a residence for a 67 year old female complaining of chest pain. The patient is pale, diaphoretic and describes are pain as crushing. Vital signs are BP 120/80, Pulse 100, Respirations are 22 and oxygen saturation is 90 percent. Select the correct medications in order:
    A. Oxygen, Morphine, Digoxin
    B. Metoprolol, Nitroglycerine, Aspirin
    C. Nitroglycerine, Aspirin
    D. Oxygen, Aspirin, Nitroglycerine
    D. Oxygen, Aspirin, Nitroglycerine
  38. What inherited metabolic disease of the lungs and digestive system manifests during childhood?
    A. Multiple sclerosis
    B. Cerebral palsy
    C. Cystic fibrosis
    D. Sickle cell disease
    C. Cystic fibrosis
  39. Which stage of shock occurs when cardiac output and systolic blood pressures are maintained by increasing catecholamine production?
    A. Compensated shock
    B. Uncompensated shock
    C. Irreversible shock
    D. Diffuse shock
    A. Compensated shock
  40. How many people does it take to provide Optimal BVM ventilation?
    A. 1
    B. 4
    C. 3
    D. 2
    C. 3 (Likely wrong, D. 2 is better IMO)
  41. During the resuscitation of a 45 year old female you note a change in rhythm and the ECG now shows sinus bradycardia. What should you do?
    A. Assess for a pulse
    B. Administer atropine
    C. Obtain a blood pressure
    D. Begin transcutaneous pacing
    A. Assess for a pulse
  42. A 23 year old alert and pregnant female complains of severe abdominal pain after an motor vehicle crash. You note her stomach is rigid and distended. There is no vaginal bleeding or discharge. Vital signs are BP 94/58, P 128, R 30 and shallow, SpO2 90%. What should you suspect?
    A. Uterine rupture
    B. Placenta previa
    C. Placenta abruptio
    D. Pre-term labor
    A. Uterine rupture
  43. A motor vehicle collided with an electrical pole resulting in downed power lines. A large crowd has gathered around the accident. Whom should you call for support?
    A. Volunteer Fire Department and Rescue Squad
    B. Local Power Company and State Police
    C. Volunteer Fire Department and State Police
    D. Local Power Company and Volunteer Fire Department
    B. Local Power Company and State Police
  44. What effect does reducing preload have in a patient with pulmonary edema?
    A. Decreased respiratory demand
    B. Decreased cardiac efficiency
    C. Increased afterload
    D. Improved cardiac efficiency
    D. Improved cardiac efficiency
  45. Which of the following would properly represent a female who has been pregnant three times, given birth twice, and had one spontaneous abortion?
    A. P3, G2, AB1.
    B. P5, G2, AB1.
    C. G3, P2, AB1.
    D. G2, P2, AB1.
    C. G3, P2, AB1.
  46. A 28 year old male has severe flank pain radiating to his back. He squirms around on the floor unable to find a comfortable position, rating his pain a 9/10. He has consumed no fluids over the past day. Vital signs are BP 200/80, P 100, R 18. What should you do?
    A. Administer morphine or hydromorphone.
    B. Apply oxygen by nonrebreather mask and transport.
    C. Establish IV access and administer a fluid bolus.
    D. Administer aspirin and nitroglycerin.
    A. Administer morphine or hydromorphone.
  47. An 18 year old female with a suspected cervical spine injury has a significant amount of blood in her mouth. What should you do?
    A. Stabilize her head and suction her airway.
    B. Perform a jaw-thrust maneuver and attempt to ventilate her.
    C. Perform a jaw-thrust and finger sweep in order to remove blood from her mouth.
    D. Assess the her respiratory rate for no longer than 30 seconds.
    A. Stabilize her head and suction her airway.
  48. A a family calls you to take their 20 year old son to the hospital for an involuntary psychiatric evaluation. They state that he is physically aggressive and refuses to take his antipsychotic drugs. He is quiet and refuses to communicate. What should you do?
    A. Ask the parents to help you restrain the patient.
    B. Talk firmly and tell him that you are going to restrain him now, but you will release the restraints enroute if he behaves himself.
    C. Call for police assistance to get a transport hold.
    D. Call another medic or fire unit to assist you.
    C. Call for police assistance to get a transport hold.
  49. You are dispatched for a 78 yo male who has fallen. He currently has no complaints and is alert and oriented. He is unsure if he lost consciousnessness at any point. He takes metoprolol and atorvastatin. His blood pressure when lying down is 140/90 with a HR of 58 which becomes 100/60 with a HR of 59 when standing. You suspect which of the following as a the most likely etiology of his fall:
    A. hemorrhagic stroke
    B. unsteady gait with poor mobility
    C. medication side effect
    D. acute myocardial infarction
    C. medication side effect
  50. What is an injury to ligaments around a joint called?
    A. Dislocation
    B. Sprain
    C. Fracture
    D. Strain
    B. Sprain
  51. Which structure filters blood into a nephron?
    A. loop of Henle
    B. Collecting duct
    C. Bowman’s capsule
    D. Glomerulus
    D. Glomerulus
  52. What drug is a specific antagonist to opiate derived drugs?
    A. Charcoal
    B. Ipecac
    C. Midazolam
    D. Narcan
    D. Narcan
  53. A 16 year old was freed from entanglement in a motor vehicle crash. Their right arm is amputated below the elbow. What should you do with the amputation?
    A. Place it into its anatomical position and wrap it with dry gauze
    B. Place it in a bag packed with ice and transport it with patient
    C. Wrap it in moistened gauze, place it in a bag, and keep it cool
    D. Wrap it in dry gauze and keep it warm during transport
    C. Wrap it in moistened gauze, place it in a bag, and keep it cool
  54. During the action potential of a cardiac cell, what causes the initial conformational change in the sodium-potassium pump?
    A. Slow channel ion gates
    B. Adenosine triphosphate
    C. Calcium
    D. Sodium and potassium
    B. Adenosine triphosphate
  55. A tachypneic 17 year old male has an open gunshot wound to the right anterior chest. What should you do?
    A. Administer high-flow oxygen.
    B. Begin bag valve mask ventilation.
    C. Apply an occlusive dressing.
    D. Decompress the right chest.
    C. Apply an occlusive dressing.
  56. What neurotransmitter is responsible for stimulation of the postganglionic fibers in the parasympathetic nervous system?
    A. Norepinephrine
    B. Epinephrine
    C. Acetylcholine
    D. Cortisone
    C. Acetylcholine
  57. A 6 year old male complains of belly pain. What should you use to determine his pain level?
    A. 1 to 10 pain rating scale
    B. Wong/Baker pain scale
    C. Oucher pain scale
    D. Analog visual scale
    C. Oucher pain scale
  58. Which of the following substances is most likely to cause premature atrial contractions?
    A. Caffeine
    B. Aspirin
    C. Marijuana
    D. Phenobarbital
    A. Caffeine
  59. An alert 17 year old male was struck in the abdomen with a large wrench. Vital signs are BP 90/78, P 120 R 22. He is cool, pale and diaphoretic. Palpation of the abdomen reveal bruising, pain, and rigidity to the right upper quadrant. To which organ should you suspect injury?
    A. Spleen
    B. Liver
    C. Right kidney
    D. Small intestine
    B. Liver
  60. An intoxicated 45 year old homeless man was hit in the chest. He complains of pain with inspiration. Bilateral lung sounds are clear. Vital signs are BP 130/80, P 76, R 12, SpO2 94%. What should you suspect?
    A. Rib fractures
    B. Tension Pneumothorax
    C. Head injury
    D. Pericardial tamponade
    A. Rib fractures
  61. What does oversecretion by the adrenal cortex cause?
    A. Cushing’s disease
    B. Diabetes mellitus
    C. Myxedema
    D. Graves’ disease
    A. Cushing’s disease
  62. What is the main site of drug metabolism?
    A. Liver
    B. Kidney
    C. Spleen
    D. Lungs
    A. Liver
  63. Your 65 year old male patient presents with a fever and sharp, steady pain along the upper neck and shoulder that worsens when he takes a breath. Vital signs are BP 144/92, P 99 and regular, R 18 and regular, SpO2 of 95% on room air. What should you do?
    A. Administer nitroglycerin
    B. Obtain intravenous access
    C. Perform a 12-lead ECG
    D. Apply oxygen
    C. Perform a 12-lead ECG
  64. A 23 year old pregnant female is pushing when you arrive. You find her bag of water broke and the fluid was green. Fetal heart tones are 80. What should you do?
    A. Encourage her to continue pushing.
    B. Start at least one large bore IV.
    C. Place her on her side and apply oxygen.
    D. Place her in knee-chest position and apply oxygen.
    C. Place her on her side and apply oxygen.
  65. A 16 year old boy tells you that he “caught” his 40 year old mother “holding a handful of sleeping pills.” The son explains that she is depressed, and he is convinced that she meant to kill herself. She is extremely agitated and is refusing treatment. What should you do?
    A. Request law enforcement assistance to transport the patient.
    B. Contact social services to recommend they follow up on this family.
    C. Carefully document the woman’s refusal and have her sign the form.
    D. Encourage the patient to see a mental health professional that day.
    A. Request law enforcement assistance to transport the patient.
  66. A 56 year old female has an arm injury with arterial bleeding. The dressing becomes soaked with blood. What should you do?
    A. Wrap an elastic bandage around the blood soaked dressing.
    B. Apply additional dressing material with direct pressure.
    C. Remove and replace the dressing with a thicker dressing.
    D. Cover the existing dressing with an occlusive dressing.
    B. Apply additional dressing material with direct pressure.
  67. A 32 year old female attempted to commit suicide by taking sleeping pills and cutting her wrists. She has dark red blood oozing from the wrist lacerations. Vital signs are BP 90/52, P 130 and weak, R 18. What type of bleeding should you suspect?
    A. Arterial
    B. Superficial
    C. Venous
    D. Capillary
    C. Venous
  68. A 51 year old male with a history of COPD is having difficulty breathing. Vital signs are BP 136/74, R 22, P 100, SpO2 95%. What should you do?
    A. Apply high-flow oxygen by bag-valve mask.
    B. Withhold oxygen to avoid decreasing the hypoxic drive.
    C. Apply low-flow oxygen by nasal cannula.
    D. Administer oxygen by nonrebreather mask.
    D. Administer oxygen by nonrebreather mask.
  69. What is the leading cause of cardiac arrest in pediatric populations?
    A. Infection
    B. Blunt force trauma
    C. Respiratory compromise
    D. Hypovolemic shock
    C. Respiratory compromise
  70. A 55 year old female has difficulty breathing and is unable to lie supine. She has pedal edema and bilateral crackles are heard on auscultation. What should you suspect?
    A. Congestive heart failure
    B. Asthma exacerbation
    C. Chronic bronchitis
    D. Myocardial infarction
    A. Congestive heart failure
  71. An alert, ambulatory, 26 year old female admits to laying her motorcycle down at 25 mph to avoid a crash. She has extreme pain to her left side where you note severe abrasions. She denies any neck or back pain. What should you do?
    A. Sterile dressings and treat her pain with ice.
    B. Oxygen and transport in a semi-sitting position of comfort.
    C. C-collar and immobilize her fully in the standing position.
    D. C-collar and transport her on his left side.
    C. C-collar and immobilize her fully in the standing position.
  72. A 45 year old female has type I diabetes and a non-healing foot ulcer. You find that she lacks sensation in her foot. What should you suspect?
    A. Brown-Sequards
    B. Peripheral neuropathy
    C. Claudication
    D. Ranaud’s syndrome
    B. Peripheral neuropathy
  73. What environmental emergency is life-threatening?
    A. Chilblains
    B. Heat exhaustion
    C. Heat stroke
    D. Frostbite
    C. Heat stroke
  74. Where is the blood found during hemoptysis?
    A. Stool
    B. Esophagus
    C. Sputum
    D. Urine
    C. Sputum
  75. What distinguishes pre-eclampsia from eclampsia?
    A. Presence of albuminuria
    B. Occurrence of convulsion or coma
    C. Weight gain of over 40 pounds
    D. Systolic blood pressure over 200 mg Hg
    B. Occurrence of convulsion or coma
  76. A 19 year old female is semi-conscious with snoring respirations. She has overdosed on benzodiazepines. You should use a/an
    A. nasopharyngeal airway.
    B. oropharyngeal airway.
    C. endotracheal airway.
    D. dual lumen airway.
    A. nasopharyngeal airway.
  77. Where does the electrical impulse in the heart begin?
    A. Bundle of His
    B. AV node
    C. Purkinje fibers
    D. SA node
    D. SA node
  78. In what leads does ST elevation suggest an anterior wall MI?
    A. V3, V4
    B. II, III, and aVF
    C. aVR, aVL, and aVF
    D. V1, V2
    A. V3, V4
  79. What is the term used to describe the obligation you have to render assistance to a patient?
    A. Advanced directive
    B. Standing order
    C. Duty to act
    D. Good Samaritan law
    C. Duty to act
  80. What type of drug is Verapamil?
    A. Beta blocker
    B. Antihypertensive medication
    C. Angiotension receptor blocker
    D. Calcium channel blocker
    D. Calcium channel blocker
  81. What is limiting the number of people or resources you are supervising known as?
    A. Unified command
    B. Command and control
    C. Span of control
    D. Unity of command
    C. Span of control
  82. An alert 45 year old female is pale and cyanotic. Her vitals are BP 180/104, P 102 and irregular, R 28 with bilateral crackles. What is the most likely cause of her shortness of breath?
    A. Anaphylaxis
    B. Asthma
    C. Pulmonary edema
    D. Simple pneumothorax
    C. Pulmonary edema
  83. Which of the following medications is an adrenergic agonist that can stimulate either alpha or beta receptors depending on the dose?
    A. Epinephrine
    B. Dopamine
    C. Dobutamine
    D. Atenolol
    B. Dopamine
  84. A 32 year old, 36 week pregnant, female complains of lower back pain. When she is supine, she complains of feeling short of breath and faint. As you continue to assess her, she becomes pale and anxious. Vital signs are BP is 92/76, P 102, R 22. What should you do?
    A. Prepare for a rapid delivery
    B. Reposition her on her left side
    C. Apply oxygen and start an IV
    D. Obtain an orthostatic set of vital signs
    B. Reposition her on her left side
  85. What are established interventions that can be performed without calling medical direction?
    A. Standing orders
    B. Scope of practice
    C. Protocols
    D. Advanced directives
    A. Standing orders
  86. A 20 year old male struck his head on a diving board and is unconscious and apneic. Vital signs are BP 88/70, P 80. He has a large amount of swelling and deformity in the cervical spine area. What should you suspect?
    A. Increased intracranial pressure
    B. Cervical spine injury
    C. Cerebral contusion
    D. Hypovolemic shock
    B. Cervical spine injury
  87. A 55 year old male was pulled from a burning building. He has difficulty breathing and is coughing. High flow oxygen does not provide relief. You are hearing bilateral wheezing the upper lobes of his lungs. What should you do?
    A. Obtain a set of vital signs.
    B. Insert an oropharyngeal airway.
    C. Administer nebulized albuterol.
    D. Administer 0.3 mg epinephrine 1:1000 SQ.
    C. Administer nebulized albuterol.
  88. A man is threatening to beat his wife to death. The police have checked for weapons and called you to evaluate him. While interviewing the patient, what should you do?
    A. Keep him between yourself and the exit.
    B. Quickly expedite transport.
    C. Enter his personal space.
    D. Keep large furniture between him and you.
    D. Keep large furniture between him and you.
  89. What is a contraindication of a traction splint?
    A. Potential spinal injury
    B. Pelvic fracture
    C. Signs of hypoperfusion
    D. Swelling or deformity to the thigh
    B. Pelvic fracture
  90. A patient with AIDS vomited blood during transport. How should you disinfect your ambulance?
    A. Soak the mattress pad for 20 minutes in disinfectant, then let the mattress stand for 6 hours before use.
    B. Using universal precautions, spray a bleach-based cleaning solution and carefully wipe down the stretcher.
    C. Replace the mattress and throw out the sheets.
    D. Place the ambulance out of service until a full OSHA cleaning can be completed.
    B. Using universal precautions, spray a bleach-based cleaning solution and carefully wipe down the stretcher.
  91. You are dispatched for a 78 year old person fallen. They have no complaints and are alert and oriented. They state they are unsure if they lost consciousness at any point. The patient takes metoprolol and atorvastatin. Vital signs: BP 130/84, P 88, R 16, SpO2 = 95%. The patient wants to refuse care. You should first:
    A. acquire a 12 lead ECG
    B. counsel the patient to accept transport
    C. repeat the vital signs
    D. complete a fingerstick glucose
    A. acquire a 12 lead ECG
  92. What is the most effective way to terminate ventricular fibrillation?
    A. Defibrillation
    B. Cardioversion
    C. Chest compressions
    D. Amiodarone
    A. Defibrillation
  93. A 36 year old male and his 34 year old wife complain of severe headaches, confusion, and dizziness. He also complains of chest pain. Both are short of breath. What type of poisoning should you suspect?
    A. Organophosphate
    B. Carbon dioxide
    C. Carbon monoxide
    D. Ammonia
    C. Carbon monoxide
  94. When the vascular container enlarges without proportional increase in the fluid volume, what type of shock should you suspect?
    A. Hypovolemic
    B. Decompensated
    C. Cardiogenic
    D. Distributive
    D. Distributive
  95. You need to perform a needle cricothyrotomy for transtracheal jet ventilation on an adult patient. What is the smallest catheter that is recommended for use?
    A. 16ga.
    B. 12ga.
    C. 18ga.
    D. 14ga.
    A. 16ga.
  96. A patient has been taking pain medication for several months. To maintain the desired effect, they must periodically increase his dose. What is this known as?
    A. Cumulative effect
    B. Tolerance
    C. Synergism
    D. Additive effect
    B. Tolerance
  97. A 70 year old male with a history of CHF complains of severe dyspnea. During transport his SpO2 drops to 76% and he starts to cough up pink sputum. What should you do?
    A. Assist his ventilations
    B. Increase the flow of oxygen
    C. Administer 40mg of Lasix IVP
    D. Perform a 12 lead EKG
    A. Assist his ventilations
  98. A 47 year old, 98 kg male has burns on 25% of his body. In the first 8 hours post-burn, what fluid treatment should he recieve?
    A. 9800 ml of normal saline (NS).
    B. 4900 ml of normal saline (NS).
    C. 4900 ml of lactated ringers (LR).
    D. 9800 ml of lactated ringers (LR).
    C. 4900 ml of lactated ringers (LR).
  99. What is the correct dosage of ipratropium bromide during an asthma attack?
    A. 0.3-0.5 mg, subcutaneously
    B. 250 mg, by nebulizer
    C. 0.5 mg, by nebulizer
    D. 0.25 mg, subcutaneously
    C. 0.5 mg, by nebulizer
  100. What structure contains the respiratory center of the brain?
    A. Cerebrum
    B. Cerebellum
    C. Pons
    D. Medulla
    D. Medulla
  101. What is the movement of drugs in the body as they are absorbed, distributed, metabolized, and excreted called?
    A. Biotransformation
    B. Pharmacokinetics
    C. The mechanism of action
    D. Pharmacodynamics
    B. Pharmacokinetics
  102. Which cranial nerve is responsible for sensation in the face?
    A. VII
    B. V
    C. VIII
    D. II
    B. V
  103. A known alcoholic is unresponsive and coughing up bright, red blood. Which of the following would be contraindicated?
    A. Dual lumen airway
    B. Bag-valve mask
    C. Nasopharyngeal airway
    D. Endotracheal tube
    A. Dual lumen airway
  104. A 20 year old female was involved in a car crash. She complains of midline neck pain and is unable to move her extremities. What secondary injury should you suspect?
    A. Difficulty breathing
    B. Vertebral swelling
    C. Subluxation
    D. Ruptured vertebrae
    B. Vertebral swelling
  105. What additional resources would you need at a motor vehicle accident with a trapped 22 year old female?
    A. Additional State Police and the Fire Department.
    B. Rescue Squad and the Fire Department.
    C. Second ambulance and additional State Police.
    D. Second ambulance and the Fire Department.
    B. Rescue Squad and the Fire Department.
  106. A crew member has thick facial hair. Which of the following is the most appropriate protection against airborne pathogens?
    A. Surgical particulate mask
    B. An NH 15 escape hood
    C. A slightly larger HEPA (N95) mask
    D. Powered air purifying respirator
    D. Powered air purifying respirator
  107. A 24 year old female responds to a sternal rub by opening her eyes and moaning incomprehensibly. She tries to push your hands away from her sternum. What is her Glasgow Coma Scale score?
    A. 9
    B. 3
    C. 6
    D. 12
    A. 9
  108. What is the most prevalent cation in the cell?
    A. Potassium
    B. Sodium
    C. Calcium
    D. Chloride
    A. Potassium
  109. A 23-year old male motorcyclist was struck by a car that ran a stop sign. He is pale and diaphoretic, and no external bleeding is noted. Vital signs are P 136 and R 28. Which of the following is most likely?
    A. Femur fracture
    B. Neurogenic shock
    C. Pneumothorax
    D. Closed head injury
    A. Femur fracture
  110. What genetic disease is characterized by progressive muscle weakness and skeletal muscle degeneration?
    A. Muscular dystrophy
    B. Multiple sclerosis
    C. Amyotrophic lateral sclerosis
    D. Myasthenia gravis
    A. Muscular dystrophy
  111. A 46 year old male is pale, diaphoretic, and complains of light-headedness after palpitations. An ECG shows paroxysmal supraventricular tachycardia. When assessing his perfusion status, what would you suspect?
    A. Delayed capillary refill
    B. Dry, warm skin
    C. Normal vital signs
    D. Rapid, bounding pulse
    A. Delayed capillary refill
  112. On arrival of a motor vehicle crash, what degree of scene assessment needs to be completed?
    A. 90 degrees from the front of the car
    B. 180 degrees of the patient
    C. 360 degrees of the scene
    D. 220 degrees from side of approach
    C. 360 degrees of the scene
  113. What is the pathophysiological reason behind severe joint and abdominal pain in a patient with Drepanocytosis?
    A. Inflammatory reaction from arthritis
    B. Vascular occlusion from abnormal shaped cells
    C. Severe infection involving his appendix
    D. Microvascular occlusion from nitrogen bubbles
    B. Vascular occlusion from abnormal shaped cells
  114. What is the role of angiotensin II in the body?
    A. Inhibits the creation of aldosterone increasing blood pressure.
    B. Stimulates the creation of aldosterone increasing blood pressure.
    C. Stimulates the creation of aldosterone decreasing blood pressure.
    D. Inhibits the creation of aldosterone decreasing blood pressure.
    B. Stimulates the creation of aldosterone increasing blood pressure.
  115. A 38 year female is in labor in her car. This is her fourth child, contractions last 30 seconds and occur every 1-2 minutes. What should you do?
    A. Tell her to bear down hard and to push.
    B. Call for air medical transport.
    C. Move her to the ambulance and prepare for delivery.
    D. Transport the patient to the hospital for treatment.
    C. Move her to the ambulance and prepare for delivery.
  116. A 62 year old man is unresponsive 5 minutes into his dialysis treatment. He has snoring respirations. Pulses are weak and you are unable to obtain a blood pressure. ECG reveals a wide-complex idioventricular rhythm at a rate of 50. What should you do?
    A. Administer calcium chloride
    B. Check his blood glucose
    C. Start CPR
    D. Cardiovert at 360J
    A. Administer calcium chloride
  117. An alert 18 year old female is trapped in her car after a single car crash. She complains of hip and left leg pain. When she is finally extricated from the car, her vital signs decrease. What stages of shock is she transitioning from?
    A. Decompensated to hypovolemic
    B. Compensated to decompensated
    C. Decompensated to compensated
    D. Neurogenic to hypovolemic
    B. Compensated to decompensated
  118. What is the name of the tracheal ring that sits directly inferior to the larynx?
    A. Cuneiform cartilage
    B. Cricoid cartilage
    C. Thyroid cartilage
    D. Corniculate cartilage
    B. Cricoid cartilage
  119. What condition is caused by twisting of the spermatic cord, resulting in a sudden onset of severe scrotal pain?
    A. Epididymitis
    B. Urinary calculi
    C. Priapism
    D. Testicular torsion
    D. Testicular torsion
  120. At a mass gathering several hundred people suddenly began to experience severe salivation, lacrimation, urination, and vomiting. What should you do?
    A. Set up a command post and ensure scene safety.
    B. Alert the Center for Disease Control.
    C. Contact the Department of Homeland Security.
    D. Evacuate all of the patients from the premises.
    A. Set up a command post and ensure scene safety.
  121. A 88 year old female has acute onset of diplopia, slurred speech, difficulty swallowing, and unilateral facial drooping. What should you suspect?
    A. Bacterial meningitis
    B. An ischemic stroke
    C. A subdural hematoma
    D. A postictal state
    B. An ischemic stroke
  122. A 13 year old has been found unresponsive on her bedroom floor. She has shallow respirations at four-times-a-minute, unobtainable radial pulse, pinpoint pupils, and a slow carotid pulse. Her mother provides you with a prescription bottle labeled morphine. Assisted ventilations are started with a bag-valve mask and an IV has been established. What medication would benefit this patient the most?
    A. Lidocaine
    B. Atropine
    C. Naloxone
    D. Epinephrine
    C. Naloxone
  123. Which of the following is the MOST common cause of airway obstruction?
    A. Trauma
    B. The tongue
    C. Laryngeal spasm
    D. Foreign bodies
    B. The tongue
  124. What contains detailed information about all potentially hazardous substances found in a workplace?
    A. MSDS
    B. Placards
    C. NFPA labels
    D. UN number signs
    A. MSDS
  125. What ion plays a major role in regulating the distribution of water in the body?
    A. Magnesium
    B. Calcium
    C. Sodium
    D. Potassium
    C. Sodium
  126. After initial treatments of an arterial bleed in the left forearm are unsuccessful. What should you do?
    A. Apply pressure to the radial artery.
    B. Apply a tourniquet.
    C. Apply pressure to the subclavian artery.
    D. Lower the extremity.
    B. Apply a tourniquet.
  127. What complication may occur as a result of incomplete delivery of the placenta?
    A. Uterine rupture
    B. Chronic urinary tract infections
    C. Disseminated intravascular coagulation
    D. Postpartum vaginal hemorrhage
    C. Disseminated intravascular coagulation
  128. Which is a sign of respiratory failure for an adult?
    A. Mouth breathing
    B. Breathing 16 times per minute.
    C. Nasal flaring
    D. Skin is pink and dry.
    C. Nasal flaring
  129. What personal protective equipment is worn during intubation?
    A. Gloves, eye protection, mask, and gown
    B. Gloves
    C. Gloves, eye protection, and mask
    D. Gloves and eye protection
    C. Gloves, eye protection, and mask
  130. An alert and oriented 35 year old female is sitting on the ground after a car accident. Vital signs are BP 90/62, P 96, R 24 and shallow. She denies any numbness or tingling in her extremities. What should you do?
    A. Transport her immediately.
    B. Obtain her medical history.
    C. Reassess her vitals.
    D. Secure her to a long backboard.
    A. Transport her immediately.
  131. What complicates coronary perfusion during in CPR?
    A. Hyperinflation of the lungs
    B. Esophageal intubation
    C. Gastric distention
    D. Poor airway management
    C. Gastric distention
  132. When completing the narrative portion of your prehospital care report, the “O” in SOAP stands for:
    A. Objection
    B. Observation
    C. Objective
    D. Operation
    C. Objective
  133. How do you determine that you are doing adequate CPR on a child?
    A. Check for a change in rhythm on the cardiac monitor.
    B. Compress the chest at a rate greater than 200.
    C. Check for a carotid pulse during compressions.
    D. Compress the chest at least three inch minimum depth.
    C. Check for a carotid pulse during compressions.
  134. A 72 year old female has altered mental status, irritability, and delirium. Her husband states she has lost 10 lbs in the last 2 weeks. Vital signs are BP 170/100, P 160, temp 102F, skin warm and dry. What should you suspect?
    A. Hypothyroidism
    B. Thyrotoxicosis
    C. Dementia
    D. Cushing’s Syndrome
    B. Thyrotoxicosis
  135. A 44 year old female complains of diffuse abdominal pain with cramping. She states that she has vomited brownish-greenish yellow fluid several times. Her abdomen is distended and she has been constipated for the last few days. What should you suspect?
    A. Cholecystitis
    B. Diverticular disease
    C. Pancreatitis
    D. Bowel obstruction
    D. Bowel obstruction
  136. What common site of arteriovenous malformation and aneurysm results in hemorrhagic stroke?
    A. Middle meningeal artery
    B. Basilar artery
    C. Posterior cerebral artery
    D. Anterior cerebral artery
    B. Basilar artery
  137. What can rough handling of a patient with severe hypothermia cause?
    A. Ventricular fibrillation
    B. Ventricular tachycardia
    C. Decreased respirations
    D. Increased shivering
    A. Ventricular fibrillation
  138. Prior to a seizure, a patient reported seeing strange flashes of light and hearing high pitched sounds. What is this phase of the seizure known as?
    A. Hypertonic
    B. Clonic
    C. Aura
    D. Tonic
    C. Aura
  139. An 18 year old male presents with symptoms of hypoglycemia. What criteria must be met for him to receive oral glucose?
    A. He must be conscious, with a patent airway.
    B. He must have a history of type 1 diabetes.
    C. He must have a prescription for oral glucose.
    D. His blood glucose analysis must be between 70 and 95 mg/dL.
    A. He must be conscious, with a patent airway.
  140. A 41 year old female is bleeding profusely from an extremity wound. Vital signs are BP 78/40, P 130 and weak, R 22. After stopping the bleeding, what should you do?
    A. Reassess her vital signs.
    B. Begin rapid IV fluid replacement.
    C. Place her in the shock position.
    D. Apply a tourniquet above the wound.
    B. Begin rapid IV fluid replacement.
  141. A 73 year old female complains of shortness of breath. While treating her, she stops breathing. What should you do?
    A. Ventilate with a BVM.
    B. Check her pulse.
    C. Start CPR.
    D. Perform orotracheal intubation.
    B. Check her pulse.
  142. What process is involved in the formation of a blood clot?
    A. Tissue plasminogen activator creates fibrin
    B. Thrombin converts fibrinogen into fibrin
    C. Hemoglobin is converted to platelets
    D. Prothrombin stimulates the release of troponin
    B. Thrombin converts fibrinogen into fibrin
  143. What is the inherent rate for the AV node?
    A. 150-250/min
    B. 20-40/min
    C. 60-100/min
    D. 40-60/min
    D. 40-60/min
  144. Which of the following medications can produce anticholinergic effects.
    A. Epinephrine.
    B. Atropine sulfate.
    C. Etomidate.
    D. Nitroglycerin.
    B. Atropine sulfate.
  145. How quickly should you expect a clot to form from a venous laceration?
    A. 1 minute
    B. 7-10 minutes
    C. 11-14 minutes
    D. 4-6 minutes
    D. 4-6 minutes
  146. While assessing a 32 year old pregnant female, you discover fetal heart tones at 98 bpm. What should you do?
    A. Obtain a secondary set of vital signs
    B. Start a dopamine drip
    C. Turn to her left side and apply oxygen
    D. Administer atropine
    C. Turn to her left side and apply oxygen
  147. A 42 year old female complains of tinnitus, nausea and vomiting. Her husband meets you at the door and states that he returned home this morning finding his wife and two children lying on the floor with nausea and headaches. What type of poisoning should you suspect?
    A. Severe randon gas
    B. Carbon monoxide
    C. Toxic black mold
    D. Hydrogen sulfide
    B. Carbon monoxide
  148. A 30 year old female has recently given birth at home. During transport she has a sudden onset of shortness of breath and chest pain. Vital signs are BP 96/60, P 130, R 30, SpO2 92%. What should you administer?
    A. High-flow oxygen and a fluid bolus.
    B. Low-flow oxygen and tocolytic.
    C. A bronchodilator and perform a detailed exam.
    D. Nitroglycerin and obtain a 12 lead ECG.
    A. High-flow oxygen and a fluid bolus.
  149. A 45 year old female has been sexually assaulted with a blunt object. She complains of diffuse abdominal pain, and her vital signs are BP 100/60, P 120, R 26. What should you suspect?
    A. Internal hemorrhage
    B. Ectopic pregnancy
    C. Prolapsed uterus
    D. Uterine cramping
    A. Internal hemorrhage
  150. How should you manage the airway of an unconscious 55 year old male with snoring respirations?
    A. Head-tilt/chin-lift maneuver
    B. Endotracheal intubation
    C. Modified jaw-thrust maneuver
    D. Sellick’s maneuver
    A. Head-tilt/chin-lift maneuver
  151. Lead aVR is often discounted. What is the significance of ST elevations in this lead?
    A. Old STEMI
    B. Proximal LMCA occlusion
    C. No significance
    D. Significant lateral MI
    B. Proximal LMCA occlusion
  152. In which factor of clotting physiology are the clotting proteins, platelets, and red blood cells “washed out” of the vascular space through ongoing hemorrhage?
    A. Medication
    B. Hemodilution
    C. Hypothermia
    D. Acidosis
    B. Hemodilution
  153. A patient vomits blood onto your uniform at the hospital. Your service has no more ambulances available. What should you do?
    A. Alert dispatch that you are out of service for the next 6 hours.
    B. Rinse your uniform out in the sink and notify dispatch of your location.
    C. Wash your uniform only if your patient had a communicable disease.
    D. Thoroughly clean yourself, the ambulance and don a pair of scrubs.
    D. Thoroughly clean yourself, the ambulance and don a pair of scrubs.
  154. During transport a 32 year old pregnant female tells you the baby is coming. You look and see two feet at the vaginal canal. What should you do?
    A. Apply fundal pressure and prepare for a breech delivery.
    B. Direct your partner to pull over and come back to assist you.
    C. Tell her to pant and have your partner begin rapid transport.
    D. Apply oxygen and place her in a knee-chest position.
    D. Apply oxygen and place her in a knee-chest position.
  155. When measuring the R-R interval, you note three large boxes in between. What would the heart rate be?

A. 75 beats/min

B. 100 beats/min

C. 150 beats/min

D. 60 beats/min
B. 100 beats/min

  1. A 22 year old female complains of body aches, severe headache and blurred vision. She admits to having large amounts of alcohol the night before. What diagnosis you should consider LAST?
    A. Cerebral vascular accident
    B. Hangover
    C. Migraine headache
    D. Meningitis
    B. Hangover
  2. A febrile 78 year old male in respiratory distress has a productive cough with yellow sputum. Lung sounds are absent in the lower third of his lung fields. What is a DEFINITIVE treatment for this patient?
    A. Albuterol
    B. Furosemide
    C. Ipratropium bromide
    D. Antibiotics
    D. Antibiotics
  3. You arrive on the scene of a motor vehicle crash involving a car and a bus. You should
    A. Determine number of patients, determine if any additional resources are needed and proceed into the scene
    B. Assess the scene for any hazards, determine the number of patients while determine if any additional resources are needed
    C. Assess the scene for any hazards, determine the number of patients than start to hand out triage tags
    D. Assess the scene for hazards, call for a mass casualty unit and wait for a second ambulance prior to entering the scene
    B. Assess the scene for any hazards, determine the number of patients while determine if any additional resources are needed
  4. What chemical mediator released from mast cells is responsible for producing an acute allergic response?
    A. Acetylcholine
    B. Histamine
    C. Dopamine
    D. Epinephrine
    B. Histamine
  5. Which glands release their chemical products through ducts for a local effect?
    A. Endocrine
    B. Adrenal
    C. Exocrine
    D. Hormone
    C. Exocrine
  6. What is the dose for lorazepam for seizures?
    A. 2 g IV
    B. 200 mcg IV
    C. 2 gr IV
    D. 2 – 4 mg IV
    D. 2 – 4 mg IV
  7. A 4 year old female is warm and withdrawn when you arrive. The mother states that she had a 5 minute seizure prior to your arrival. She has had previous seizures, but that this one was longer and more violent. What should you do?
    A. Administer 4.5 mg diazepam.
    B. Suggest that she see her primary care physician.
    C. Apply oxygen and obtain a 12 lead ECG.
    D. Apply oxygen and give a fluid bolus.
    D. Apply oxygen and give a fluid bolus.
  8. A 62 year old female has difficulty breathing. She is pale, diaphoretic, and has cyanosis of the lips. Lung sounds reveal coarse crackles. Vital signs are, BP 200/104, P 142, R 40. What is your goal in providing treatment?
    A. Decrease preload to reduce oxygen demand.
    B. Decrease afterload to increase cardiac output.
    C. Increase afterload to increase end systolic volume.
    D. Increase preload to increase end diastolic volume.
    A. Decrease preload to reduce oxygen demand.
  9. What condition can present with paranoia, delusions, and auditory hallucinations?
    A. Obsessive-compulsive disorder
    B. Organic brain syndrome
    C. Schizophrenia
    D. Bi-polar disorder
    C. Schizophrenia
  10. Which structure separates the upper airway from the lower airway?
    A. Hyoid bone
    B. Trachea
    C. Carina
    D. Vocal cords
    D. Vocal cords
  11. What is the antidote for Verapamil overdose?
    A. Benadryl
    B. Calcium
    C. Glucagon
    D. Narcan
    B. Calcium
  12. Which of the following is NOT a communicable disease?
    A. Cancer
    B. Hepatitis B
    C. Tuberculosis
    D. HIV/AIDS
    A. Cancer
  13. A 20 year old female has burns on her chest, abdomen, and on the anterior side of her right arm. What percentage of body is burned?
    A. 18%
    B. 22.5%
    C. 40.5%
    D. 27%
    B. 22.5%
  14. You find a patient in the tripod position. What should you suspect?
    A. A cardiac emergency
    B. Gastrointestinal distress
    C. Respiratory distress
    D. A transischemic attack
    C. Respiratory distress
  15. After the birth of male child, you note he is blue all over, is not breathing, and has no pulse. What is his APGAR?
    A. 5
    B. 8
    C. 0
    D. 2
    C. 0
  16. A 20 year old male struck his head while jumping into a swimming pool. He is unconscious and apneic. Vital signs are BP 88/70, P 80. There is a large amount of swelling and deformity in the cervical spine area. What should you do?
    A. Use the head tilt/chin lift to open his airway
    B. Administer oxygen via non-rebreather mask
    C. Secure him to a long spineboard
    D. Apply and inflate MAST
    C. Secure him to a long spineboard
  17. How should you dispose of dressings and bandages that are saturated with blood?
    A. A cardboard box and sealed with tape
    B. In any normal garbage container
    C. In a red bag with a bio hazard seal
    D. At a hospital ED trash container
    C. In a red bag with a bio hazard seal
  18. A 43 year old male complains of a stiff neck, persistent cough, and fever for the past week. He states he has been in and out of homeless shelters for the past 5 months. What should you do?
    A. Apply oxygen by non-rebreather mask.
    B. Put on eye protection and a surgical mask.
    C. Have the patient wear a HEPA mask.
    D. Use gloves, gown, and HEPA mask.
    D. Use gloves, gown, and HEPA mask.
  19. What medical term describes wheals on the skin during an allergic reaction?
    A. Urticaria
    B. Angioedema
    C. Pruritus
    D. Myxedema
    A. Urticaria
  20. What are the 5 variables needed to interpret cardiac rhythms?
    A. Rate, regularity, QRS width, P waves, and P-R interval
    B. Rate, irregularity, U wave, P waves, and P-R interval
    C. Rate, regularity, ST segment, QRS width, and T waves
    D. Rate, regularity, QRS width, ST segment, and P-R interval
    A. Rate, regularity, QRS width, P waves, and P-R interval
  21. A 42 year old male complain of hives and itching on his lower right leg immediately after mowing the lawn. Vital signs are BP 146/84, P 92, R 16. What should you suspect?
    A. Anaphylaxis
    B. Allergic reaction
    C. Eczema
    D. Insecticide poisoning
    B. Allergic reaction
  22. A 25 year old female is in labor and reports an urge to push. Suddenly her contractions weaken. You should suspect:
    A. uterine rupture
    B. false labor
    C. uterine inversion
    D. placenta previa
    A. uterine rupture
  23. An 27 year old male suffered a partial amputation of his foot in a farming accident. Bleeding is controlled. He is alert and oriented but refuses treatment. He speaks directly with online medical control but still refuses treatment and transportation. What should you do?
    A. Call law enforcement for an assisted transport.
    B. Carefully obtain and document the patient’s refusal.
    C. Assume implied consent and transport the patient.
    D. Call social services and report suspicions of neglect.
    B. Carefully obtain and document the patient’s refusal.
  24. An unresponsive 35 year old female has been stabbed in the abdomen. She flexes her arms when you establish an IV and mumbles incomprehensible words, but her eyes remain closed. Respirations are 8-10 times per minute. How should you protect her airway?
    A. Nasal airway
    B. Endotracheal tube
    C. Head tilt chin lift
    D. Oropharyngeal airway
    B. Endotracheal tube
  25. What does S-T segment elevation greater than 0.1 mV in at least two contiguous leads indicate?
    A. Active ischemia
    B. Unstable angina
    C. A Mobitz II heart block
    D. A myocardial infarction
    D. A myocardial infarction
  26. A 25 year old female is unresponsive and apneic after being ejected from a vehicle. What is the best way to manage her airway?
    A. Jaw-thrust maneuver
    B. Endotracheal intubation
    C. Sellick’s maneuver
    D. Esophageal intubation
    B. Endotracheal intubation
  27. An alert 18 year old female is trapped in her car. Her waist and legs are pinned against the dashboard and she complains of hip and left leg pain. After a prolonged extrication from the car, her vital signs begin to deteriorate. What should you suspect?
    A. Neurogenic shock
    B. Crush syndrome
    C. Air embolism
    D. Internal injuries
    B. Crush syndrome
  28. Which of the following enzymes is most specific for cardiac injury?
    A. Myoglobin
    B. Troponin
    C. Creatine kinase
    D. Lactic dehydrogenase
    B. Troponin
  29. You are treating a 56 year old female with severe hypoglycemia who is unconscious. How should you elevate her blood sugar?
    A. Give glucose orally or IM if there is no gag reflex
    B. Try glucagon IM, if it fails give dextrose IO
    C. Give dextrose IV, if IV fails give glucagon IM
    D. Give glucagon IM or administer glucose buccally
    C. Give dextrose IV, if IV fails give glucagon IM
  30. A 22 year old male fell off a ladder. His respiratory rate is 10 and lung sounds are diminished bilaterally. What should you do?
    A. Perform nasotracheal intubation.
    B. Check for a carotid pulse.
    C. Assist his ventilations.
    D. Place a cervical collar.
    C. Assist his ventilations.
  31. A 37 year old female complains of a possible overdose. She is hyperventilating and tachycardic. Which drug is most likely to result in her symptoms?
    A. Opiates
    B. Methamphetamine
    C. Alcohol
    D. Beta-blockers
    B. Methamphetamine
  32. You are the first ambulance to arrive on the scene of a school bus rollover. The fire department has stabilized the bus. What should you do?
    A. Have your partner begin triaging patients.
    B. Wait for your manager to arrive on scene.
    C. Load and transport the first patient you find.
    D. Begin filling out patient care reports.
    A. Have your partner begin triaging patients.
  33. What is a temporary occlusion caused by a coronary artery spasm known as?
    A. Prinzmetal’s angina
    B. Acute angina pectoris
    C. Unstable angina
    D. Pre-infarct angina
    A. Prinzmetal’s angina
  34. A 26 year old female is supine and only responding to voice with unintelligible gurgling noises. Her husband tells you she is an epileptic. She had a seizure during which he was able to gently assist her to the ground. What should you do?
    A. Load the patient on a stretcher and apply soft restraints.
    B. Roll the patient onto her side and suction the airway.
    C. Perform a thorough trauma assessment and patient history.
    D. Apply a c-collar and place her on a long backboard.
    B. Roll the patient onto her side and suction the airway.
  35. What causes carpopedal spasms in patients who are hyperventilating?
    A. Hypercalcemia
    B. Hypocalcemia
    C. Hypocapnea
    D. Clonic focal motor seizures
    B. Hypocalcemia
  36. A 64 year old female complains of weakness and nausea. A 12 Lead ECG shows normal sinus rhythm with ST elevation in V1 and V2. Vital signs are BP 132/64, R 18, P 98. What should you do?
    A. Administer nitroglycerin
    B. Administer aspirin
    C. Administer ondansetron
    D. Administer morphine
    B. Administer aspirin
  37. What medication is an appropriate treatment for anaphylaxis?
    A. Nitroglycerin
    B. Epinephrine
    C. Naloxone
    D. Albuterol
    B. Epinephrine
  38. The spread of an electrical impulse through the ventricles that normally measures 0.08-0.12 seconds in length is represented on an ECG as the
    A. QRS complex.
    B. T wave.
    C. P wave.
    D. Atrial complex.
    A. QRS complex.
  39. What should you do when you suspect child abuse?
    A. Question the parent about the events leading to the injury.
    B. Remove the child from the abuser’s care.
    C. Confirm your suspicions of abuse.
    D. Report your suspicions to the appropriate authorities.
    D. Report your suspicions to the appropriate authorities.
  40. What type of pain allows an examiner to pinpoint the area of irritation?
    A. Somatic
    B. Visceral
    C. Referred
    D. Parietal
    A. Somatic
  41. What class of medication causes the uterus to contract?
    A. Antipyretics
    B. Oxytocics
    C. Antihistamines
    D. Steriods
    B. Oxytocics
  42. What is the innermost layer of the heart?
    A. Hypercardium
    B. Endocardium
    C. Myocardium
    D. Epicardium
    B. Endocardium
  43. A 25 year old female is found lying in the shrubs on a college campus. Her clothing is torn and she is covered in dirt. She does not remember how she got there and cannot answer all of your questions. What should you do?
    A. Perform a detailed assessment
    B. Check her blood glucose
    C. Call for police assistance
    D. Notify her parents
    C. Call for police assistance
  44. A 57 year old male complains of bleeding with defecation. He claims no significant medical history, has had no recent illness, and takes no medications. What should you suspect?
    A. Pancreatitis
    B. Viral gastroenteritis
    C. Ulcerative colitis
    D. Cholecystitis
    C. Ulcerative colitis
  45. Where in the respiratory system does gas exchange occur?
    A. Alveoli
    B. Bronchioles
    C. Pharynx
    D. Larynx
    A. Alveoli
  46. What is the study of the drug receptor interaction called?
    A. Pharmacology
    B. Physiology
    C. Pharmacodynamics
    D. Pharmacokinetics
    C. Pharmacodynamics
  47. What are the primary ions involved in action potential synthesis and transmission?
    A. K+, Na+, Hg+++
    B. Na+, K+, Ca++
    C. Ca++, Na+
    D. Ca++, H+
    B. Na+, K+, Ca++
  48. An 80 year old male is confused and in his apartment on a hot summer day. Vital signs are BP 80/P, P 132 and regular, R 40 and shallow, skin is hot and dry. What should you suspect?
    A. Heatstroke
    B. Hypoglycemia
    C. Heat exhaustion
    D. Dementia
    A. Heatstroke
  49. What will initial compensatory mechanisms do during hemorrhaging?
    A. Decrease respiratory rate and increase heart rate.
    B. Increase stroke volume and increase cardiac output.
    C. Increase cardiac output and reduce mean arterial pressure.
    D. Decrease peripheral vascular resistance and increase respiratory rate.
    B. Increase stroke volume and increase cardiac output.
  50. What molecule are ketone bodies metabolized from?
    A. Carbohydrates
    B. Lipids
    C. Glycogen
    D. Proteins
    B. Lipids
  51. What neural signal prevents the lungs from overinflating?
    A. Crutzfelts-Jacobs reflex
    B. Epstein-Barr reflex
    C. Frank-Starling reflex
    D. Hering-Breuer reflex
    D. Hering-Breuer reflex
  52. A 72 year old female woke with dyspnea. Crackles are audible to her scapulae bilaterally. She is coughing frothy pink sputum. Vital signs are BP 158/98, P 128, R 8. What should you do to improve her minute volume?
    A. Administer nitroglycerin 0.4 mg sublingual
    B. Administer furosemide 1 mg/kg IV at 20 mg/min
    C. Intubate and ventilate with a BVM
    D. Provide oxygen by non-rebreather mask
    C. Intubate and ventilate with a BVM
  53. A 55 year old female was in a motor vehicle crash. She starts vomiting while you attempt to nasotracheally intubate her. What should you do?
    A. Remove the tube and use an ET tube.
    B. Auscultate breath sounds over the lungs and epigastrium.
    C. Remove the tube and clear the airway.
    D. Attempt to intubate through her other nostril.
    C. Remove the tube and clear the airway.
  54. A 43 year old patient was struck in the chest and has a large bruise over her sternum. She states that her heart “feels like it is beating funny. The 3 lead ECG shows sinus tachycardia with ST elevation and frequent unifocal premature ventricular contractions (PVCs). Vital signs are BP 108/80, P 124 and irregular, R 22. What medication should you administer?
    A. Adenosine, 6 mg over five minutes
    B. Amiodarone, 150 mg over 10 minutes
    C. Lidocaine, 1.0 mg/kg over 10 minutes
    D. Magnesium, 1.5 mg over 5 minutes
    B. Amiodarone, 150 mg over 10 minutes
  55. A 30 year old female is delivering a newborn. Which areas of the newborn’s head should you avoid touching?
    A. Eyes and jaw
    B. Mouth and fontanelles
    C. Mouth and nose
    D. Eyes and fontanelles
    D. Eyes and fontanelles
  56. What does asymmetrical chest wall movement that weakens respiratory efficiency indicate?
    A. Flail chest
    B. Pneumothorax
    C. Tracheal obstruction
    D. Air embolism
    A. Flail chest
  57. Which of the following should be documented on any patient refusal report?
    A. The style and type of vehicle the patient drives
    B. The last time the patient saw a primary care physician
    C. The patient’s job occupation
    D. The willingness for EMS to return if needed
    D. The willingness for EMS to return if needed
  58. What is a contraindication of an oropharyngeal airway?
    A. Broken teeth
    B. A basal skull fracture
    C. A gag reflex
    D. A GCS greater than 5
    C. A gag reflex
  59. What occurs naturally in white blood cells and produces allergic and inflammatory reactions?
    A. Histamine blockers
    B. Leukotrienes
    C. Eosinophils
    D. Thromboxanes
    B. Leukotrienes
  60. The negative electrode is placed on the right arm, the positive electrode is placed on the left leg, the ground lead is on the left arm. This represents the correct lead placement for what Lead?
    A. MCL Lead.
    B. Lead I.
    C. Lead II.
    D. Lead III.
    C. Lead II.
  61. An unresponsive 20 yo male struck his head. Vital signs are BP 180/20, P 40, R 8. What should you do?
    A. Perform endotracheal intubation
    B. Ventilate using a BVM at 12/min
    C. Apply oxygen 10lpm via mask
    D. Administer 0.5 mg Atropine
    D. Administer 0.5 mg Atropine
  62. Expiratory stridor indicates obstruction or narrowing of what part of the airway?
    A. Lower trachea
    B. Supraglottic
    C. Larynx
    D. Upper trachea
    B. Supraglottic
  63. What is the phase after a generalized seizure called?
    A. Postictal
    B. Aura
    C. Psychosis
    D. Translucent
    A. Postictal
  64. What can you administer to manage the airway of a patient whose jaw is clenched?
    A. Neuromuscular blocking agent
    B. Parasympatholytic agent
    C. Potent sedative agent
    D. Vagal reducing agent
    A. Neuromuscular blocking agent
  65. What is the first sign of decompensated shock?
    A. A decrease in blood pressure
    B. A decrease in body temperature
    C. A decrease in mental status
    D. A decrease in respiratory rate
    C. A decrease in mental status
  66. You are completing your patient care report and cannot remember the correct spelling of a medical term. What should you do?
    A. Attempt the correct spelling.
    B. Use plain English instead.
    C. Cross out the sentence.
    D. Make up an abbreviation.
    B. Use plain English instead.
  67. A 28 year old female is 30 weeks pregnant with twins. Her water has broken and she feels like she has to move her bowels. The nearest hospital is 40 miles away. A medical helicopter is responding but is 45 minutes from your location. What should you do?
    A. Load and go and deliver the babies en route
    B. Call for an additional ambulance
    C. Prepare for imminent delivery
    D. Administer brethine and wait for the helicopter
    C. Prepare for imminent delivery
  68. A patient has been struck by lightning. You should be most concerned that the electrical current caused him to do which of the following?
    A. Become apneic
    B. Suffer circumferential burns
    C. Develop respiratory alkalosis
    D. Suffer brain damage
    A. Become apneic
  69. Where should you perform a surgical airway?
    A. At the larynx superior to the hyoid bone.
    B. Superior to the thyroid cartilage.
    C. Caudally and at an angle to the cricoid cartilage.
    D. Midline through the cricothyroid membrane.
    D. Midline through the cricothyroid membrane.
  70. What abdominal complaint is MOST concerning?
    A. Pain that is relieved by rocking or movement.
    B. A sudden, tearing pain.
    C. Pain that increase with eating.
    D. A dull, gradual-onset pain.
    B. A sudden, tearing pain.
  71. A 73 year old female is her left side and responsive only to pain. You note a fruity odor on her breath. Her blood glucose is 431 mg/dL. Vital signs are BP 96/80, P 120, R 44. What should you administer?
    A. Furosemide
    B. Sodium bicarbonate
    C. Normal saline
    D. Diazepam
    C. Normal saline
  72. Which cells within the islets of Langerhans produce the hormone insulin?
    A. Beta
    B. Delta
    C. Gamma
    D. Alpha
    A. Beta
  73. A 46 year old male was struck in the eye with a racquet ball. He complains of eye pain, cheek pain and bleeding from one nostril. Upon exam his eye will move side to side but not up or down. What should you suspect?
    A. Le fort II Fracture
    B. Orbital blow-out fracture
    C. Le fort I Fracture
    D. Le fort III Fracture
    B. Orbital blow-out fracture
  74. A 6 year old boy was struck by a car and thrown 30 feet. His skin is pale. He is unconscious and unresponsive to painful stimulus. Vital signs are BP 80/30, P 160, R 60 and shallow. Which of the following best describes his condition?
    A. Decompensated shock with respiratory distress
    B. Compensated shock with respiratory distress
    C. Decompensated shock with respiratory failure
    D. Compensated shock with respiratory failure
    C. Decompensated shock with respiratory failure
  75. What classification is give to drugs that have accepted medical indications but a high abuse potential that may lead to physical or psychological dependence?
    A. Schedule II
    B. Schedule III
    C. Schedule IV
    D. Schedule V
    A. Schedule II
  76. The end tidal CO2 detector is reading 0 mmHg after endotracheal intubation. Where does this indicate your tube has been placed?
    A. Left mainstem bronchus
    B. Esophagus
    C. Trachea
    D. Right mainstem bronchus
    B. Esophagus
  77. A 50 year old male complains of light headedness and chest discomfort. Vital signs are BP 100/86, P 40, R 16. An ECG reveals a second degree type I AV block. What should you do?
    A. Start transcutaneous pacing
    B. Administer atropine
    C. Administer amiodarone
    D. Start a dopamine drip
    B. Administer atropine
  78. Which of the following hormones affects the body’s biological clock?
    A. Calcitonin
    B. Cortisol
    C. Melatonin
    D. Aldosterone
    C. Melatonin
  79. Your successfully placed Endotracheal tube with a capnometry of 29 suddenly decreases to 4. What method would you use to determine the cause?
    A. LEMONS
    B. MOANS
    C. RODS
    D. DOPE
    D. DOPE
  80. When communicating via the radio to your receiving hospital, it is common practice to use the “echo procedure.” This procedure
    A. ensures that only the important facts have been communicated.
    B. confirms accurate reception and understanding.
    C. eliminates the use of words that are difficult to understand during the transmission..
    D. conforms to HIPPA by protecting the patient’s privacy.
    B. confirms accurate reception and understanding.
  81. “https://testing.fisdap.net/testing_images/MH2009/aeh82892_3512_11412.jpg”>
    In this image, what does the circles dark tissue of the heart illustrate?
    A. cardiac contusion
    B. pericardial effusion
    C. pulmonary contusion
    D. cardiac laceration
    A. cardiac contusion
  82. A 55 year old male patient complains of shortness of breath and chest tightness. His skin is pale and cool, and his respirations are 30 and shallow. You should deliver oxygen via:
    A. Nonrebreather mask.
    B. Bag-valve mask.
    C. Nasal cannula.
    D. CPAP device.
    B. Bag-valve mask.
  83. A joint dislocation occurs when the normal articulating ends of two or more bones are displaced. A complete dislocation is called a __.
    A. Luxation
    B. sprain
    C. subluxation
    D. strain
    C. subluxation
  84. A contraindication for the use of morphine sulfate is:
    A. pulmonary edema
    B. kidney stones
    C. hypovolemia
    D. chest pain
    C. hypovolemia
  85. How should you treat an impaled object in the abdomen?
    A. Remove the object, and apply a porous dressing moistened with sterile saline to the wound.
    B. Secure the object, control bleeding, and use bulky dressings to stabilize the object.
    C. Remove the object, apply an occlusive dressing to the wound.
    D. Manual secure the object and inflate the pneumatic anti-shock garment to stabilize the object.
    B. Secure the object, control bleeding, and use bulky dressings to stabilize the object.
  86. A 34 year old female complains of an open sore on her leg that will not go away. She says that she cut her leg while shaving in the shower at her local gym 2 weeks ago. Her doctor prescribed antibiotics, but nothing has changed. What should you suspect?
    A. Boils
    B. MRSA
    C. Varicose vein
    D. Necrotizing fasciitis
    B. MRSA
  87. After intubation you hear gurgling over the stomach and no breath sounds over the chest. What should you suspect?
    A. Esophageal intubation
    B. Right mainstem placement
    C. A malfunction of the ET tube
    D. Proper placement
    A. Esophageal intubation
  88. When examining the abdomen of a female who is five months pregnant, where would you expect to palpate the superior portion of the uterus?
    A. Above the symphysis pubis
    B. Hypochondriac region
    C. Epigastric region.
    D. At the umbilicus
    D. At the umbilicus
  89. A 26 year old female is trapped after a motor vehicle collision. The driver’s side window is broken, the airbags did not deploy, and the car came to rest on a steep grassy slope. When approaching her, what should you do?
    A. Pop the hood to disconnect the battery.
    B. Cut out the windshield to gain access.
    C. Have her sit perfectly still.
    D. Place cribbing under the tires.
    D. Place cribbing under the tires.
  90. What is the leading cause of infant and child death?
    A. Allergic reactions
    B. Congenital defects
    C. Vascular stenosis
    D. Respiratory arrest
    D. Respiratory arrest
  91. What is the initial effect of parasympathetic stimulation on the heart?
    A. Decrease the heart rate and decrease stroke volume.
    B. Increase the heart rate and increase stroke volume.
    C. Increase the heart rate and decrease stroke volume.
    D. Decrease the heart rate and increase stroke volume.
    A. Decrease the heart rate and decrease stroke volume.
  92. Firefighters pulled a 44 year old male from a burning house. His lips and nasal hairs are burnt. Vital signs are BP 160/90, P 98, R 32, SaO2 94%. What should you do?
    A. Insert a combitube
    B. Insert an oropharyngeal airway
    C. Endotracheally intubate
    D. Ventilate using a bag-valve-mask
    C. Endotracheally intubate
  93. A patient with a history of sickle cell disease states he is in another crisis. You should
    A. administer a benzodiazepine hypnotic.
    B. assist the patient in taking his own pain medication.
    C. start a large bore IV and administer 20 mL/kg.
    D. administer high-flow oxygen and transport.
    D. administer high-flow oxygen and transport.
  94. A 13 year old unconscious male and a 15 year old conscious female were in a motor vehicle collision. What should you do?
    A. Assess and treat only after a police officer gives you permission.
    B. Wait to treat both of the patients until you obtain parental consent.
    C. Wait to assess the 13 year old until a parent arrives.
    D. Treat the 13 year old under implied consent.
    D. Treat the 13 year old under implied consent.
  95. After fertilization of an egg, where will embryo develop and mature?
    A. Cervix
    B. Fallopian tube
    C. Uterus
    D. Vagina
    C. Uterus
  96. A 56 year old female is developing neurologic abnormalities during her hemodialysis. What should you suspect?
    A. Hypotension
    B. Acute myocardial infraction
    C. Hyperglycemia
    D. Accumulated blood urea
    D. Accumulated blood urea
  97. What disease involves inflammation followed by demyelination of the brain and spinal cord nerve fibers?
    A. Alzheimer’s disease
    B. Myasthenia gravis
    C. Muscular dystrophy
    D. Multiple sclerosis
    D. Multiple sclerosis
  98. What is the medical term for decreased or a lack of urine production?
    A. Anuria
    B. Hematuria
    C. Polyuria
    D. Oliguria
    A. Anuria
  99. What happens to hemoglobin when the blood is more acidic?
    A. Binds carbon dioxide
    B. Releases oxygen
    C. Binds oxygen
    D. Releases carbon dioxide
    B. Releases oxygen
  100. How can persistent fetal circulation be triggered?
    A. Induced labor
    B. Hypothermia
    C. A hypovolemic state
    D. Alkalosis caused by maternal drug use
    B. Hypothermia
  101. Which neurotransmitter is found in the post ganglionic synaptic terminals of the sympathetic nerves?
    A. Acetylcholine
    B. Serotonin
    C. Norepinephrine
    D. Gamma-aminobutyric acid
    C. Norepinephrine
  102. A 76 year old nursing home patient needs weight based medication. The transfer papers say the patient weighs 154 lbs. How many kilograms does the patient weigh ?
    A. 308
    B. 70
    C. 80
    D. 67
    B. 70
  103. What is a typical dose for Ativan for seizures?
    A. 200 mcg IV
    B. 2 g IV
    C. 2 gr IV
    D. 2-4 mg IV
    D. 2-4 mg IV
  104. A 72 year old female overdosed on Norvasc (amlodipine) four hours ago. Vital signs are BP 70/50, P 40, R 20. What should you do?
    A. Dopamine drip
    B. Calcium chloride IV bolus
    C. Normal saline bolus
    D. Start transcutaneous pacing
    D. Start transcutaneous pacing
  105. A 19 year old female gives birth to a baby boy. You note blue extremities and a pink trunk. His pulse is 108 and he cries weakly. He grimaces when you suction him, but other movement is limited. What is his APGAR score?
    A. 4
    B. 8
    C. 6
    D. 2
    C. 6
  106. Why is the electrical impulse slightly delayed at the AV node?
    A. To allow the primary cardiac pacemaker to reset.
    B. To allow the bundle of His can depolarize fully.
    C. So the atria can empty into the ventricles.
    D. So the ventricles can contract completely.
    C. So the atria can empty into the ventricles.
  107. What is inflammation of the membranes surrounding the brain called?
    A. Meningitis
    B. Encephalitis
    C. Aneurysm
    D. Hydrocephalis
    A. Meningitis
  108. A 66 year old female has shortness of breath and diffuse back pain. Auscultation of lungs reveal crackles in the bases. Vital signs are BP 170/110 and P 112. What should you do administer?
    A. Furosemide
    B. Nitroglycerin
    C. Albuterol
    D. Adenosine
    B. Nitroglycerin
  109. Which part of the nervous system is responsible for basic reflexes like chewing and swallowing?
    A. Amygdala
    B. Cranial nerves
    C. Midbrain
    D. Medulla oblongata
    B. Cranial nerves
  110. A 28 year old female complains of dyspnea while eating dinner at a restaurant. She has audible wheezing and facial swelling with urticaria. Vital signs are BP 100/72, P 130, R 24, SpO2 94%. What should you do?
    A. Administer diphenhydramine.
    B. Establish two large bore IVs.
    C. Position her supine.
    D. Administer high-flow oxygen.
    D. Administer high-flow oxygen.
  111. Why do you avoid flexion of the wrist once the laryngoscope is in the mouth?
    A. Injury to the upper teeth
    B. Laceration of the vallecula
    C. Injury to the lower teeth
    D. Laceration of the tongue
    A. Injury to the upper teeth
  112. You are at a chemical plant where 12 workers are exposed to chemical fumes. They have severe dyspnea and are moaning on the ground. What should you do?
    A. Administer 2 mg of Atropine every three to five minutes
    B. Administer oxygen at 15 LPM via NRB mask
    C. Set up an incident command post
    D. Begin triage
    C. Set up an incident command post
  113. In the Incident Management System, what is the Liaison Officer is responsible for?
    A. Supporting incident operations.
    B. All of the on scene actions.
    C. Collecting data from the incident and releases it to the press.
    D. Coordinating all operations involving outside agencies.
    D. Coordinating all operations involving outside agencies.
  114. A 57 year old female was the unrestrained driver in a frontal impact collision. Which of the following injuries would lead you to believe she traveled “up and over” the dashboard, instead of “down and under”?
    A. A patellar dislocation
    B. A flail chest
    C. A ruptured diaphragm
    D. A cervical spine fracture
    C. A ruptured diaphragm
  115. An elderly male fell 12 feet onto a concrete surface and complains of head, neck, and back pain. He states he is having difficulty breathing. You note fractures of his right upper arm and lower leg. What should you do?
    A. Endotracheally intubate and immobilize the spine.
    B. Support the cervical spine while managing the airway.
    C. Perform a detailed assessment including vital signs.
    D. Take spinal precautions and apply the PASG.
    B. Support the cervical spine while managing the airway.
  116. What animal carries Lyme disease?
    A. Wasp
    B. Bee
    C. Spider
    D. Tick
    D. Tick
  117. An alert 75 year old male after slipped on his wet sidewalk. He is sober and denies hitting his head. He states his legs just went out from under him and he fell to his right knee. His right leg is internally rotated and shortened. Vital signs are BP 110/40, P 96, R 18. What should you suspect?
    A. Hip fracture
    B. Femur fracture
    C. Hip dislocation
    D. Spinal injury
    C. Hip dislocation
  118. A 42 year old male has itchy skin and hives on his lower right leg. He states that he was mowing the lawn when this occurred. Vital signs are BP 146/84, P 92 and regular, and R16 unlabored. What should you do?
    A. Administer diphenhydramine.
    B. Apply an ice pack to the rash.
    C. Administer epinephrine.
    D. Apply antibiotic ointment.
    A. Administer diphenhydramine.
  119. You arrive on the scene of a three-car motor vehicle collision. Two of the cars have major damage and some debris is scattered in the roadway. What should you do?
    A. Immediately begin triage of the injured patients.
    B. Call your supervisor to provide incident command.
    C. Contact your dispatch and request more ambulances.
    D. Ensure your vehicle is parked out of oncoming traffic.
    D. Ensure your vehicle is parked out of oncoming traffic.
  120. When should continuous positive airway pressure (CPAP) be discontinued?
    A. Patient develops hypotension
    B. Patient has crackling lung sounds
    C. Patient’s symptoms are resolved
    D. Patient is wheezing
    A. Patient develops hypotension
  121. The body will not tolerate an acute blood loss of greater than __ of blood volume.
    A. 30%
    B. 10%
    C. 20%
    D. 40%
    D. 40%
  122. A 40 year old female has chronic bronchitis and has a productive cough of white sputum. She also has JVD, peripheral edema, and hypercapnia. An ECG shows an increase P wave amplitude. What else should you suspect?
    A. Emphysema
    B. Asthma
    C. Picwickian Syndrome
    D. Cor Pulmonale
    D. Cor Pulmonale
  123. A 29 year old male was an unrestrained driver of a car crash. Vital signs are BP 114/98, P 136 and weak, R 28 and shallow. He has jugular vein distension and breath sounds are equal bilaterally. Heart sounds are very distant. What should you suspect?
    A. Pericardial tamponade
    B. Hemothorax
    C. Tension pneumothorax
    D. Simple pneumothorax
    A. Pericardial tamponade
  124. A 28 year old 32 week gestation patient complains of severe hand pain and deformity after slamming it in a car door. Vital signs are BP 110/70, P 80, R 14. In what position should you transport her?
    A. Left lateral
    B. Supine
    C. Prone
    D. Sim’s position
    A. Left lateral
  125. Which cranial nerves are involved in the cardinal positions of gaze?
    A. I, III, and IV
    B. VI, VII, and X
    C. III, IV, and VI
    D. I, III, and VI
    C. III, IV, and VI
  126. A 45 year old male was submerged in a lake for 1-2 minutes. The outside temperature is 45 degrees. He is pulseless and apneic. What is your primary concern?
    A. Laryngeal spasm
    B. Asphyxia
    C. Pulmonary edema
    D. Hypothermia
    B. Asphyxia
  127. For what medical condition is metformin prescribed?
    A. Type II diabetes mellitus
    B. Diabetic ketoacidosis
    C. Diabetes insipidus
    D. Type I diabetes melitus
    A. Type II diabetes mellitus
  128. A 16 year old unconscious male was pulled from a swimming pool. What should you do?
    A. Assess for a carotid pulse
    B. Clear the his airway of aspirated water
    C. Stabilize his c-spine
    D. Ensure an open airway and assess breathing
    A. Assess for a carotid pulse
  129. Which of the following causes unilateral paralysis of cranial nerve VII?
    A. Autonomic dysreflexia
    B. Bell’s palsy
    C. Myoclonus
    D. Dystonia
    B. Bell’s palsy
  130. A 19 year old female with head trauma has hypertension, bradycardia, Cheynes-Stokes respirations and withdraws from pain. Which area of the brain is likely affected?
    A. Temporal lobe
    B. Medulla
    C. Cerebellum
    D. Hypothalamus
    B. Medulla
  131. List all regions of human vertebral column.
    A. Intervertebral, coccyx, lumbar, thoracic.
    B. Cervical, thoracic, lumbar, sacral, coccygeal, coccyx.
    C. Cervical, thoracic, lumbar, sacral, coccygeal.
    D. Cervical, thoracic, lumbar, coccygeal, coccyx
    C. Cervical, thoracic, lumbar, sacral, coccygeal.
  132. What is one of the best ways to minimize anxiety during a physical examination?
    A. Drive slowly to the hospital.
    B. Warm your hands and instruments prior to their use.
    C. Keep the patient’s clothes on.
    D. Use a calm, professional demeanor.
    D. Use a calm, professional demeanor.
  133. What effect does AIDS have on the body’s immune system?
    A. It mutates into pneumonia.
    B. It changes the function of erythrocytes.
    C. It causes HIV to replicate and spread.
    D. It suppresses the T cell function.
    D. It suppresses the T cell function.
  134. You are treating a hypotensive woman who weighs 212 lbs. You have been ordered to give her a Dopamine drip at 9 mcg/kg/min. Your ambulance service carries 250cc bags. How much dopamine would you mix in the bag to reach a dilution of 1600 mcg/mL?
    A. 200mg
    B. 800mg
    C. 1000mg
    D. 400mg
    D. 400mg
  135. A 28 year old female is complaining of gradual-onset dyspnea after being released from the hospital. She was hospitalized for 1 day following thoracic trauma and a lung contusion. Lung sounds present with crackles in both bases. Vital signs are BP 130/80, P 110, R 32 shallow, SpO2: 88%. You should
    A. bilaterally decompress the chest, reassess vital signs, and treat accordingly.
    B. apply oxygen, establish an IV, administer morphine, and transport.
    C. apply oxygen, administer nitroglycerin and transport, establish an IV, and administer Lasix enroute.
    D. apply oxygen via non-rebreather, transport emergently, and establish an IV enroute.
    D. apply oxygen via non-rebreather, transport emergently, and establish an IV enroute.
  136. Room air contains what percentage of oxygen?
    A. 100
    B. 21
    C. 36
    D. 10
    B. 21
  137. A 59 year old female has a severe asthma attack. What is the dose and route of ipratropium bromide you should administer?
    A. 0.125-0.25 mg, nebulized
    B. 0.5 mg, nebulized
    C. 0.3-0.5 mg, subcutaneously.
    D. 0.25 mg, subcutaneously
    B. 0.5 mg, nebulized
  138. Where do afferent fibers transmit impulses from?
    A. The central nervous system to the sensory nerves.
    B. Motor nerves to the central nervous system.
    C. Sensory nerves to the central nervous system.
    D. The central nervous system to the motor nerves.
    C. Sensory nerves to the central nervous system.
  139. What sign or symptom may result from a digoxin overdose?
    A. Hypokalemia
    B. Bradycardia
    C. Hypoglycemia
    D. Altered mental status
    B. Bradycardia
  140. You deliver a baby boy without difficulty. He is blue and not crying. What should you do?
    A. Obtain a set of vital signs and cut the umbilical cord.
    B. Allow him to begin feeding to promote oxytocin stimulation.
    C. Start compressions and ventilate with supplemental oxygen.
    D. Dry and warm him, stimulate with a back rub, and suction.
    D. Dry and warm him, stimulate with a back rub, and suction.
  141. You are called to evaluate a 28 year old male. He is afraid of you and recoils when you approach him. He states he is hearing voices and appears to be hallucinating. What should you suspect?
    A. Anxiety attack
    B. Bipolar disorder
    C. Mood disorder
    D. Schizophrenia
    D. Schizophrenia
  142. What structure in the brain controls ventilation?
    A. Medulla
    B. Cerebellum
    C. Mesencephalon
    D. Diencephalon
    A. Medulla
  143. What type of procedure does a patient with chronic renal failure undergo?
    A. Renal hemolysis
    B. Peritoneal hemolysis
    C. Renal calculi
    D. Renal dialysis
    D. Renal dialysis
  144. What is an effect of epinephrine in the treatment of anaphylaxis?
    A. Negative chronotropic effect
    B. Inflammatory decrease
    C. Peripheral vasodilation
    D. Bronchoconstriction
    B. Inflammatory decrease
  145. A 52 year old patient with a history of liver failure complains of shortness of breath and abdominal swelling. What should you suspect?
    A. Acute MI
    B. Aortic aneurysm
    C. Bladder blockage
    D. Ascites
    D. Ascites
  146. How many times faster does heat loss occur during water immersion as compared to air exposure?
    A. 2
    B. 50
    C. 25
    D. 100
    C. 25
  147. Wolf Parkinson White syndrome originates from accessory pathways in what part of the heart?
    A. Purkinje fibers
    B. Bundle Branch
    C. Bundle of Kent
    D. Bundle of His
    C. Bundle of Kent
  148. A 45 year old male has been injured in a house fire and has burnt nasal hairs. He also makes high-pitch noises with inspiration. How are these respirations defined?
    A. Wheezing
    B. Snoring
    C. Stridor
    D. Rhonchi
    C. Stridor
  149. What is a high pitched noise heard during inspiration in a partial airway obstruction?
    A. Cough
    B. Dyspnea
    C. Stridor
    D. Dyphonia
    C. Stridor
  150. What is the normal arterial pO2 level at sea level?
    A. 80-100 mmHg
    B. 60-80 mmHg
    C. 40-60 mmHg
    D. 100-120 mmHg
    A. 80-100 mmHg
  151. Your patient has a small, sucking puncture wound on the right side of his chest. How should you dress the wound?
    A. Moist gauze.
    B. Roller gauze.
    C. An occlusive dressing.
    D. Butterfly bandages.
    C. An occlusive dressing.
  152. Which of the following is the first sign of shock that usually appears in patients?
    A. Altered mental status
    B. Nausea and vomiting
    C. Changes in vital signs
    D. Pale, cool, and clammy skin
    A. Altered mental status
  153. A patient has lost 800 mL of blood. Vital signs are BP 110/80, P 100, R 24. Which class of hemorrhagic shock are they experiencing?
    A. Class 2
    B. Class 3
    C. Class 1
    D. Class 4
    A. Class 2
  154. What should you do to control post-delivery bleeding?
    A. Massage gently over her uterus.
    B. Squeeze her legs together.
    C. Place her in the recovery position.
    D. Place moist dressings in her uterus.
    A. Massage gently over her uterus.
  155. What is the painless, progressive enlargement of tissue found mainly in the lymph nodes and spleen known as?
    A. Leukemia
    B. Polycythemia
    C. Hemolytic anemia
    D. Hodgkin’s disease
    D. Hodgkin’s disease
  156. Why do you administer epinephrine to a patient in cardiac arrest?
    A. Increased rate of conduction.
    B. Increased blood flow to the heart and brain.
    C. Increased contractility of cardiac muscle.
    D. Increased bronchodilation.
    B. Increased blood flow to the heart and brain.
  157. A 25 year old female has bruising and swelling to the left side of her throat. Subcutaneous emphysema and cyanosis are present. What should you suspect?
    A. Pneumothorax
    B. A ruptured esophagus
    C. Esophageal varices
    D. A tracheal tear
    D. A tracheal tear
  158. What is the coarse sound of fluid movement heard on inspiration called?
    A. Rhonchi
    B. Snoring
    C. Crackles
    D. Gurgling
    A. Rhonchi
  159. What is the process that causes a solute to move from a gradient of greater concentration to lower concentration called?
    A. Active transport
    B. Facilitated diffusion
    C. Osmosis
    D. Diffusion
    D. Diffusion
  160. Which is an alkaloid blood pH?
    A. 7.45
    B. 7.35
    C. 7.30
    D. 7.25
    A. 7.45
  161. Which anacronym is used to predict a difficult endotracheal intubation?
    A. LEMONS
    B. MOANS
    C. RODS
    D. SHORT
    A. LEMONS
  162. Why do patient refusals require more documentation than the typical EMS call?
    A. A greater set of differential diagnoses needs to be noted as the patient was left untreated.
    B. There is greater potential for abandonment charges.
    C. There are additional witnesses that must be quoted.
    D. You need to note the interventions you would have performed had they given consent.
    B. There is greater potential for abandonment charges.
  163. HIPAA legislation relates primarily to which medicolegal issue?
    A. Abandonment
    B. Consent
    C. Duty to act
    D. Confidentiality
    D. Confidentiality
  164. What gland joins the nervous system with the endocrine system
    A. Pineal
    B. Hypothalamus
    C. Thymus
    D. Pituitary
    B. Hypothalamus
  165. You are at an amusement park when you witness a 19 year old male being stabbed in the arm. After assuring the scene is safe, you approach and see that he is experiencing slow, steady bleeding. You have disposable gloves in your pocket, but no bandages. What should you do?
    A. Elevate the arm and use a pressure point to control bleeding.
    B. Send a bystander to locate a tourniquet.
    C. Locate a first aid kit and control the bleeding with bandages.
    D. Apply direct pressure to the wound using your gloved hand.
    D. Apply direct pressure to the wound using your gloved hand.
  166. One way to increase preload is fluid administration. This affects
    A. Hans Mobitz Reflex.
    B. Poiseuille’s Law.
    C. Frank Starling’s Law.
    D. Hering-Breuer Reflex.
    C. Frank Starling’s Law.
  167. A 20 year old male was shot in the leg with an arrow. What should you do?
    A. Cut off most of the exposed arrow shaft, leaving enough to grip.
    B. Push the arrow through to the other side and cut off the arrowhead.
    C. Remove the arrow and apply a topical hemostatic agent to the wound.
    D. Remove the arrow and pack the wound with gauze.
    A. Cut off most of the exposed arrow shaft, leaving enough to grip.
  168. What is a contraindication of succinylcholine?
    A. Acute exacerbation of asthma.
    B. History suggestive of hyperkalemia.
    C. Decreased level of consciousness.
    D. Increased respiratory rate.
    B. History suggestive of hyperkalemia.
  169. Why do emphysema patients have a pink skin color?
    A. Bronchodilators and corticosteroids increase oxygen binding.
    B. Increased red blood cell production.
    C. Hypertrophy of the right ventricle.
    D. Pursed lip breathing increases PEEP and prevents alveolar collapse.
    B. Increased red blood cell production.
  170. Which of the following best describes simple extrication?
    A. Opening an undamaged door
    B. Using the jaws of life
    C. Using a hammer
    D. Using a center punch
    A. Opening an undamaged door
  171. A 70 year old female has a femur fracture. How much blood loss could she experience?
    A. 1500 mL
    B. 250 mL
    C. 3000 mL
    D. 500 mL
    A. 1500 mL
  172. In what part of the ECG phase is a shock delivered in synchronized cardioversion?
    A. P wave
    B. T wave
    C. S wave
    D. R wave
    D. R wave
  173. An unconscious 40 year old male has overdosed on alcohol and cocaine. The patient is a known diabetic. Vital signs: HR 130, BP 120/100, R8. You should
    A. Ventilate with a bag valve mask
    B. Check a blood glucose level
    C. Perform endotracheal intubation
    D. Establish intravenous access
    A. Ventilate with a bag valve mask
  174. A 67 year old female with a suspected stroke, has difficulty swallowing and is unable to speak. Which cranial nerve is affected?
    A. Abducens
    B. Hypoglossal
    C. Trochlear
    D. Vestibulocochlear
    B. Hypoglossal
  175. A 30 year old male complains of difficulty breathing with a productive cough and brown sputum. He has had fever and chills for 3 days. Lung sounds reveal crackles in lower lobes. Vital signs are BP 130/60, P 110, R 30. What should you do?
    A. Transport in position of comfort without further treatment.
    B. Place in position of comfort and apply oxygen by non-rebreather.
    C. Nasally intubate the patient and assist with his respirations using a BVM.
    D. Apply oxygen by non-rebreather and administer furosemide 40 mg IV.
    B. Place in position of comfort and apply oxygen by non-rebreather.
  176. When assessing a supine patient involved in a motor vehicle collision, the patient complains of left should pain. This indicates:
    A. Grey Turner’s sign
    B. Kehr’s sign
    C. Murphy’s sign
    D. Cullen’s sign
    B. Kehr’s sign
  177. What will compromise your ability to develop rapport and trust with a patient?
    A. Using the patient’s name as often as possible
    B. Speaking with a normal rate of speech
    C. Asking closed-ended questions
    D. Maintaining a calm appearance.
    C. Asking closed-ended questions
  178. What are agonal respirations?
    A. Deep, rapid, and regular breaths.
    B. Shallow, slow, and irregular gasping breaths.
    C. Deep, very slow and irregular breaths.
    D. Shallow, rapid, and regular panting breaths.
    B. Shallow, slow, and irregular gasping breaths.
  179. A 67 year old female is unconscious at the bottom of a flight of stairs. She responds to pain by stiffening her arms and legs but no external trauma is noted. Vital signs are BP 176/122, P 66, R 22 and irregular. What should you suspect?
    A. A lung injury
    B. Decompensated shock
    C. Hypovolemic shock
    D. A head injury
    D. A head injury
  180. In asystole, what is the first medication you would administer?
    A. Lidocaine
    B. Sodium Bicarbonate
    C. Atropine
    D. Epinephrine
    D. Epinephrine
  181. A 25 year old male attempted to commit suicide by taking 6 mg Lorazepam. What is your greatest concern?
    A. Circulatory alkalosis
    B. Agitated outbursts of anger
    C. Respiratory depression
    D. Hyperemesis
    C. Respiratory depression
  182. Why are elderly trauma patients at risk for fluid overload and pulmonary edema as a result of intravenous therapy?
    A. Decreased chest wall compliance and movement
    B. Decreased baroreceptor sensitivity to pressure
    C. Decreased renal function and stroke volume
    D. Decreased chemoreceptor sensitivity
    C. Decreased renal function and stroke volume
  183. What is the mechanism of action for amiodarone?
    A. Inhibits cardiac action potential in phase 0
    B. Slows AV conduction transmittance
    C. Prolongs cardiac action potential in phase 3
    D. Stabilizes all cardiac action potential membranes.
    C. Prolongs cardiac action potential in phase 3
  184. Which of the following drugs is contraindicated in asthma patients?
    A. Digitalis
    B. Calcium Chloride
    C. Epinephrine
    D. Propranolol
    D. Propranolol
  185. A 57 year old male with CHF has shortness of breath, coughing, and blood-tinged sputum. Vital signs are BP 140/88, P 86, R 28. What should you do?
    A. Aspirin, sublingual nitroglycerin, CPAP
    B. Morphine, aspirin, CPAP
    C. CPAP, morphine, intramuscular nitroglycerin
    D. Sublingual nitroglycerin, morphine, CPAP
    A. Aspirin, sublingual nitroglycerin, CPAP
  186. What is status epilepticus?
    A. Two or more continuous seizures
    B. When the patient sees an aura prior to a seizure
    C. A seizure that occurs during a fever
    D. A seizure affecting one area of the body
    A. Two or more continuous seizures
  187. A 60 year-old female presents with acute respiratory distress. She is conscious and confused with rapid respirations and clear, equal breath sounds. What should you do?
    A. Assist her respirations with a BVM.
    B. Provide oxygen via NRB mask.
    C. Use a nasal cannula at 2 L/min.
    D. Insert an oral airway adjunct.
    B. Provide oxygen via NRB mask.
  188. Which complication of pregnancy can occur during the first trimester?
    A. Placenta previa
    B. Eclampsia
    C. Abortion
    D. Abruptio placenta
    C. Abortion
  189. What blood type is the universal donor?
    A. A
    B. B
    C. O
    D. AB
    C. O
  190. Which of the following receptors are responsible for monitoring changes in blood pressure?
    A. Reticular activating receptors
    B. Chemoreceptors
    C. Baroreceptors
    D. Carboxyhemoglobin receptors
    C. Baroreceptors
  191. A 35 year old male has been electrocuted. What other life threat should you be prepared for?
    A. internal bleeding
    B. heat stroke
    C. cardiac arrest
    D. allergic reaction
    C. cardiac arrest
  192. At the scene of a multiple vehicle accident, what should you do first?
    A. Treat the first patient you see
    B. Start triage
    C. Establish incident command
    D. Set up a command post
    C. Establish incident command
  193. A 2 year old is conscious after ingesting an unknown quantity of bleach. What should you suspect?
    A. Bright, red skin on extremities
    B. Chemical burns around the mouth
    C. Hot, dry skin and diarrhea
    D. Dilated pupils and labored breathing
    B. Chemical burns around the mouth
  194. What does insulin increase cell permeability to?
    A. Glucagon
    B. Glucose
    C. Glycosuria
    D. Glycogen
    B. Glucose
  195. The process of moving the responsibility for incident command from one Incident Commander to another is called “transfer of command.” The previous Incident Commander should:
    A. Continue as Co-Commander
    B. Leave to eliminate confusion
    C. Notify headquarters in person
    D. Be given another assignment
    D. Be given another assignment
  196. What condition presents with a fruity, acetone-like odor on the breath?
    A. Hypoglycemia
    B. Hyperglycemia
    C. Hyperosmolar hyperglycemic nonketotic coma (HHNC)
    D. Diabetic ketoacidosis (DKA)
    D. Diabetic ketoacidosis (DKA)
  197. You are ordered to give a medication based on weight. The transfer papers say the patient weighs 155 lbs. How many kilograms does the patient weigh?
    A. 71 kg
    B. 78 kg
    C. 77 kg
    D. 70 kg
    D. 70 kg
  198. What are the structures in the lungs where carbon dioxide and oxygen exchanged called?
    A. Pleura
    B. Bronchi
    C. Lung parenchyma
    D. Alveoli
    D. Alveoli
  199. A 21 year old female was lost in the woods for several hours. The ambient temperature is 25 degrees Fahrenheit. She is lethargic and confused. The skin on her hands is waxy and white. What should you do?
    A. Place several heat packs on her legs and arms.
    B. Gently wrap the patient in several dry blankets.
    C. Apply heat packs on her neck, armpits, and groin.
    D. Vigorously rub the patient’s extremities.
    B. Gently wrap the patient in several dry blankets.
  200. What is the most important landmark for visualizing endotracheal intubation?
    A. Epiglottis
    B. Arytenoid cartilage
    C. Carina
    D. Cricoid cartilage
    B. Arytenoid cartilage
  201. What risk factor may cause the development of an esophageal varice?
    A. Cirrhosis
    B. Gastritis
    C. Anticoagulation therapy
    D. Forceful vomiting
    A. Cirrhosis
  202. A 26 year old male asthmatic complains of difficulty breathing. You administer albuterol, what should you suspect?
    A. Suppression of heart activity and bronchial constriction.
    B. Stimulation of the heart and bronchial constriction.
    C. Stimulation of the heart and bronchial dilation.
    D. Suppression of heart activity and bronchial dilation.
    C. Stimulation of the heart and bronchial dilation
  203. What is an enteral route for medication administration?
    A. Subcutaneous
    B. Sublingual
    C. Intravenous
    D. Topical
    B. Sublingual
  204. Which of the following behaviors may be considered abusive?
    A. Placing a child on “time-out” when they misbehave.
    B. Insisting a high school senior work a weekend job to earn money toward her college education.
    C. Placing a grandparent in nursing care when the burdens of extended care become unmanageable.
    D. Requiring a child to help out with chores around the house to earn their allowance.
    E. Withholding of asthma medication from a child who refuses to finish eating dinner.
    E. Withholding of asthma medication from a child who refuses to finish eating dinner.
  205. When viewing a three channel cardiac monitor, leads I and II display artifact, and lead III shows a clear reading. Based on Einthoven’s Triangle, you should
    A. monitor the patient by paddles.
    B. replace the right arm electrode.
    C. monitor only in lead III.
    D. discontinue cardiac monitoring.
    B. replace the right arm electrode.
  206. Which structure in the kidney forms urine?
    A. Renal tube
    B. Renal medulla
    C. Nephron
    D. Renal cortex
    C. Nephron
  207. A positive deflection on the ECG tracing paper indicates that the electrical impulse is
    A. traveling perpendicular to the lead.
    B. moving toward the negative electrode.
    C. non existent.
    D. moving toward the positive electrode.
    D. moving toward the positive electrode.
  208. What condition results with the presence of an elevated intraocular pressure within the eye that can lead to vision loss?
    A. Strabismus
    B. Nystagmus
    C. Cataracts
    D. Glaucoma
    D. Glaucoma
  209. What are written instructions expressing an individual’s health care wishes in the event that they are incapacitated?
    A. Health care surrogates
    B. Medical practice directives
    C. Health care affidavits
    D. Advance directives
    D. Advance directives
  210. A 25 year old male was found lying on the bottom of a pool. The lifeguard pulled him out and laid him beside the pool before calling EMS. He is unresponsive and not breathing. What should you do?
    A. Stabilize c-spine and assist his ventilations.
    B. Stabilize c-spine and intubate.
    C. Place the patient on his side and suction.
    D. Assist his ventilations with a BVM.
    A. Stabilize c-spine and assist his ventilations.
  211. A 22 year old male hit a tree at a high speed and was thrown 20 feet. He has a rigid abdomen and you determine him to be in hypovolemic shock. Where is he most likely bleeding?
    A. Liver and kidney
    B. Spleen and pancreas
    C. Spleen and stomach
    D. Liver and spleen
    D. Liver and spleen
  212. A 65 year old female complains of heart palpitations and dizziness after her dialysis. An ECG reveals peaked T waves and bigeminal PVCs. What should you suspect?
    A. Hyperkalemia
    B. Hypomagnesia
    C. Bundle branch block
    D. Hypercalcemia
    A. Hyperkalemia
  213. Which of the following mechanisms of injury would likely result in coup-contrecoup trauma?
    A. A low-energy projectile.
    B. A front-impact motor vehicle collision.
    C. A fall from over 20 feet.
    D. Diving into shallow water.
    B. A front-impact motor vehicle collision.
  214. A pregnant female has a grand mal seizure during active labor. What should you administer?
    A. Oxygen
    B. Valium
    C. Sodium bicarbonate
    D. Magnesium sulfate
    D. Magnesium sulfate
  215. A 32 year old pregnant female complains of profuse vaginal bleeding. She denies any abdominal pain or discomfort. What should you suspect?
    A. Ruptured ovarian cyst
    B. Abruptio placenta
    C. Uterine rupture
    D. Placenta previa
    D. Placenta previa
  216. When is it necessary to wear gloves, gown, mask, and protective eyewear?
    A. Assisting with a prehospital childbirth
    B. Oral suctioning on a tuberculosis patient
    C. Administering an IM injection to a MRSA patient
    D. Starting an IV on an AIDS patient
    A. Assisting with a prehospital childbirth
  217. A 22 year old male complains of abdominal and joint pain following a dive. What should you suspect?
    A. Decompression sickness
    B. Pulmonary barotrauma
    C. Pneumomediastinum
    D. Tension pneumothorax
    A. Decompression sickness
  218. A 34 year old female has a contusion above her left breast and left lateral torso after a motor vehicle collision. Vital signs are BP 102/64, P 96, R 18. During transport, her vital signs become BP 90/76, P 118, R 24. Breath sounds are absent on the left side. What should you do?
    A. Begin chest compressions
    B. Administer oxygen therapy
    C. Initiate IV therapy
    D. Decompress the chest
    D. Decompress the chest
  219. An 18 year old female is found conscious and alert but trapped in her car after a single motor vehicle crash. The air bag deployed and she denies any head or neck pain, but she complains of hip and left leg pain. When your patient is finally extricated from the car, her vital signs begin to collapse. After starting a large bore IV of normal saline (NS), you should administer
    A. epinephrine.
    B. morphine sulfate.
    C. lidocaine.
    D. no other medication.
    D. no other medication.
  220. Where are the first electrical impulses stimulating the heart located?
    A. Myocardium
    B. Aorta Valve
    C. Left atrium
    D. Right atrium
    D. Right atrium
  221. What happens in the body in response to a state of hypoglycemia?
    A. Glucagon and insulin production in the pancreas decreases
    B. Insulin causes the pancreas to release glucose stores
    C. Parasympathetic stimulus releases glycogen from the liver
    D. Glucagon stimulates the liver to release glycogen stores
    D. Glucagon stimulates the liver to release glycogen stores
  222. A patient complains of night sweats, weight loss, and shortness of breath. Palpation reveals nodules in his axillary region. He denies any sputum production. What should you suspect?
    A. Pneumonia
    B. Hepatitis
    C. Lymphoma
    D. Tuberculosis
    C. Lymphoma
  223. A 56 year old male complains of diarrhea, nausea, and diffuse pain in his abdomen for the past 48 hours. What should you suspect?
    A. Cholecystis
    B. Gastroenteritis
    C. Diverticulitis
    D. Peptic ulcer
    B. Gastroenteritis
  224. A 52 year old female was the passenger in a car crash in which the airbag deployed. She complains of violent pain in her left shoulder. What is this known as?
    A. Grey-Turner’s sign
    B. Cullen’s sign
    C. Kehr’s sign
    D. Shoulder dystocia
    C. Kehr’s sign
  225. A 4 year old male is choking on a marble. He is coughing and drooling. What should you do?
    A. Perform 5 back blows.
    B. Perform 5 abdominal thrusts.
    C. Coach him to cough.
    D. Visualize and remove the marble.
    C. Coach him to cough.
  226. What happens to the diaphragm during inspiration?
    A. Flattens out
    B. Remains static
    C. Moves up
    D. Inverts
    A. Flattens out
  227. What is the leading cause of maternal death associated with pregnancy?
    A. Hemorrhage
    B. Emboli
    C. Eclampsia
    D. Stroke
    A. Hemorrhage
  228. You are dispatched to 243 Main St for a 52 year old female patient complaining of generalized sickness. You arrive on scene and find the patient sitting in a recliner. The patient complains she has been nauseous, having body aches, and shortness of breath for the past eight hours. After ABCs what is the MOST appropriate intervention/assessment?
    A. Acquire a 12-lead.
    B. Transport the patient to the hospital.
    C. Establish an IV.
    D. Acquire a temperature.
    A. Acquire a 12-lead.
  229. What is a series of two or more generalized motor seizures without an intervening period of consciousness known as?
    A. Hypertonic seizure
    B. A petit mal seizure
    C. Status epilepticus
    D. A complex partial seizure
    C. Status epilepticus
  230. You place your 34 year old patient on the monitor and find a rhythm with these characteristics. QRS width 0.10secs, slurred up-stroke of the PR interval, Rate-168/reg. What type of rhythm do you suspect?
    A. Supraventricular tachycardia with WPW
    B. Ventricular tachycardia
    C. Atrial flutter
    D. Atrial fibrillation
    A. Supraventricular tachycardia with WPW
  231. A 28 year old female has bilateral pelvic pain and yellow vaginal discharge. What should you suspect?
    A. Ruptured ovarian cyst
    B. Pelvic inflammatory disease
    C. Ectopic pregnancy
    D. An acute urinary tract infection
    B. Pelvic inflammatory disease
  232. What buffer is used to maintain acid-base chemistry in the blood?
    A. Water
    B. Bicarbonate
    C. Chloride/sodium
    D. Sodium/potassium
    B. Bicarbonate
  233. A 26 year old male scuba diver reports severe joint pain an hour after ascent. Vitals are: BP 110/68, P 60, R 14, and SpO2 100%. What should you do?
    A. Transport to a hyperbaric facility.
    B. Provide oxygen via a CPAP mask.
    C. Establish an IV to provide pain relief.
    D. Administer oxygen via an NRB.
    D. Administer oxygen via an NRB.
  234. What specialized bands of tissue are inserted between myocardial cells that increase the rate of cellular conduction?
    A. Purkinje system
    B. Intercalated discs
    C. Atrial syncytium
    D. Internodal paths
    B. Intercalated discs
  235. A 45 year old female with a 10 year cardiac history complains of severe dyspnea. She takes furosemide daily and has frothy sputum. Lung sounds reveal bilateral crackles. Vital signs are BP 90/70, P 56, R 32, SpO2 93%. What should you do?
    A. Begin rapid sequence intubation (RSI).
    B. Intubate and assist ventilations via bag-valve mask.
    C. Assist her respirations and administer nitroglycerin.
    D. Initiate CPAP and administer furosemide.
    C. Assist her respirations and administer nitroglycerin.
  236. What is the generic name for Dilaudid?
    A. hydrochlorothiazide
    B. hydromorphone
    C. codeine
    D. hydrocodone
    B. hydromorphone
  237. What is the responsibility of the Safety Officer?
    A. Collects hazard and risk data for documentation
    B. Supports operations
    C. Monitors the scene for potential hazards
    D. Alerts media regarding hazards to the community
    C. Monitors the scene for potential hazards
  238. How is cardiac output calculated?
    A. Heart rate multiplied by stroke volume.
    B. Heart rate divided by stroke volume.
    C. Heart rate added to stroke volume.
    D. Stroke volume divided by heart rate.
    A. Heart rate multiplied by stroke volume.
  239. What condition is NOT commonly associated with sickle cell anemia?
    A. Excessive bleeding
    B. Renal disease
    C. Musculoskeletal pain
    D. Abdominal pain
    A. Excessive bleeding
  240. What is a form of off line or indirect medical control signed by the medical director?
    A. Medical direction
    B. Advanced directive
    C. Protocol
    D. Standing order
    D. Standing order
  241. You find a critically injured female in single vehicle automobile collision on a rural road. What additional resource should you mobilize?
    A. Local sheriff
    B. A second ambulance
    C. A rescue squad
    D. Air medical transport
    D. Air medical transport
  242. An alert 65 year old male speaks in 2 word dyspnea. He has a 10 year cardiac history. Lung sounds reveal crackles in the bases with expiratory wheezing. Vital signs are BP 140/90, P 110, R 30. What should you do?
    A. Administer nitroglycerin
    B. Obtain IV access
    C. Acquire a 12 lead ECG
    D. Administer aspirin
    D. Administer aspirin
  243. After several attempts at intubating a patient you are unsuccessful. What should you do?
    A. Insert an oral airway and ventilate using a bag-valve mask.
    B. Continue your attempts until you are too exhausted to continue.
    C. Provide high flow oxygen with a non-rebreather mask.
    D. Call medical control and request permission to stop the code.
    A. Insert an oral airway and ventilate using a bag-valve mask.
  244. An 8 year old complains of severe lower leg pain. He was released from a local emergency room 8 hours ago following treatment for a closed tibia fracture. The whole lower leg is swollen, pale and has decreased sensation. What should you suspect?
    A. An acute arterial occlusion
    B. Compartment syndrome
    C. Ineffective immobilization
    D. An associated calf muscle strain
    B. Compartment syndrome
  245. What is the most common cause of abdominal trauma?
    A. Gun shot wounds
    B. Multiple stab wounds
    C. Blunt trauma
    D. Motor vehicle crashes
    C. Blunt trauma
  246. Which sound indicates an upper airway obstruction?
    A. Rales
    B. Wheezing
    C. Crackles
    D. Stridor
    E. Rhonchi
    D. Stridor
  247. Where should you position your vehicle at a motor vehicle collision on the highway?
    A. 500 feet behind the patient’s vehicle
    B. At a location determined by law enforcement
    C. At a staging area
    D. In front of the patient’s vehicle
    D. In front of the patient’s vehicle
  248. To administer dextrose, what must a patient’s blood sugar be under?
    A. 40 mg/dL
    B. 100 mg/dL
    C. 80 mg/dL
    D. 60 mg/dL
    D. 60 mg/dL
  249. You arrive at the scene of a motor vehicle collision. There is no fire, leaking fluid, or fumes. How far from the wreckage should you park?
    A. 15 feet
    B. 150 feet
    C. 50 feet
    D. 100 feet
    C. 50 feet
  250. Oral hypoglycemic agents primarily work by what mechanism?
    A. Stimulate the creation of insulin
    B. Stimulate the release of insulin
    C. Lower sugar production
    D. Convert glycogen to glucose
    B. Stimulate the release of insulin
  251. What pacemaker in the heart has an intrinsic rate of 60-100 beats per minute?
    A. SA Node
    B. Bundle of His
    C. AV Node
    D. Ductus arteriosus
    A. SA Node
  252. What is the purpose of the National Incident Management System?
    A. Provide plans for covering up terrorist attacks.
    B. Allow for federal control of all major incidents.
    C. Standardize the multi-agency response to an MCI.
    D. Provide funding to local agencies that deal with MCI’s.
    C. Standardize the multi-agency response to an MCI.
  253. What solution has a solute concentration lower than cells?
    A. Hypotonic
    B. Isotonic
    C. Eutonic
    D. Hypertonic
    A. Hypotonic
  254. A 67 year old female is undergoing dialysis treatment for renal failure. Which of her organs is compromised?
    A. Lungs
    B. Pancreas
    C. Liver
    D. Kidneys
    D. Kidneys
  255. When assessing a patient with a fractured tibia, what is an example of a pertinent negative?
    A. The patient a loss of circulation below the injury.
    B. The patient is unable to wiggle his/her toes.
    C. The patient has no pedal pulse.
    D. The patient denies numbness or tingling.
    D. The patient denies numbness or tingling.
  256. What is the best position to optimally hear heart sounds?
    A. Left lateral recumbent
    B. Sitting up leaning forward
    C. Rght lateral recumbent
    D. High Fowler’s leaning back
    B. Sitting up leaning forward
  257. A 62 year old female presents with diminished lung sounds in all lobes, SpO2 of 86% on NRB @15 lpm, HR 120 and BP of 190/100. She reports a history of HTN, CHF, COPD and A-Fib. The patient is prescribed Metoprolol, Diltiazem, Nitroglycerin, Albuterol and Flovent. She should receive which intervention first?
    A. Nitroglycerin
    B. Aspirin
    C. Albuterol
    D. CPAP
    A. Nitroglycerin
  258. What part of the brain is responsible for balance and coordination?
    A. Cerebellum
    B. Cerebrum
    C. Occipital lobe
    D. Hypothalamus
    A. Cerebellum
  259. A patient is trapped in a car with side to side rocking. What should you do?
    A. Extricate the patient
    B. Remove the steering wheel
    C. Stabilize the car
    D. Remove the roof
    C. Stabilize the car
  260. A patient has excessive bleeding from the neck. What should you apply?
    A. A band-aid
    B. A tourniquet
    C. An occlusive dressing
    D. A pressure dressing
    D. A pressure dressing
  261. A 59 year old female missed her weekly renal dialysis appointment. She is feeling weak and unable to care for herself. What should you suspect?
    A. Anemia
    B. Polyuria
    C. Fluid overload
    D. Hypokalemia
    C. Fluid overload
  262. A 66 year old male complains of nausea and vomiting for the past day. He has been unable to eat any food as it causes his symptoms to worsen. He has pain that radiates from his stomach to his back. What should you suspect?
    A. Irritable bowel syndrome
    B. Gastroenteritis
    C. Actute MI
    D. Pancreatitis
    D. Pancreatitis
  263. A 32 year old female is unconscious after being struck in the chest with a bat. Vital signs are BP 90/60, P 34, R 8. Her skin is cool, diaphoretic, and cyanotic. Breath sounds are absent and her trachea is deviated to the left. What should you suspect?
    A. Hemothorax
    B. Pneumothorax
    C. Flail chest
    D. Tension pneumothorax
    A. Hemothorax
  264. It is 0300 and you respond to a person having a headache. There is no traffic on the road at this time. How should you respond?
    A. With lights and sirens.
    B. Expeditiously without lights and sirens.
    C. With lights only.
    D. Routine, without lights and sirens.
    A. With lights and sirens.
  265. A 58 year old male complains of fever, nausea and blood tinged sputum for the past 3 weeks. He has bilateral rhonchi in all fields and says that it is difficult to catch his breath because he has been coughing so much. What should you do?
    A. Cover his face with a surgical mask
    B. Obtain a baseline set of vitals
    C. Administer a diuretic and apply oxygen
    D. Apply oxygen and start an IV
    A. Cover his face with a surgical mask
  266. A 41 year old male has been injured in an explosion at a chemical laboratory. He has a ruptured tympanic membranes and sinus injuries. At what phase of the blast did the patient sustain the sinus injuries?
    A. Ultimate
    B. Primary
    C. Tertiary
    D. Secondary
    B. Primary
  267. An alert 87 year old woman has peripheral cyanosis and complains of dyspnea. Vitals signs are BP 150/70, P 110, R 26. Suddenly she passes out on her sofa. What should you do?
    A. Insert a nasal airway and apply high flow oxygen.
    B. Check her airway for a FBAO.
    C. Position her airway using a modified jaw thrust maneuver.
    D. Use a head-tilt/chin-lift and assist her ventilations.
    D. Use a head-tilt/chin-lift and assist her ventilations.
  268. What neurotransmitter is found in the synapse between the alpha or beta receptors and the postganglionic neuron?
    A. Norepinephrine
    B. Dopamine
    C. Adrenaline
    D. Acetylcholine
    A. Norepinephrine
  269. A 38 year old male has been struck with a brick. His words are incomprehensible, and he has no eye opening or motor response. He has unequal pupils and has blood draining from his left ear. Vital signs are BP 178/110, P 58, R 8. What should you suspect?
    A. A narcotic overdose
    B. Subdural hematoma
    C. A basal skull fracture
    D. Hypovolemic shock
    C. A basal skull fracture
  270. Which of the following patients should have the pneumatic anti-shock garment applied and inflated?
    A. 22 year old female gunshot wound to the flank, BP 84/44
    B. 35 year old female blunt abdominal trauma, BP 180/90.
    C. 25 year old male head injury, apneic and tachycardia, BP 86/30
    D. 16 year old male bilateral tibia and fibula fractures, BP 90/54
    D. 16 year old male bilateral tibia and fibula fractures, BP 90/54
  271. What mechanism does the body lose the majority of heat?
    A. Evaporation
    B. Conduction
    C. Convection
    D. Radiation
    D. Radiation
  272. What is the expected breathing rate for a child under the age of eight?
    A. 10-20 breaths per min.
    B. 5-10 breaths per min.
    C. 20-40 breaths per min.
    D. 40-60 breaths per min.
    C. 20-40 breaths per min.
  273. What level should EMS Professionals responding to high risk low incident rescues be trained?
    A. Operations
    B. Specialist
    C. Awareness
    D. Technical
    C. Awareness
  274. What is an abnormal respiratory pattern characterized by deep, rapid breaths?
    A. Kussmaul’s
    B. Cheyne-Stokes
    C. Ataxic (Biot’s)
    D. Apneustic
    A. Kussmaul’s
  275. What is your priority as the incident commander at a mass casualty incident?
    A. Monitor incident organization.
    B. Notification of the media.
    C. Directing extrication of casualties.
    D. Ensuring safety of EMS personnel.
    A. Monitor incident organization.
  276. A 31 year old female has gun shot wound to the right lateral chest. She is awake and alert with obvious difficulty breathing. Where should she be transported?
    A. Respiratory center
    B. Trauma center
    C. Local hospital
    D. Emergency department
    B. Trauma center
  277. What does chronotropy refer to?
    A. Stroke volume
    B. Blood pressure
    C. Cardiac output
    D. Heart rate
    D. Heart rate
  278. A healthy, athletic male undergoes a routine physical screening. He presents in no acute distress. His ECG shows the following rhythm: “https://testing.fisdap.net/testing_images/EKGParis1.png” (Sinus Brady at 60 BPM) You should:
    A. initiate IV access and administer atropine 0.5 mg.
    B. not do anything; this is likely normal for this patient.
    C. prepare for immediate transcutaneous pacing.
    D. administer a fluid bolus of 250 cc and reevaluate.
    B. not do anything; this is likely normal for this patient.
  279. What is a psychosomatic complaint?
    A. An often delusional mental state characterized by feelings of dejection, insomnia, or weight loss.
    B. A condition in which psychic energy from a repressed idea is converted into physical symptoms.
    C. A feeling of apprehension, uncertainty, and fear.
    D. A mental disorder characterized by abnormal suspicions or other delusions.
    B. A condition in which psychic energy from a repressed idea is converted into physical symptoms.
  280. Which statement about suicide is correct?
    A. People who talk about suicide rarely act on it.
    B. There is a high correlation between suicide and alcohol use.
    C. All suicidal people are mentally ill.
    D. The suicide rate is lowest during holiday seasons.
    B. There is a high correlation between suicide and alcohol use.
  281. You respond to a call when you see a stopped school bus with its red lights flashing. What should you do?
    A. Select an alternate route that is both safe and quick.
    B. Slow down and be alert for any children who may be crossing.
    C. Call dispatch and request the response of another ambulance.
    D. Stop and proceed only after the flashing lights are off.
    D. Stop and proceed only after the flashing lights are off.
  282. Why is it beneficial to have two rescuers dedicated solely to ventilating a patient via Bag Valve Mask (BVM)?
    A. So one rescuer can squeeze the bag and the other rescuer can provide cricoid pressure
    B. So they can switch every two minutes to avoid rescuer fatigue
    C. So one can ventilate and the other can insert a nasopharyngeal airway (NPA)
    D. So one rescuer can squeeze the bag and the other rescuer can maintain an adequate seal
    D. So one rescuer can squeeze the bag and the other rescuer can maintain an adequate seal
  283. A 56 year old male complains of dull and cramping abdominal pain that makes him nauseous. What type of pain is describing?
    A. Parietal
    B. Somatic
    C. Referred
    D. Visceral
    D. Visceral
  284. Hepatojugular reflux is an indication of what condition.
    A. Pancreatitis
    B. Bowel Obstruction
    C. Liver Failure
    D. Heart Failure
    D. Heart Failure
  285. A 26 year old male is leaning over the toilet vomiting a steady stream of bright, red blood. He is a chronic alcoholic. What should you do?
    A. Transport in a position of comfort.
    B. Position him supine and suction his airway.
    C. Immobilize the patient and transport.
    D. Apply a non-rebreather mask.
    A. Transport in a position of comfort.
  286. You are treating a patient exhibiting signs and symptoms hypoglycemia. What hormones would you expect are being released in their body?
    A. Glucagon, epinephrine, cortisol
    B. Glucagon, ADH, oxytocin
    C. Cortisol, thyroxine, norepinephrine
    D. Insulin, epinephrine, ADH
    A. Glucagon, epinephrine, cortisol
  287. What is the degree of skill and judgment that would be expected of a similarly trained, reasonable paramedic acting under similar circumstances is referred to as?
    A. Standard of care
    B. Professional standard
    C. Paramedic code
    D. Scope of practice
    A. Standard of care
  288. What is the natural tendency of the body to maintain a steady and normal internal environment known as?
    A. Constant metabolism
    B. Positive feedback
    C. Homeostasis
    D. Osmosis
    C. Homeostasis
  289. What blood type is known as the “universal recipient”?
    A. AB
    B. B
    C. A
    D. O
    A. AB
  290. A 44 year old male complains of a headache. He states that there are too many people talking to him and asking him questions. He is agitated and confused. He states that “his friend,” Martin is going to hurt you. You and your partner are alone. What should you do?
    A. Tell him to walk to the ambulance with you.
    B. Determine if he wants to hurt himself or you.
    C. Ask him why Martin is going to hurt you.
    D. Forcefully tell him to sit down on his hands.
    B. Determine if he wants to hurt himself or you.
  291. A 34 year old male was involved in a car accident. He has significant facial trauma and is bleeding profusely from his mouth and nose. When performing your assessment, what findings are most indicative of neurological disorders?
    A. Changes in level of consciousness
    B. Tachycardia and hypotension
    C. Narrowing pulse pressure
    D. Head trauma
    A. Changes in level of consciousness
  292. A carrier-mediated process that can move substances against a concentration gradient is defined as.
    A. Increased capillary permeability.
    B. Facilitated diffusion.
    C. Active transport.
    D. Diffusion.
    C. Active transport.
  293. A 24 year old female was burned in a fire. She has third degree burns to her anterior chest, both her arms and hands, and burns to her face and head. What percentage of her body is burned?
    A. 27%
    B. 45%
    C. 18%
    D. 36%
    D. 36%
  294. How should EMS units respond to violent situations to ensure safety?
    A. Remain away from the scene and have law enforcement bring the patient to you.
    B. Enter as quickly as possible because every patient needs prompt treatment.
    C. Enter the scene before law enforcement to avoid any potential confrontation.
    D. Remain away from the scene until law enforcement arrives and secures the scene.
    D. Remain away from the scene until law enforcement arrives and secures the scene
  295. A 46 year old man is responsive to verbal stimuli after being stabbed. He has a bubbling wound to the left chest. His skin is cyanotic and diaphoretic. Vital signs are BP 108/76, P 120 and weak, R 10 and shallow. What should you suspect?
    A. Simple pneumothorax
    B. Tension pneumothorax
    C. Hemothorax
    D. Open pneumothorax
    D. Open pneumothorax
  296. How should you alter your intubation technique for a 2 year old child?
    A. Do not hyperextend the neck as you visualize the vocal cords.
    B. Do not pre-oxygenate prior to beginning your intubation
    C. Insert the ET tube from the right side of the mouth.
    D. Insert the blade in the right side of the mouth instead of the left.
    A. Do not hyperextend the neck as you visualize the vocal cords.
  297. What is the amount of blood ejected by the heart during one cardiac contraction known as?
    A. Stroke volume
    B. Cardiac cycle
    C. Ejection fraction
    D. Diastolic volume
    A. Stroke volume
  298. A 22 year old female complained of a headache and collapsed suddenly. She has pinpoint pupils and withdraws from painful stimulus. She vomited once prior to your arrival. Vital signs are BP 190/126, P 60, R 36. What should you suspect?
    A. Hypoglycemia
    B. Grand mal seizure
    C. Hemorrhagic stroke
    D. Narcotic overdose
    C. Hemorrhagic stroke
  299. An 11 year old male has been sick for two weeks and now has a distinct “whoop” sound after he coughs. What is the most likely cause?
    A. Bronchitis
    B. Meningitis
    C. Pertussis
    D. Tuberculosis
    C. Pertussis
  300. A 24 year old male is lying supine and is unresponsive. You have opened his airway with a head-tilt/chin-lift maneuver and determined that he is apneic with a bradycardic pulse. You attempt to ventilate, but are unsuccessful. What should you do?
    A. Begin continuous chest compressions.
    B. Check for a carotid pulse.
    C. Reposition the airway and give 2 ventilations.
    D. Attempt to give 2 more forceful ventilations.
    C. Reposition the airway and give 2 ventilations.
  301. Your ALS unit responds to a 55 year-old male complaining of chest pain and shortness of breath. Upon assessment, you find the patient has a rapid and regular narrow complex rhythm at a rate of 188 beats per minute. What is the correct treatment for this patient.
    A. Adenosine 6 mg IV Push
    B. Synchronized Cardioversion at 50-100 Joules
    C. Valsalva Maneuver
    D. Defibrillate at 100 Joules
    B. Synchronized Cardioversion at 50-100 Joules
  302. How should you treat shock resulting from a tension pneumothorax?
    A. Pleural decompression
    B. Administration of a fluid challenge
    C. Cricothyrotomy
    D. Endotracheal intubation and hyperventilation
    A. Pleural decompression
  303. You respond for an unconscious 5 year old. The patient’s skin is dusky, he responds to deep painful stimuli, and his breathing is slow and shallow. After initiating ventilation with a bag-valve mask, his mentation and respiratory effort do not change. His pulse is 46. What should you do?
    A. Initiate transport.
    B. Attempt to place an oral airway.
    C. Initiate chest compressions.
    D. Increase the rate of ventilation.
    C. Initiate chest compressions.
  304. Which symptoms are collectively known as Cushing’s triad?
    A. Decreased blood pressure, decreased heart rate, increased respirations
    B. Increased blood pressure, decreased heart rate, altered respirations
    C. Increased blood pressure, increased heart rate, decreased respirations
    D. Decreased blood pressure, increased heart rate, altered respirations
    B. Increased blood pressure, decreased heart rate, altered respirations
  305. What is the inherent pacing rate of the AV junction?
    A. 60-100
    B. 10-20
    C. 20-40
    D. 40-60
    D. 40-60
  306. A 32 year old pregnant female states “I have been in labor for an hour and my water broke.” What should you do?
    A. Transport emergently.
    B. Check for crowning.
    C. Obtain baseline vital signs.
    D. Obtain a SAMPLE medical history
    B. Check for crowning.
  307. What is the medical term for excessive urine production?
    A. Oliguria
    B. Hyperuria
    C. Polyuria
    D. Nocturia
    C. Polyuria
  308. What viral infection commonly found in children ages 6 months to 4 years has a distinct “seal-like” cough?
    A. Epiglottitis
    B. Croup
    C. Asthma
    D. Bronchiolitis
    B. Croup
  309. If transport is delayed or prolonged, and if circulation is impaired, an attempt should be made to reposition a grossly deformed fracture or dislocated joint (if allowed by medical control or protocol). The exception is an injury to the _
    A. Hip
    B. Elbow
    C. Knee
    D. Ankle
    A. Hip
  310. Which of the following manual airway maneuvers should be used to open the airway when caring for a patient with a suspected cervical spine injury?
    A. Sellick’s
    B. Jaw/tongue lift
    C. Head-tilt/chin lift
    D. Modified jaw-thrust
    D. Modified jaw-thrust
  311. What refractory phase happens when the ventricles depolarize?
    A. Common
    B. Absolute
    C. Relative
    D. Negative
    B. Absolute
  312. A 33 year old male is unconscious after attempting to hang himself. First responders have placed him supine on the ground. Vital signs are BP 70/30, P 46, R 4 and shallow. What should you do?
    A. Apply a cervical collar.
    B. Insert an oral airway.
    C. Assist his ventilations.
    D. Start CPR.
    C. Assist his ventilations.
  313. An unresponsive 2 day old female is in respiratory distress. Her skin is cyanotic and vital signs are P 184, R 72. What should you do?
    A. Asses for a blood pressure.
    B. Administer albuterol.
    C. Assist her ventilations.
    D. Apply blow-by oxygen.
    C. Assist her ventilations.
  314. An injury produced by wounding forces of compression and change of speed, both of which can disrupt tissue is defined as.
    A. Blunt trauma.
    B. Penetrating trauma.
    C. Cavitation.
    D. Third degree Trauma.
    A. Blunt trauma.
  315. A 40 year old male is short of breath after being kicked in the chest by a horse. He is alert and disoriented with absent lung sounds on right side. Vital signs are P 120, R 24 and shallow. What finding would indicate an immediate needle thoracostomy?
    A. Jugular vein distension
    B. Muffled heart tones
    C. Blood pressure of 80/40
    D. Subcutaneous emphysema
    C. Blood pressure of 80/40
  316. Pertinent negatives are a part of the narrative component of the patient care report because they
    A. determine a treatment plan for the patient.
    B. confirm a thorough patient assessment.
    C. provide the field diagnosis.
    D. provide the most important information about the patient’s complaint.
    B. confirm a thorough patient assessment.
  317. A 16 year old male has a broken nose. He is bleeding heavily from both nostrils. What should you do?
    A. Apply direct pressure to his nostrils.
    B. Gently pack both nostrils with sterile gauze.
    C. Pinch his nose and lean him forward.
    D. Pinch his nose and lean him back.
    C. Pinch his nose and lean him forward.
  318. A 19 year old male has a traumatic chest injury. You discover that his trachea deviates away from the injury. What should you suspect?
    A. Tension pneumothorax
    B. Atelectasis
    C. Upper airway obstruction
    D. Flail chest
    A. Tension pneumothorax
  319. Stroke volume is a function of all of the following EXEPT:
    A. Cardiac Contractility
    B. Heart Rate
    C. Afterload
    D. Preload
    B. Heart Rate
  320. What ECG tracing is associated with atrial systole?
    A. T-waves
    B. S-waves
    C. R-waves
    D. P-waves
    D. P-waves
  321. How does endogenous fibrinolysis occur?
    A. Through administration of heparin.
    B. By the production of plasmin.
    C. Through conversion of plasmin to plasminogen.
    D. By administration of tissue plasminogen.
    B. By the production of plasmin.
  322. What should treatment for cardiogenic shock with associated pulmonary edema include?
    A. Rapid fluid replacement with a crystalloid solution.
    B. Placing the patient in the Trendelenburg position.
    C. The application and inflation of PASG.
    D. Elevating the patient’s head and shoulders.
    D. Elevating the patient’s head and shoulders.
  323. A 24 year old male was struck in the head, his eyes can only follow your finger upwards. What type of fracture should you suspect?
    A. Basilar skull
    B. LeFort II
    C. Nasal bone
    D. Orbital
    D. Orbital
  324. A 40 year old male is unresponsive and pale, and you notice a deformity above his right wrist and bruising around his navel. His wife tells you that her husband was out drinking the previous evening and he stumbled while walking up the front steps. What should you suspect?
    A. Alcohol poisoning
    B. Hypoglycemia
    C. An extremity fracture
    D. Internal bleeding
    D. Internal bleeding
  325. A 34 year old male was hit in the chest with a baseball bat. Portions of his chest move in the opposite directions when he breathes. What should you suspect?
    A. Pulmonary contusion
    B. Pneumothorax
    C. Rib fracture
    D. Flail chest
    D. Flail chest
  326. You arrive at motor vehicle collision involving two cars and four patients. There is no entrapment and the scene is safe. Which command system should be utilized?
    A. Unified command
    B. Singular command
    C. NIMS command
    D. START command
    B. Singular command
  327. A 62 year old male complains of malocclusion and numbness to the chin. You also suspect a nasal fracture with significant facial bleeding and bruising. How should you secure his airway?
    A. Positioning and suctioning
    B. A nasopharyngeal airway
    C. Digital intubation
    D. A nasotracheal ET tube
    A. Positioning and suctioning
  328. What is the correct technique when splinting a suspected finger fracture?
    A. Immobilize the hand and finger in a pillow and apply ice.
    B. Tape the fractured finger to a neighboring finger only.
    C. Place an arm board under the wrist and wrap with curlex.
    D. Apply a straight, flat, and rigid splint to the finger.
    D. Apply a straight, flat, and rigid splint to the finger.
  329. During confirmation of tube placement, you auscultate no sounds over epigastrium, breath sounds present on right side of chest but absent sounds on the left. Where do you suspect your tube is located?
    A. Right mainstem bronchus
    B. Esophagus
    C. Left mainstem bronchus
    D. Stomach
    A. Right mainstem bronchus
  330. What condition is defined by sagging muscles beneath the eye and cheek and asymmetrical movement of the mouth on one side of the face?
    A. Ipsilateral paralysis
    B. Bell’s palsy
    C. Facial droop
    D. Hemi-paralysis
    B. Bell’s palsy
  331. When ventilating a patient with a BVM, how do you know you are delivering effective ventilation?
    A. Observe good chest rise and fall
    B. No resistance when squeezing the BVM
    C. Pulse ox reading increased to 99%
    D. Improved color in the patient’s skin
    A. Observe good chest rise and fall
  332. What relaxes bronchial smooth muscles?
    A. Beta agonists
    B. Alpha agonists
    C. Alpha antagonists
    D. Beta antagonists
    A. Beta agonists
  333. What classification is given to drugs that attach to a receptor site and block the expected response?
    A. Agonists
    B. Antagonists
    C. Partial agonists
    D. Mimetics
    B. Antagonists
  334. The spinal cord runs from the base of the brain down through the spine. The spinal cord ends at about vertebra _. Below that area, a collection of nerve roots continues, looking somewhat like a horse’s tail (cauda equina).
    A. T10
    B. C7
    C. L2
    D. T7
    C. L2
  335. What gland secretes chemical substances directly into the blood and is also called a ductless gland?
    A. Adrenocorticotropic
    B. Endocrine
    C. Exocrine
    D. Sebaceous
    B. Endocrine
  336. After assisting your patient with the self-administration of their prescribed metered-dose inhaler medication, what should you do?
    A. Gather an additional set of vital signs.
    B. Discontinue high-flow oxygenation of the patient.
    C. Ensure the medication belongs to the patient.
    D. Wait for the patient’s condition to alleviate.
    A. Gather an additional set of vital signs.
  337. A 72 year old male has slurred speech and left-sided weakness. Which diagnostic assessment is most important to narrow your field impression?
    A. End-tidal CO2 analysis
    B. 12-lead electrocardiogram
    C. Blood glucose analysis
    D. Oxygen saturation
    C. Blood glucose analysis
  338. An 18 year old male is hypotensive and bradycardic. His skin is warm and dry. He was hit during a football game and lost consciousness. What should you suspect?
    A. Septic
    B. Cardiogenic
    C. Hypovolemic
    D. Neurogenic
    D. Neurogenic
  339. What is normal pulmonary artery pressure is equal to?
    A. Right ventricular diastolic pressure
    B. Right ventricular systolic pressure
    C. Left ventricular systolic pressure
    D. Left ventricular diastolic pressure
    B. Right ventricular systolic pressure
  340. You are giving 3 mg of midazolam. Your concentration is 5 mg/5 mL. How many mL should you give?
    A. 7
    B. 1
    C. 5
    D. 3
    D. 3
  341. A 38 year old female has severe dyspnea and a swollen, protruding tongue. She recently left her physician’s office after receiving an injection of antibiotics. What should you do?
    A. Perform needle cricothyrotomy.
    B. Insert a nasal airway.
    C. Administer epinephrine.
    D. Administer warm humidified oxygen.
    C. Administer epinephrine.
  342. A lethargic 78 year old female has chest pain. She is dyspneic and nauseated. An ECG shows widened polymorphic complex and a ventricular rate of 130. What should you do?
    A. Administer amiodarone
    B. Start chest compressions
    C. Synchronized cardioversion
    D. Administer lidocaine
    C. Synchronized cardioversion
  343. In order for a vaccine to be effective, it must cause:
    A. a minor illness to the person vaccinated
    B. development of antibodies specific to the disease
    C. the body to store the antigen information
    D. pain during administration at the site of injection
    B. development of antibodies specific to the disease
  344. A 34 year old female with a history of hypothyroidism is unresponsive. A physical exam reveals thin hair, facial swelling, and very cold, pliable skin. Vital signs are BP 90/62, P 56, R 10, temperature 95F. What should you suspect?
    A. Addison’s disease
    B. Thyrotoxicosis
    C. Myxedema
    D. Hypothermia
    C. Myxedema
  345. A 72 year old male complains of sudden onset chest pain in his midclavicular, left, 5th intercostal space. Vital signs are BP 140/80, P 102, R 26. He states he recently had a hernia surgery. What should you suspect?
    A. Pulmonary embolism
    B. Myocardial infarction
    C. Pneumothorax
    D. Acute respiratory distress syndrome
    A. Pulmonary embolism
  346. What is the normal bodily pH?
    A. 7.10-7.25
    B. 7.35-7.45
    C. 6.25-6.45
    D. 7.55-8.0
    B. 7.35-7.45
  347. Which of the following has the thickest muscle wall?
    A. Ventricular myocardium
    B. Visceral pericardium
    C. Atrial epicardium
    D. Parietal endocardium
    A. Ventricular myocardium
  348. What signals the end of the second stage of labor?
    A. Delivery of the baby
    B. Complete cervical dilation
    C. Delivery of the placenta
    D. Bulging of the perineum
    A. Delivery of the baby
  349. What disease has the greatest impact on alveolar oxygen diffusion?
    A. Bronchitis
    B. Pneumonia
    C. Emphysema
    D. Asthma
    C. Emphysema
  350. What is the responsibility of Finance during an MCI?
    A. Tracking personnel staffing for overtime records
    B. Coordinating multiple agency’s efforts in mitigating the incident
    C. Ordering ambulance strike teams to staging areas
    D. Securing portable showers and toilets for the rescue personnel
    A. Tracking personnel staffing for overtime records
  351. You are treating a 65 year old male with a history of CHF in acute respiratory distress. He complains of difficulty breathing and chest pain. You note crackles bilaterally, pulse 100, respiratory rate of 22, and BP 160/90. You should:
    A. Administer 0.4 mg of nitroglycerin.
    B. Have him chew 162 mg of aspirin.
    C. Place the patient on CPAP.
    D. Prepare for an advanced airway.
    C. Place the patient on CPAP.
  352. What is the most common route of hazardous materials exposure?
    A. Absorption
    B. Injection
    C. Inhalation
    D. Ingestion
    C. Inhalation
  353. What type of impact to an unrestrained occupant commonly results in a multiple system injury?
    A. Rotational
    B. Frontal
    C. Lateral
    D. Broadside
    A. Rotational
  354. You are called to a bar for a 52 year old male who has slurred speech and difficulty sitting upright. He is unable to follow your commands and refuses to leave the bar. Which of the following should you consider LAST?
    A. Stroke
    B. Diabetic reaction
    C. Seizure
    D. Alcohol intoxication
    D. Alcohol intoxication
  355. You suspect a tension pneumothorax in a 26 year old male. How should you perform needle decompression?
    A. 20-22 gauge catheter above the rib in the midclavicular, 3rd intercostal space.
    B. 14-16 gauge catheter below the rib in the midclavicular, 3rd intercostal space.
    C. 20-22 gauge catheter below the rib in the midclavicular, 2nd intercostal space.
    D. 14-16 gauge catheter above the rib in the midclavicular, 2nd intercostal space.
    D. 14-16 gauge catheter above the rib in the midclavicular, 2nd intercostal space.
  356. A 5 year old male swallowed three of his grandmother’s digoxin tablets several hours ago. He is alert and oriented. His vitals are BP 90/40, P 48 R 30. The cardiac monitor shows a sinus bradycardia with a first degree block. What should you do?
    A. 0.2 mg/kg atropine
    B. 20 mg/kg calcium chloride
    C. Start transcutaneous pacing
    D. 0.01 mg/kg epinephrine
    C. Start transcutaneous pacing
  357. Once common practice, this is now considered inappropriate because it is cause of bradycardia in neonates.
    A. ventilation with a BVM
    B. bulb suctioning
    C. delayed cord clamping
    D. flicking of the feet soles
    B. bulb suctioning
  358. _ syndrome usually results from compressive forces or blunt trauma to muscle groups confined in tight fibrous sheaths with a minimal ability to stretch.
    A. Traumatic stress
    B. Shaken baby
    C. Compartment
    D. Post-concussion
    C. Compartment
  359. What sexually transmitted disease are cold sores are a form of?
    A. Chlamydia
    B. Herpes
    C. Gonorrhea
    D. Syphilis
    B. Herpes
  360. A 37 year old male has been pulled from the water after diving into a shallow lake. He is unconscious, flaccid, and has diaphragmatic breathing. Vital signs are BP 100/70, P 68, R 20. What should you do?
    A. Apply high-flow oxygen
    B. Manually stabilize the cervical spine
    C. Activate air-medical transport
    D. Insert a blind airway device
    B. Manually stabilize the cervical spine
  361. A 22 year old male working at a factory has recently inhaled a poison. After ensuring scene safety, what should you do?
    A. Insert and oral airway.
    B. Administer high-flow oxygen.
    C. Move the patient to an area outside.
    D. Obtain a set of vital signs.
    C. Move the patient to an area outside.
  362. A 24 year old male complains of shortness of breath. He has a dry cough and burning feeling under his sternum. His symptoms started after he finished diving two hours ago. Vital signs are BP 120/70, P 90 and regular, R 18. What should you suspect?
    A. An air embolism
    B. Pneumothorax
    C. Decompression sickness
    D. An acute cerebrovascular accident
    C. Decompression sickness
  363. What is the classification for atropine?
    A. Parasympatholytic
    B. Sympathomimetic
    C. Vasopressor
    D. Antidysrhythmic
    A. Parasympatholytic
  364. A 78 year old male complains of 10/10 chest pain with palpitations. He is short of breath. His skin is pale, cool and clammy. An ECG shows an narrow complex at a rate of 200. What should you do?
    A. Perform a precordial thump
    B. Start chest compressions
    C. Administer amiodarone
    D. Administer adenosine
    D. Administer adenosine
  365. Which neurotransmitter is found in the preganglionic sympathetic nervous system and the entire parasympathetic nervous system?
    A. Serotonin
    B. Norepinephrine.
    C. Acetylcholine
    D. Gamma-aminobutyric acid
    C. Acetylcholine
  366. A 24 year old male complains of abdominal pain, decreased appetite, weight loss, dark urine, and clay-colored stools. His upper-right quadrant is tender and rigid on palpation. What should you suspect?
    A. Pancreatitis
    B. Hepatitis
    C. Gastritis
    D. Appendicitis
    B. Hepatitis
  367. What chemical substance, released by a gland, controls processes in other glands or body systems?
    A. Hormone
    B. Steroid
    C. Catecholamine
    D. Glucocorticoid
    A. Hormone
  368. A factory worker has a 3/4 inch laceration on his forehead that bleeds through the dressings despite direct pressure. What should you do?
    A. Remove the dressing and apply direct digital pressure.
    B. Continue direct pressure and apply additional dressings.
    C. Apply pressure to the facial artery.
    D. Apply unilateral carotid pressure.
    B. Continue direct pressure and apply additional dressings.
  369. How should you place a pregnant female on a backboard?
    A. Raise the feet 12-18 inches in the air.
    B. Raise the head 12-18 inches in the air.
    C. You should never put them on the backboard.
    D. Raise the right side of the board 6-12 inches in the air.
    D. Raise the right side of the board 6-12 inches in the air.
  370. What is homeostasis?
    A. The body’s ability to maintain a temperature equal to the external temperature.
    B. The control mechanism is often referred to as a positive feedback loop.
    C. The act of movement from higher concentrations to lower concentrations.
    D. The relative stability and constancy of the body’s internal environment.
    D. The relative stability and constancy of the body’s internal environment.
  371. A 67 year old male is complaining of crushing chest pain. Vital signs are BP 110/60, P 80, R 20. SaO2 92% Your 12 lead ECG shows S-T elevation in leads 2,3, and aVF. What should you administer?
    A. Oxygen
    B. Oxygen, Aspirin, Nitroglycerin, and Morphine
    C. Oxygen, Aspirin, Morphine
    D. Oxygen and Aspirin
    D. Oxygen and Aspirin
  372. What does digoxin increase in the heart?
    A. Conduction velocity
    B. Heart rate
    C. Contractility
    D. Preload
    C. Contractility
  373. A 26 year old male underwent a recent surgery. He has fever, altered mental status, and decreased urine output. What should you suspect?
    A. Anaphylactic shock
    B. Neurogenic shock
    C. Hypovolemic shock
    D. Septic shock
    D. Septic shock
  374. You are preparing to deliver a woman’s first child at 39 weeks gestation. During your SAMPLE history, you are told the patient has a history of insulin-dependent diabetes. What should you suspect?
    A. The baby may be too large to deliver.
    B. The mother will be likely to hemorrhage.
    C. The mother will become hypoglycemic
    D. The baby will be hypoglycemic.
    A. The baby may be too large to deliver.
  375. You are to give Lasix 60 mg IV push. Lasix is supplied as 20mg/2ml. What volume of solution will you administer?
    A. 12 mL
    B. 6 mL
    C. 4 mL
    D. 10 mL
    B. 6 mL
  376. A 60 year old male with a head injury is moaning and making incomprehensible sounds. He withdrew when you started his IV but is otherwise not moving. His eyes are wide open. What is his Glasgow Coma Score?
    A. 13
    B. 3
    C. 6
    D. 10
    D. 10
  377. A 76 year old male is unresponsive. Family members state that he has been ill for several days with decreasing mental status. Vital signs are BP 60/30, P 136, R 36, BG 780. His skin is pale, cool, and dry. He has a history of Type II diabetes. What should you suspect?
    A. Thyrotoxicosis
    B. HHS
    C. DKA
    D. Adrenal insufficiency
    B. HHS
  378. Fractures of long bones may result in moderate to severe hemorrhage within the first 2 hours. As much as _ of blood may be released from a pelvic fracture.

A. 2000ml
B. 1550ml
C. 500ml
D. 1000ml
A. 2000ml

  1. During the management of a critical incident, who determines the strategies for the incident?
    A. Safety Officer
    B. Planning Chief
    C. Operations Chief
    D. Deputy Incident Commander
    B. Planning Chief
  2. A 50 year old female presents with narrow complex tachycardia and has the following signs and symptoms: respiratory rate of 22 and non labored, BP of 117/68, radial pulse rate of 176 strong and regular, and skin is pink, warm, and dry. What would be the most appropriate first treatment.
    A. vagal maneuvers
    B. synchronized cardioversion at 50 J
    C. synchronized cardioversion at 100 J
    D. Adenosine 6 mg fast IV push
    A. vagal maneuvers
  3. What level of hazmat protection includes turnout gear?
    A. Level B
    B. Level D
    C. Level A
    D. Level C
    B. Level D
  4. Which sector coordinates operations with the staging officer and routes patients to the hospital?
    A. Communications
    B. Command
    C. Transportation
    D. Triage
    C. Transportation
  5. Which structure would an endotracheal tube be most likely to enter should it be advanced too far?
    A. Right main bronchus
    B. Left main bronchus
    C. Esophagus
    D. Trachea
    A. Right main bronchus
  6. When a child presents with a high fever, stiff neck and petechial rash, what personal protective equipment should you use?
    A. Gloves and gown only
    B. Gloves only
    C. Gloves and mask only
    D. Gloves, mask and gown
    D. Gloves, mask and gown
  7. A 47 year old female complains of generalized weakness, nausea, and dizziness. She has been extremely thirsty over the last three days. Vital signs are unremarkable. What should you suspect?
    A. Diabetes insipidus
    B. Hypothyroidism
    C. Diverticulitis
    D. Grave’s disease
    A. Diabetes insipidus
  8. An elderly patient complains of weakness, nausea, and diarrhea. He has not felt like eating or drinking anything for the last few days. His breathing is regular, his lungs are clear, and he does not present with a fever. What is this patient is most likely suffering from?
    A. Hepatitis C.
    B. Dehydration.
    C. Pneumonia.
    D. Sepsis.
    B. Dehydration.
  9. What term is given to the number of times a woman has been pregnant?
    A. Primipara
    B. Parity
    C. Gravidity
    D. Gestation
    C. Gravidity
  10. Which of the following women is most likely in the first stage of labor?
    A. A woman whose contractions began approximately thirty minutes ago.
    B. A woman whose contractions occur five minutes apart and last sixty seconds.
    C. A woman who feels a strong pressure and an urge to bear down.
    D. A woman whose amniotic sac just ruptured, releasing a large amount of clear liquid
    C. A woman who feels a strong pressure and an urge to bear down.
  11. Following administration of a beta agonist to your emphysema patient, why are you likely to notice an increase in her pulse?
    A. Stimulation of the sympathetic nervous system.
    B. Stimulation of the parasympathetic nervous system.
    C. Direct and indirect alpha 1 and 2 stimulation.
    D. Decreased oxygen concentration and increasing hypoxia.
    A. Stimulation of the sympathetic nervous system.
  12. How much blood can a patient lose in the abdomen if left untreated?
    A. 10 L
    B. 1500 L
    C. 15 L
    D. 1000 L
    A. 10 L
  13. A 23 year old female gave birth to a boy. What is the correct way to stimulate him?
    A. Rub his head vigorously with a dry towel.
    B. Flick the soles of his feet and rub his back.
    C. Vigorously rub the child up and down.
    D. Gently squeeze his ear.
    B. Flick the soles of his feet and rub his back.
  14. What is the most common communicable disease contracted by blood-borne pathogen exposure?
    A. AIDS
    B. Hepatitis B
    C. Hepatitis A
    D. HIV
    B. Hepatitis B
  15. A 19 year old male struck his head against the platform while doing a reverse flip. Vital signs are BP 88/70, P 96, R 8 What type of shock is he in?
    A. Distributive
    B. Hypovolemic
    C. Hemorrhagic
    D. Cardiogenic
    A. Distributive
  16. What is the study of how drugs are absorbed, distributed, biotransformed, and eliminated known as?
    A. Pharmacokinetics
    B. Pharmacology
    C. Pharmacodynamics
    D. Physiology
    A. Pharmacokinetics
  17. A 67 year old female states that “she feels like she is drowning when she lays down to go to bed.” Her nitroglycerin has not helped. Vital signs are BP 160/96, P 110, R 26 with crackles. What should you do?
    A. Apply 3 lead ECG
    B. Administer 40 mg furosemide
    C. Apply CPAP
    D. Obtain orthostatic blood pressure
    C. Apply CPAP
  18. V1 is placed at the
    A. Fifth intercostal space to the left of the sternum
    B. Fourth intercostal space to the right of the sternum
    C. Fifth intercostal space to the right of the sternum
    D. Fourth intercostal space to the left of the sternum
    B. Fourth intercostal space to the right of the sternum
  19. What does a drop in blood pressure greater than 10 mmHg during inspiration indicate?
    A. Biot’s respirations
    B. Hypercarbia
    C. Pulsus paradoxus
    D. Hyperventilation
    C. Pulsus paradoxus
  20. Excessive urination is known by what medical term?
    A. Polyphagia

B. Glycosuria

C. Polyuria

D. Polydipsia
C. Polyuria

  1. A driver is sitting with her legs trapped between the dash and the floor. The roof is collapsed between the B and C posts. To free the patient, you should use
    A. cribbing, air chisel, and hydraulic cutter.
    B. airbags, K12 rescue saw, and chain.
    C. wedges, high-lift jack, and come-along tool.
    D. step chocks, reciprocating saw, and halligan bar.
    C. wedges, high-lift jack, and come-along tool.
  2. A 56 year old female responds inappropriately to your questions. Vital signs are BP 184/88, P 88, R 14. She has an asymmetrical smile, tearing in her right eye, and drifting of her right arm. What should you suspect?
    A. Bell’s Palsy
    B. A right sided stroke
    C. A left sided stroke
    D. Dementia
    C. A left sided stroke
  3. A 34 year old male has a left leg amputation. The injury is spurting bright red blood. Bystanders have unsuccessfully attempted to control the bleeding with direct pressure. What should you do?
    A. Apply a tourniquet above the damaged tissue and tighten until the bleeding stops.
    B. Remove the dressings placed on the wound by the bystanders to get a better look.
    C. Apply direct pressure with a gloved hand and elevate the extremity above the heart.
    D. Attempt to find the open artery and apply direct finger pressure until the bleeding stops.
    A. Apply a tourniquet above the damaged tissue and tighten until the bleeding stops.
  4. What does the P-R interval represent?
    A. Atrial depolarization and A-V delay
    B. Atrial repolarization and filling
    C. A-V delay and ventricular repolarization.
    D. Ventricular depolarization.
    A. Atrial depolarization and A-V delay
  5. What area of the brain is primarily responsible for respiration?
    A. Pons
    B. Medulla
    C. Hypothalamus
    D. Amygdala
    B. Medulla
  6. Which of the following terms best describes ventricular filling?
    A. Afterload
    B. Preload
    C. Systole
    D. Diastole
    D. Diastole
  7. What is the maximum amount of time allowed to complete endotracheal intubation?
    A. 10 seconds
    B. 20 seconds
    C. 30 seconds
    D. 15 seconds
    C. 30 seconds
  8. A 56 year old male tripped and fell into a manhole in the sidewalk. He complains of chest pain and tenderness when he takes a deep breath. Lung sounds are clear but diminished bilaterally. Vital signs are BP 140/90, P 90, R12. What is the most likely cause of his symptoms?
    A. Rib fractures
    B. Tension pneumothorax
    C. Flail chest
    D. Left sided hemothorax
    A. Rib fractures
  9. What signs and symptoms should make you suspect a heroin overdose?
    A. Pinpoint pupils, coma, and respiratory depression
    B. Dilated pupils, coma, and tachypnea
    C. Hypertension, tachycardia, and urinary retention
    D. Lacrimation, bradycardia, and hyperactivity
    B. Dilated pupils, coma, and tachypnea, def A though!
  10. “https://testing.fisdap.net/testing_images/EKG-Scans-016.jpg” (NS at 70 BPM)
    A 54 year old male complains of short bursts of chest pain when his internal defibrillator fires. You note he is internally defibrillated every 10 seconds. The ECG returns to the rhythm shown just before the patient is defibrillated. What should you do?
    A. Begin overdrive pacing with low joules setting.
    B. Obtain a 12 lead ECG and transport emergently.
    C. Monitor vital signs and sedative agent.
    D. Apply a magnet on the patient’s chest.
    D. Apply a magnet on the patient’s chest.
  11. A 54 year old male has burns over 60% total body surface. Which type of fluid should you administer?
    A. 5% dextrose in water
    B. Normal saline
    C. Lactated ringers
    D. Hextend
    C. Lactated ringers
  1. A 30 year old climber is experiencing sever shortness of breath after reaching an elevation of 12000feet. The PT is very anxious and has rales noted to all quadrants but is otherwise healthy. He is not accustomed to high altitudes. What should you do?
    Administer oxygen while descending to a lower altitude.
  2. A 26 year old female does not respond to verbal stimuli but moans in response to pain. A friend states the patient has ingested approx 150 amitriptyline tables. Vital Signs are BP 88/50 R16. You establish IV access and administer a fluid challenge. En route to the hospital her ECG shows a widening QRS. You should administer
    Sodium Bicarbonate.
  3. You PT experienced trauma yesterday and you are transferring her to a larger facility. She has been experiencing progressive hypoxia. You notice coarse crackles in all lung fields on ventilation. Vital signs are BP 90/50 and SPO2 90%. You should
    Transport emergency and continue ventilation.
  4. An alert 22 year old male was just disentangled from debris in a house collapse. He denies head or neck pain, but complains of server hip pain. Vital signs are BP 88/40. P120 R 16 you should administer.
    sodium bicarbonate
  5. Braxton Hicks contractions
    occur in the second trimester and are false labor.
  6. A 40 year old male crashed his motorcycle. He had a brief loss of consciousness, woke up and
    walked home. You find him 2 hours later at home unconscious. You should suspect.
    epidural hematoma
  7. A 47 year old male who was found unconscious on the side of the road respond only to painful stimulus. When you attempt to establish IV access he stiffens his arms and legs. He has minor abrasions all over his body. Vital signs are BO 178/122 P56 R irregular ETCO2 30. What should you do?
    Assist ventilations at 12 breaths per minHg.
  8. A 28 year old female has a sudden onset of tightness in her chest and server difficulty breathing. Her tongue face and neck appear swollen and she has red itchy hives on her torso. Her signs are BP 80/40 P 130. You should administer
    Epinephrine 0.3mg
  9. You are asked to set up a patient treatment area after a box labeled “radio active” was found unattended in a park. Which of the following would be the best location.
    Building in the park
  10. You are at a house fire with multiple patients. Which patient should be transported First to a burn center?
    52 year old with second degree burns to face and right hand
  11. What is the most common cause of status epilepticus in adults.
    Failure to take prescribed medications
  12. A 58 year old male with history of renal failure complains of dizziness lethargy and difficulty
    breathing. He says he missed his last dialysis appointment. Vital signs are BP 160/90 P 100 R 28 with rales heard in the basses. SPO2 92% you should suspect.
    Fluid Overload.
  13. A 12 year old girl is seizing. Her mom tells you that her daughter has been feeling tired for days. Her skin is dry and she has been very thirsty. Vital signs are BP 118/80 P115 R26 you should suspect.
    Diabetic ketoacidosis
  14. A 42 year old female has fallen down a flight of stairs. She appears anxious her skin is cool and clammy her radial pulses are weak at 112 beats per min and she complains of abdominal pain in the left upper quadrant with radiation to the left scapula. You should suspect.
    Torn or lacerated spleen.
  15. Emergency medical responders are ventilating an apneic medical patient with a bag valve mask. You note abdominal distention and minimal chest rise. What should you do?
    Reposition the head.
  16. A 6 month old male was found lying face down in the crib. He is cyanotic and apneic. A weak
    brachial pulse is present. You are unable to see a chest rise with your second attempt at rescue
    breathing after repositioning the head. What should you do.
    Start CPR
  17. You have just been called to a dialysis clinic for a 58 year old male in cardiac arrest. The online physician may order you to administer which of the following medications.
    Sodium Bicarbonate.
  18. A 27 year old male has been assaulted with a pen in the supraclavicular area. He is anxious and pale and his vital signs are BP 80/42 P 128 R 32 and shallow. First responders have sealed the wound with a bandage. Assessment reveals lung sounds on the right and poor chest rise what should you do.
    Burp dressing
  19. A 50 pound child is unresponsive pulseless and apneic. The cardiac monitor displays ventricular fibrillations. CPR is in progress. The initial defibrillator should be at.
    50 joules.
  20. An 11 year old male has been skateboarding in 100 degrees temperature and has been drinking caffeinated soft drinks. He complains of palpations. The cardiac monitor displays a narrow QRS tachycardia at a rate of 160 beats. Vitals signs BP 110/60 P160 R 20 What should you do?
    Transport the patient by ambulance monitor closely and administer fluids en route.
  21. An 80 year old male in server respiratory distress. He has a history of hypertension and breathing problems. He takes inderal daily. Vital signs are BP 170/70 P72 and irregular R40 and extremely labored skin warm and diaphoretic. Lung sounds are diffuse bilateral crackles and wheezing. He coughs up blood tinged sputum. Which of the following is most important.
    Decreasing preload
  22. A 50 year old patient is found lying outside on the ground. The ambient temperature is 40 degrees F. You thermometer reads 20C 86F. He is in V-fib and pulseless and apneic. You should
    Defibrillate once perform CPR intubate and transport rapidly.
  23. To successfully accomplish nasotracheal intubation you should advance the endotracheal tube though the glottic opening to coincide with patient
    Inhalation
  24. An alert 23 year old male presents with a sudden onset of shortness of breath and chest pain after running upstairs. Breath sounds are diminished in the right side. The most likely cause is.
    Spontaneous pneumothorax
  25. First responders are treating an apneic adult female. You notice that they are applying frequent long ventilations with an oxygen resuscitator. What effects with their actions create.
    Increased capillary elasticity.
  26. You are called to a nursing home on a hot summer day for a 88 year old male. The nurse tells you he missed his dialysis appointment because he was complaining of nausea vomiting headache and running a fever. Which differential diagnosis should you consider first.
    Sepsis and hyperkalemia
  27. You have just administered adenosine to a 62 year old patient with narrow complex tach. The ECG shows an irregular rate of 50 then steadily climbs back to a rate of 169. What would should suspect
    rapid atrial fibrillation.
  28. A 63 year old male is having a diabetic emergency . He is unresponsive R12 glucometer 28. You cannot establish IV access. You should
    administer glucagon IM
  29. When setting up a night time landing zone for a helicopter what is the recommended area needed.
    100×100
  30. A 67 year old with hypo-tension and bradycardia has no responded to pacing or atropine. You should
    administer dopamine infusion
  31. A 19 year old male has suffered blunt trauma on his left chest. He complains of server dyspnea and pain over the affected area. Vital signs are BP 92/60 P 128 R 32. Breath sounds are very diminished over the left chest. You should
    perform needle thoracostomy
  32. A 44 year old patient has a persistent cough and occasional spits up bloody sputum . He has been living in a a shelter for the past three weeks and he smells like alcohol you should.
    Put on N95 mask and eye protection.
  33. While treating a patient with suspected TB. What type of personal protective equipment is essential
    HEPA respirator
  34. A 2 year old male has been sick over the last 48 hours with a low grade fever runny nose and upper respiratory infection. He has a seal bark cough. The patient is using accessory muscles to breath and appears fatigued his upper airway has stridours sounds. His lung sounds appear clear. SPO2 92% how would you treat the patient.
    Oxygen nebulized racemic epinephrine transport.
  35. 78 year old male lying unresponsive in his bed vital signs are BP unobtainable. R 12 and normal depth radial pulse rate matches ECG below ECG bradycardia
    Perform transcutaneous pacing.
  36. A 48 year old patient is chocking on a hot dog. He is unresponsive apneic and has a strong pulse. Chest compression are infective at relieving the obstruction. You should.
    use lagosopy and magill forceps to remove the object.
  37. 87 year male is complaining of dizziness. Physical exam revals pale cool and diaphoretic skin. Pulse matches ECG bradycardia. You should
    administer atropine
  38. A 14 year old patient was pulled form a indoor pool. She is pulseless and apneic. You should
    perform CPR endotracheal intubation establish an IV and administer epinephrine
  39. A trauma patient has lost approx 500ml of blood the amount of crystalloid solutions needed to replace this blood loss is
    1500 ml of saline ( 3 to 1 rule)
  40. A 43 year old female has been stuck in the chest with a baseball bat. There is a large bruise over her sternum. She is obversely in pain an states her heart feels like it is beating funny Vital signs are BP 109/80 P 124 and irregular R 22 The ECG show sinus tachycardia with ST elevation and frequent PVC’s. You should suspect
    myocardial contusion
  41. A 28 year female is speaking very quickly and stating that she is going see the president of the united states after she buys all her family members a new car. She has not slept in four days because she does not need sleep. She usually takes lithium but ran out of her prescription about a week ago . vitals 144/88 P 128 R 28 you should suspect.
    manic phase of bipolar disorder
  42. A restrained 23 year old pregnant female is her third trimester was involved in a head on motor vehicle collision. She complains of abdominal pain. initial vital signs are BP 110/70 P 90 R 20. After rapid extrication and spinal immobilization via long spine board her vital signs are BP 90/40 P 110 R 28 what is the most likely explanation for her change in vitals.
    Supine hypotensive syndrome
  43. A 3 month old infant who weighs 6kg has a history of congenital heart disease. Vital signs are R60 the ECG hows a narrow complex tachycardia at a rate or 240 cyandic nail beds and spo2 at 88% what should you administer.
    0.6 or adenosine IV
  44. A 44 year old male fell from a ladder and is now complaining of pain in the middle of his back at the level of his shoulders blades. He responds to verbal stimulus and vitals signs are BP 70/50 P64 R 20. The patient is most likely suffering from
    neurogenic shock
  45. A 63 year old female chocked on a piece of steak and is apneic. You partner has visualized a partial obstruction in the trachea. He has been trying to grab it with forceps for more than 2 minutes. What should you advise him to do?
    Remove the blade and attempt to ventilate.
  46. An unconscious 13 year old patient has minor abrasion on his torso after being struck by a vehicle. The child’s parents are not on scene law enforcement and an EMS supervisor are on scene You should?
    transport the patent to hospital
  47. A 38 year old female is crowing. You note the amniotic fluid contains a substance resembling pea soup. An apneic blue limp infant with a heart rate of 80 beats is delivers. How should you treat the infant.
    Suction with a bulb syringe and ventilate.
  48. A 56 year old female has a sudden onset of substernal chest pressure. She denies any shortness of breath or nausea. She has a history of arthrosporic heart disease and is talking verapamil. Vital signs are BP 160/90 pulse matches the ECG rhythm below R 16 skin warm and dry. and lungs clear bilateral. What should you do.
    Nitroglycerin
  49. A 64 year old is sitting in his living room in moderate distress. He is able to speak to you in full sentences but claims he is not feeling right. Vitals signs are BP 1110/64 R 16. Pulse matches monitor (super vertical tachycardia) what should you do.
    vagal fluids adenosine.
  50. Which I V will be able to administer the most fluid in the least amount of time.
    14 gauge 1/2 inch IV catheter.
  51. A 30 year old man is breathing fast and deep. He works at a preschool and believes that the
    children plan to harm him. Which of the following statement is most helpful when treating the patient.
    I understand that you are concerned. Can you tell me more about why you fell this way.
  52. A 67 year old man has collapsed and is on the floor. He is pale clammy and moaning with vomit around his mouth. Vital signs are BP 180/30 P110 and weak R28 and shallow. Oropharynx is clear and lungs are clear bilaterally. His ECG shows ventricular tachycardia. That should you do?
    Synchronized cardio100J
  53. You arrive at the scene of a single vehicle crash. A woman walks up to a your ambulance with a cut on her forehead. You should
    determine if she is the only victim
  54. A 73 year old male has several fractured ribs on his right side with unequal chest wall movement. He has dyspnea and diminished breath sounds on the right. Vital signs are BP 124/76 P124 R 26 and shallow. His trachea is mid line and neck veins are flat. Most likely has
    a flail chest segment
  55. You have just converted a 318 lb cardiac arrest patient into a normal sinus rhythm. She is
    hypotensive and you have been ordered to start the an epinephrine drip at 3mcg/min using a 60 drip set. You have mixed a 4:1 concentration of epinephrine and should set the drip rate at
    45 drops/min (clock)
  56. You are assisting with a deliver and the infants head has been deviled. You notice the umbilical cord is wrapped around the infants neck. You should
    gently slip the cord over the infants head or clamp the cord in two places and cut between the clamps to
    release the cord.
  57. A 50 year old male is complaining of a chronic cough. He has lost his appetite and has had hot flashes and periods of sweating during the night. He has noticed some red specks in his sputum when he coughs. You should suspect
    Tuberculosis
  58. A patient has surfaced rapidly from a deep dive and is now complaining of difficultly breathing and extreme muscle and joint pain. You should
    Transport rapidly to the nearest hyperbaric chamber.
  59. It is mid-December and a 13 month old child is awakened in the middle of the night with barking cough and inspiratory stridor. The child has been ill for a few days with chills and congestion. You should suspect
    croup
  60. An 80 year old male has just collapsed. Vital signs are BP 90/40 pulse matches ECG 3 degree block R 20 What should you do?
    Apply transcutaneous pacing.
  61. A patient involved in a motor vehicle crash has an obvious open femur fracture and an angulated right forarm. He is bleeding servery from a scalp laceration. His left arm is guarding his chest and he gasps “i cant breath”. You should be MOST concerned by the
    respiratory difficulty
  62. A 37 year male is chocking and has become unconscious. Attempts to ventilate are unsuccessful. you should
    perform chest compressions
  63. An unconscious 78 year old male post cardiac arrest is being actively ventilated with a bag valve mask. Initial vitals are BP 98/40 P 82 and irregular. After several minutes reassessment of vital signs are etco2 22 BP 68/28 and irregular spo2 92 you should
    decrease the rate of respirations
  64. An 86 year old female presents with a rhythm in the ECG she is immobile due to server arthritis and has a history or atherosclerosis and hypertension. What should you suspect
    Left ventricular hypertrophy LVH
  65. A 37 year old suicidal male is bing violent. Police has pinned him in a prone position but he
    continues to struggle and fight. You should
    sedate the patient and place him supine on the stretcher in soft restraints
  66. The termination of pregnancy for maternal health reasons is called a
    therapeutic abortion
  67. While using the START system at a major incident you encounter a patient not breathing. After positioning the airway the patient breaths 16 a min. You should catagorize him as?
    Immediate
  68. You are transporting the victim of a major trauma which occurred yesterday to a larger facility. This patient has been experiencing progressive hypoxemia. You hear coarse crackles blatantly during auscultation of the lungs and the pulse oximetry reading is falling in spite of the patent being intubated. You should suspect
    that acute respiratory distress syndrome has developed
  69. A 19 year old female is 25 weeks pregnant. She has server headache blurred vision and edema she has not regularly seen a doctor during her pregnancy. What other symptom should you expect.
    hypertension
  70. Which of the following is a cause of multifocal atrial tachycardia
    Electrolyte imbalance
  71. After receiving Narcan. A patient is waking up quickly and pulling at the airway you have placed. Before removing it you should
    prepare to begin suctioning
  72. A 66 year old male complains of server dyspnea. His has breath sounds on the right and diminished on the left. He has a long history of cigarette smoking. Vital signs are BP 80/30 P124 R 26 and shallow SPO2 88% on 4 lpm What should you suspect?
    Emphysema exacerbation
  73. A patient is complains of left calf pain. The left leg is cool to the touch and paler than the right leg. You should suspect
    Arterial occlusion
  74. After placing a combitube bilateral breath sounds are present and chest rise is equal. You notice quiet gurgling in the stomach with each ventilation. You should.
    add air to the distal cuff
  75. A seven month old is breathing at a rate of 60 per min. He has nasal flaring and intercostal
    retractions . You hear stridorous noises on inspiration . This is most likely caused by a
    foreign body obstruction the upper airway
  76. A 28 year old suffered a head injury. Vital signs a BP 140/120 P 55 R 30 and irregular. Which of the following signs would you expect to see in this patient.
    Decerebrate posturing
  77. During transportation you PT becomes very upset complaining about her medical expenses. How should you respond?
    You seem angry is there any way hat i can help
  78. Which of the following pharmacological agents are most likely to cause premature arterial
    contractions.
    stimulants
  79. You are dispatched to a house early on the morning for a family complaining of headaches and nausea. You should
    evacuate the house.
  80. A patient is threatening suicide because his wife left him. Which of the following statements or questions would be appropriate?
    What method had you intended to use to kill yourself
  81. A 44 year old male complains of extreme fatigue headache and fever with pain in this right upper quadrant and joints. He admits to a history of alcoholism and smoking two pack per day for the past 20 years his skin and sclera are yellow in color. You should suspect
    hepatitis
  82. A patient was just pulled from a burning building. You have started oxygen and initiated transport. While en route to the hospital you notice singed hairs around his face and a spot around his nose and mouth.. He is coughing up black sputum mixed with blood and shows signs of increasing hypoxia. Which developing condition is your biggest concern.
    Laryngeal edema
  83. A 65 year old female complain of sudden onset of server migraine headache. Vitals signs are BP 140/90 P 120 R 22. She has been non complaint with her prescribed imitrex. You should administer
    morphine
  84. Which of the following will cause a patient with a tension pneumothorax to become hypotensive
    an increase in intrathoracic pressure
  85. A 23 year old male complains of chills and headache. He was sleeping outside before waking up with these symptoms. He has a reddish ring with a small fluid filed vesicle in the center on his arm. What should you suspect
    Spider bite.
  86. Which of the following is most important when document a patient refusal
    The patient’s mental status
  87. You have orally intubated an adult pt and are evaluating the tube placement. You hear auscultation sounds over the epigastrium and an absence of breath sounds over the right and left chest. You should
    remove the ET and ventilate the patent with a bag valve mask
  88. Your elderly patent is supine. She complains of labored respirations. She is cyanotic and has a history of CHF vitals signs are BP 230/100 P120 R 36 What lung sounds would you expect to hear.
    Crackles
  89. You are treating an unresponsive elderly male from a nursing home. He has a surgical incision on his right hip that is is red and warm to the touch. You notice widespread deep purple bruising on his body. Vitals sings are BP 80/40 P116 R 10 spo2 89%. You should suspect
    disseminated intravascular coagulation
  90. When arriving on a potential HAZMAT incident on the highway you should stage the emergency apparatus.
    Uphill and upwind
  91. After delivery of a newborn she cries weakly with stimulation and moves all extremities P140 R 18 and irregular her body is pink but the extremities are blue. What is her APGAR.
    7-8
  92. A 21 year old male is intoxicated he was struck in the head with a bat 3 hours prior to arrival. He has suddenly began acting aggressive towards his friends. A police officer on scene states ” i want to arrest him” you should
    suspect hypoxia and head injury and transport the patient to a hospital
  93. A 5 year old has a sudden onset of high fever score throat and pain on swallowing. His voice is muffled and he is drooling. You should suspect
    epiglottis
  94. A 19 year old intoxicated female is distraught after being involved in a MVC she is complaining of pain in her head and multiple superficial facial lacerations. Vitals 88/40 P 128 R 28 you should suspect
    neurogenic shock or Hypovolemia do to bleeding from other injury’s
  95. You are working during inclement weather in a remote location. The nearest hospital is more than two hours away. Your patient just delivers a healthy baby boy but has inverted her uterus. What orders you expect from an online physician
    Replace the inverted uterus with a gloved hand
  96. A 67 year old female with a history of diabetes complainants of nausea and began vomiting an hour ago. She denies any pain her blood glucoses is 208 her skin is cool and pale vital signs are BP 146/88 R 24 P 126 and irregular. What should you suspect
    Myocardial infarction
  97. A 34 year old female crashed her motorcycle while wearing an open face helmet. She is
    unconsciousness her facial bones and soft tissue are missing from the nose to the upper part of her neck. You are having difficulty ventilating her and you can see bubbles coming from the center of a large mass of tissues when she exhales. You should
    insert an ET at the site of the bubbles
  98. A 14 year old female is crying and complain of knee pain after a soccer injury. The knee is
    deformed and obviously angulated. Distal pulses are intact. You should
    apply ice split in position found and transport
  99. A 53 year old man fell from a ladder. he has multiple abrasions and localized swelling on his head. You have oratorically intubated him. His ETOC2 is 30 his pupils are midpoint and reactive how many times per minute should you ventilate
    10
  100. A patient involved in a fist fight. He complains of point tenderness in the right upper abdominal quadrant. You should suspect injury to the
    liver
  101. Successful intubation of a newborn is more successful by placing padding under the
    shoulders
  102. For the past 5 min you have been providing ventilations to an intubated patient and performing CPR it is becoming progressively more difficult to ventilate and the chest is moving less during inspiration. What should you suspect
    tension pneumothorax
  103. An elderly pt is complaining of dyspnea that woke her from sleep about an hour ago. She denies chest pain. Auscultation reveals wheezing and crackles in both lower lobes of her lungs. Vitals signs are 158/108 P 128 and irregular R 32 and slightly labored what should you administer
    albuterol
  104. A 59 year old pt is lethargic with shallow respiration he has a valid DNR order during transport his respiratory rate diminishes to 8 breaths a min you should
    begin assisting with a bvm
  105. a 89 year old nursing home pt is in respiratory distress. She is on 3 lpm of oxygen by nasal
    cannula. She has a history of COPD asthma and diabetic and you note wheezing in all lung fields. Vitals BP 146/84 P 114 R 24 you should
    place her on 15 lpm or oxygen via nonrebreather mask
  106. you are performing Orotracheal intubation with a mac blade. You can see the cords when you notice the top of the blade is in the vellicula you should
    pass an ET through the cords
  107. A confused 34 year old male is ambulatory ofter crashing his car. Airbags were deployed he complains of extreme wrist pain and double vision . You note deformity and superficial burns on his writs what should you do
    cspine

110 A patient with server closed head injury vitals 180/112 P 48 R 8 and irregular immediate treatment should include
intubate and ventilate at 20 breaths per min

110 A 22 year old pregnant patient is experiencing a significant amount of painless bleeding shorty after sexual intercourse. The most likely cause is
placenta previa

  1. An elderly patient has been ill for last few days and has been vomiting. He reports chest
    discomfort and is now pale and diaphoretic. Vitals BP 130/70 R24. ECG shows (afib RVR) You
    administer oxygen and now have an IV you should.
    Fluid bolus
  2. Signs and symptoms of hypoglycemia may mimic which of the following conditions
    CVA
  3. A patient complains of server right upper quadrant pain. Right scapula pain and nausea after eating a cheeseburger you should suspect
    cholecystitis
  4. Your patent is suffering from exacerbated emphysema which of the following medications is least likely to be a part of his standard therapy
    nitrates
  5. A 7 year old has a suspected broken arm and numerous bruises what finding would lead you to suspect that a mothers account of the injures may not be true.
    You note purple yellow and green bruises ( indicates different stages of healing)
  6. A 19 year old female collapsed in the top bleaches during a concert. She is unresponsive and does not appear to be breathing. how should you asses her respiratory status?
    place a hand on the chest
  7. Followng a cardioversion of a 72 year old female she coverts to the rhythm bellow ( 3 degree) you identity this as
    third degree heart block
  8. A 13 year old has been found unresponsive. Her mother proves you with a prescription bottle of Oxycontin. What should you give?
    narcan

119 A man was involved in a lateral impact auto accident that deformed the vehicle. The patient could have sustained a?
traumatic aortic dissection

120 You are assisting with vaginal delivery. The infant head has been delivered and the mouth and nose suction. To deliver the anterior shoulder you should
gently guide the infants head downward

  1. A 22 year old female 22 weeks pregnant is experiencing abdominal discomfort during evaluation you notice the umbilical cord is pressing the vaginal canal. How should the patient be positioned
    supine with legs open
  2. Which of the following situations constitutes abandonment?
    A patient is unattended in the ambulance while loading a second patient
  3. A patient is arching his back and rolling his eyes upward. He was recently prescribed compazine. How should you teat this patient?
    25 mg benadryl IVP
  4. A 38 year old male has been sick for a few days he is now complaining of sharp mid chest pain vitals 118/69 pulse 124 and regular and R 24 and slight laberod temp 102. You should suspect?
    pericarditis

125 46 year old male is alert oriented and complains of a headache. He has tremors in his hands he has been in the detox unit for 32 hours . Vitals signs are BP 210/100 P100 R 25. He has a history of alcoholism and seizures. He takes tegretol you should administer
20mg diazepam

  1. You are reliving a crew form a 24 hour shift before you compete the ambulance check you are dispatched to a cardiac arrest with CPR in progress. what should you do before leaving for the call?
    ensure essential equipment is present and the ECG monitor is functional
  2. A pregnant patient is experiencing contractions upon examination you notice that the infantst head is bulging from the premium and the patent has the urge to push what stage of labor is this?
    two
  3. you are dispatched to a playground for a patent with difficulty breathing. Upon arrival you find a five year old male who appears anxious with noted cyanosis and intercostal retraction BP 85/pb P112 R24 with swelling of the upper airway. You should suspect
    anaphylaxis
  4. A 28 year old female is 34 weeks pregnant and experiences sudden sharp tearing pain in her abdomen and developers a rigid abdomen what should you suspect
    abruptio placenta
  5. After a prehospital delivery the mother should be allowed to breastfeed her newborn because
    breast feeding helps constrict the uterus and reduce bleeding
  6. A two year old male has been ill for several days with a respirator tract infection. Yesterday he developed server headache high fever and extreme lethargy. You should suspect.
    meningitis
  7. When starting an external jugular IV the needle should point
    toward the feet
  8. while using the backwards upward right pressure BURP maneuver during an orotracheal
    intubation you patient begins to vomit. What should you do?
    Stop the intubation attempt and suction
  9. You have just assisted with the delivery of a 32 week old infant. She is apneic and has a pulse of 64. What should you do?
    open the airway ventilate with a bag valve mask for 30 seconds and reassess
  10. During a yearly heath screening a clinic discovered your patient has the heart rhythm shown below (bradycardia). She alert and oriented but scared because the clinic call for an ambulance Vital signs bp 114/50 R12 spo2 99%. You should administer
    atropine
  11. A 18 year old male was 40 miles into a long distance bike race when he had a syncopal episode and fell off his bike. He responds to painful stimulation BP 60/40 P160 skin cool and diaphoretic. You should administer?
    lactated ringers

137 An 18 year old male suffered second degree burns to his entire head and neck anterior chest and both arms while removing a radiator cap from his overheated engine. Using the rule of nines what percentage of his body is burned?
36%

138 You are unable to ventilate a trauma patient with extreme facial trauma using advanced airway maneuvers. You should
insert a 14 g needle in cricothyroid membrane 45 degree angle.

  1. Which of the following is a prehospital medication given for compartment syndrome
    sodium bicarbonate
  2. which of the following would be an indication for the use of needle cricothyrotomy?
    Inability to ventilate
  3. A pale diaphoretic patient complains of chest pain at a level of 6 on 1-20 BP 132/88 you see the following ECG (ST elevation) You have established oxygen and an IV you should give?
    ASA and admin nitroglycerin
  4. Identify this rhythm
    paced rhythm

143 A 25 year old female was bitten by something while hiking 12 hours ago. She has slurred speech double vision and excessive salivation her boyfriend states she had a seizure about 20 min ago. You should:
thoroughly wash and immobilize the wound and start an IV

  1. An obese 77 year old woman called EMS complaining of dyspnea cough with hemoptysis and diaphoresis. You note tachypnea tachycardia crackles and wheezing. She also has varicose veins. Find significance differences in BP between the right and left arms. You should suspect.
    aortic aneurysm
  2. A 76 year old patient is lying in soiled sheets with diarrhea and vomit . Both the stools and emesis look thick black and tarry similar to coffee grounds What should you suspect?
    upper gastrointestinal bleeding
  3. Your 3 lead ECG is shown below. You correctly identity the rhythm as?
    1st degree
  4. A six year old female has suffered blunt trauma to her right upper quadrant . She responds to painful stimulation vitals signs 50/30 P132 R 32 she is in?
    hypovolemic shock
  5. You are first on scene of a crash. Where should you park your vehicle.
    place the ambulance to block traffic
  6. A 50 year old male at a mountain resort has begun to experiencing shortness of breath headache and dizziness. Lung sounds reveal inspiratory crackles. You should suspect
    high altitude pulmonary edema
  7. A seven year old is having difficulty breathing. He is lethargic with central cyanosis and poor muscle tone. Vitals Signs are P 70 and R 10. You should suspect?
    Respiratory failure
  8. An 84 year old male has extreme difficulty breathing apprehension cyanosis and diaphoresis. Assessment findings include elevated pulse and blood pressure and rales and rhonchi you should treat this patient for?
    left ventricular failure
  9. Online medical direction has ordered you to administer 13 mcgkf/min of dop to a patient who weighs 190lbs. Your concentration of dopamine is 1600 mc/mi you are going to administer this through a Micro-drip tubing. At how many drops per minute should you set the IV infusion
    42 gtts/min
  10. You have just given a bolus of dextrose to an unconscious female patient with a blood glucose level of 30 . The repeat blood glucose level is now 120. She is now alert and oriented and she refuses further treatment of transport. What should you do?
    ask a neighbor to monitor the patient after you leave.
  11. Ten workers were exposed to fluorine gas fumes but only two exhibit symptoms. One additional worker was exposed to hydro-fluoric acid on his skin. He is being decontaminated with water by the company’s response team. Two ambulances are on scene . You should plan and prepare to?
    order a full scale mass casualty response including eleven ambulances and additional fire and law enforcement agencies
  12. A 54 year old male has fallen and is holding pressure on his abdomen with bloody kitchen rag. He has had abdominal surgery last week and his incision has ruptured and the abdominal contents are protruding. Vitals are BP 100/68 P 124 R 20 what should you do?
    remove the rag apply a moist sterile dressing
  13. You have just been stuck with a blood contaminated needle. You should you immediately deal with the exposure?
    wash the affected area with soap and water
  14. A container of hydro-flouric acid has spilled at a local metal plant. The hazmat team is responding and you are asked to stage at the front gate of the business where the company safety manger is waiting for you. What info would ou expect his manger to have
    material safety data sheet MSDA
  15. A 33lbs child is experiencing fever poor appetite purpura and vomiting. His cap refill is 3
    seconds he is tachycardic you should administer
    300ml of crystalloid solution
  16. Your crew is the first unit to arrive for a suspected radiation emergence after a box labeled recitative material was found unattended in a park. What three principals should you follow?
    time distance and shielding
  17. A 19 year old male has suffered a significant blunt trauma to his left anterior chest. He is in obvious respiratory distress vital signs are BP 80/p P 140 R 32 lung sounds are diminished and there is dullness to percussion over the left side of the chest. You should suspect
    hemothorax
  18. Side effects of oral contraceptives include
    pulmonary embolism and stroke.
  19. You are first on scene of a car crash. You partner stops the ambulance a safe distance from a single vehicle that crashed into a concrete barrier. Black smoke is coming from underneath the hood. The vehicles only occupant screams for assistance you should?
    rapidly extricate the occupant without a cervical collar
  20. A 50 year old female is experiencing chest pressure her ECG is ( sinus tack)
    she becomes unresponsive while you are obtaining iv access her ECG changes to the rhythm below. What should you do?
    asses for a pulse
  21. A 50 year old female is experiencing chest pain then goes into ventricular fibrillation. What should you do?
    Defibrillate

165 The medications prepreanolol atenolol and metoproloe are examples of?
beta adrengic blockers

165 A 17 year old male alert was struck in abdomen with large wrench vitals 90/78 P128 weak R 22 skin is pale and cool and diaphoretic. The abdomen is bruised and rigid you should?
rapidly transport to a level one

  1. A 14 year old male is found in an intoxicated condition. His friends report he admitted to drinking alcohol taken from his fathers workshop. The container is marked methanol not for human consumption. Definitive treatment for his condition is administration of
    ethanol
  2. A 74 year old patent is short of breath he has had a slight fever since yesterday and he suffers from frequent urinary track infections. Lung sounds are present with rhonchi and slight crackles in all fields. Vitals 110/80 P100 R30 temp 102 you should suspect?
    pneumonia
  3. The ECG of a post cardiac arrest adult shows sinus tachycardia at a rate of 120 with bp 60/40 you should give?
    dopamine 5-20

169 One year old has been ill with multiple episodes of emesis and diarrhea. Asses shows a lethargic 35 pond child who has mottled skin dry mucous membranes cap >2 sounds P 220. After securing airway you providing high consultation oxygen and establishing an IV you should?
give 320 cc fluid bolus

  1. A 79 year old female complains of labored respirations she is cyanotic and has a history of CHF identify the following rhythm (sinus tach) and appropriate treatment
    sinus tachycardia administer nitroglycerin
  2. A patient who complains of pain in left upper quadrant is most likely suffering from
    pancreatitis
  3. A 28 year old woman complains of sudden onset severe abdominal pain that radiates to her shoulder 88/60 P 100 R 20 skin cool pale and clammy states her last menstrual period was 8-10 weeks ago you should suspect
    ectopic pregnancy
  4. You have just fished a stressful shift. you are frustrated and angry with how some of your calls resolved you should
    talk to an internal critical incident stress counselor
  5. A 78 year old female is lying supine in an nursing home bed with labored respiration she is semi-conscious cyanotic and the nurse tells you she has a history of CHF 80/40 spo2 90 pulse is ventricular tachycardia you should?
    perform sync cardiovert

175 An adult has a foot laceration from a farming accident he is alert and refusing treatment his religious beliefs do not permit medical you should
document the patient wishes

  1. A 50 year old female tripped down four steps. Her airway is patent radial P 100 R 16 what other initial information do you need to decide if she is unstable?
    mental status
  2. Management of all newborns includes
    prevent heat loss opening the airway and evaluating of the infant

178 A 6 year old female has suffered blunt trauma to her upper right quadrant she responds to painful stimulation BP 50/30 P110 you should most likely suspect damage to her?
liver

179 A 12 lead show 4mm of ST elevation in leads II III and avf you should suspect?
an inferior wall MI

  1. A 58 year old male is complaining of swollen legs and feet and increased heart rate. you note distended neck veins and abdominal distention you should suspect
    right sided heart failure
  2. In what hazardous material zones does triage and treatment occur
    cold
  3. An 87 year old patient with recent history of hematuria and lower abdominal pain now presents with altered mental status her physical exam is otherwise unremarkable. She is resident of adult foster home. A likely cause is?
    urinary tract infection
  4. A 26 year old male is experiencing dyspnea after being struck in the chest during a bar fight. He has a history of asthma and has taken his inhaler without relief he has busing to the right side of the rib cage with crepitus P128 R24 with clear bilateral lung sounds spo2 86% what should you do?
    assist ventilations with a bag valve mask

184 You have just intubated and adult and you have been successful you hear no sounds in epigastric and equal lung sounds you should?
attach etco2 monitor

  1. A 50 year old female has just been struck by a car she is alert and complains of shortness of breath. You note crepitation and paradoxical movement in her right chest R 26 spo2 89 you should
    augment ventilation with positive pressure
  2. While treating a patient for a cardiac event, you forget to administer aspirin. The patient suffers a MI and subsequently dies. The family is suing you for negligence, which element of negligence still needs to be proven?
    proximate cause
  3. A patent has sustained a soft tissue injury to the face you main concern is?
    managing the airway

188 Your partner started an IV and dropped the needed. The exposed needle is wedged between the stretcher bracket and crack on the tile you should?
use a needle nose clap to properly dispose of the needle

  1. Which of the following medication is most likely to be prescribed for a server resp conditions
    ipratropium
  2. You should use lactated ringers and normal saline if you goal is to deliver a
    isotonic what will remain in the intravascular space longer than other crystalloids
  3. You have been assigned to triage at an MCI a patent complains of small laceration on his forearm R24 and absent radial pulses using start triage you should assign this person as?
    immediate
  4. A 26 year old female has been involved in an explosion at a chem lab. She has first and second degree burns to her face. Witnesses state that her shirt had been on fire and stopped burning you should suspect?
    extensive airway burns
  5. A 3 year old female has surfed burns to both her hands after grabbing a hot exhaust pipe the injured area shows dark chard skin. These burns should be considered?
    critical
  6. A 200 pound woman is having difficulty breathing which of the following devices is most appropriate for carrying her down stars.?
    stair chair

195 A patient has polymorphic ventricular fibrillation you would give?
mag

196 A 65 year old female is short of breathe clear lungs and no jvd she has a significant cardiac history what should you admin?
oxygen and 12 lead

197 An 18 month has a seizure for first time and according to the parents she has become increasing ill over the last few hours and her current temp is 104 they describe the patent arms curling toward her chest and her legs straightening and pointing together. The parents do not want to go to the hospital tonight. You should?
have the parents speak with an online physician

  1. A 44 year old female has sustained server blunt trauma to the chest she responds to painful stimulation and her vitals are 88/76 P 130 R 24 muffled heart sounds and flat neck veins. The definitive treatment for this patient is
    pericardiocentesis
  2. A 12 year old becomes ill yesterday and has become progressively sicker. His temp is 103F and he has small specks on his chest the patient appears to be unable to turn his head you should suspect?
    meningitis

200 A 65 year old male with emphysema is sitting in a chair he is lethargic and struggling to keep his head up his skin is ashen vital sings a {P 134 R 30 and shallow what should you do?
assist ventilation with a bag valve mask

Leave a Comment

Scroll to Top